Matemática Aplicada

Descargar como pdf o txt
Descargar como pdf o txt
Está en la página 1de 126

NUMEROS COMPLEJOS – FUNCIONES ELEMENTALES

1. EL CONJUNTO DE LOS NÚMEROS COMPLEJOS. LA UNIDAD IMAGINARIA

Existen varias operaciones que no tienen solución en el campo de los Números Reales (R) por
4
ejemplo: √−64 ó ln (−2) , así como determinadas ecuaciones, por ejemplo: x 2 +9 = 0 ó

senx = 8 . Sin embargo todas estas expresiones tienen solución en un conjunto de números
denominado Números Complejos C el cual incluye a los Reales. El conjunto de los Números
Complejos se forma mediante la unión de los Números Reales con los Números Imaginarios (Img):
ℂ = ℝ ∪ Img

El Conjunto de los Reales, así como el Conjunto de los


Imaginarios están incluidos en el Conjunto de los
Complejos, es decir, son Subconjuntos de éste último.
ℝ ⊂ ℂ Img ⊂ ℂ

y se lee: R está incluido en C; Img está incluido en C.

Entre el Conjunto de los Reales y el de los Imaginarios,


no existen elementos en común: ℝ ∩ Img = ∅

y se lee: R intersección Img es igual a Conjunto Vacío.

El conjunto de los Números Imaginarios surge a partir de la aparición de la Unidad Imaginaria,


designada generalmente con i ó j (usaremos i) definida como:

i2 = −1 → i = √−1 DEFINICION UNIDAD IMAGINARIA

La unidad imaginaria es un número que al ser elevado al cuadrado resulta igual a -1, cumpliendo
así con una propiedad NO verificada por ningún número Real, ya que las potencias pares de estos
últimos, siempre resultan positivas. Desde el momento que apareció en escena un número cuyo
cuadrado es -1, fue posible resolver las operaciones/ecuaciones que no tenían solución en el
campo de los Reales, por ejemplo:

MATEMATICA APLICADA - Números Complejos - Funciones Elementales – Cursada 2020/2021. 1


x 2 +9 = 0 → x 2 = −9 → x = √−9 = √(−1) 9 → x = √−1 √ 9 = ±3 i

El descubrimiento de la unidad imaginaria permitió no solo resolver ecuaciones como la anterior,


sino también obtener el logaritmo de números reales negativos y varias cuestiones más, que
iremos viendo.

2. REPRESENTACION DE NUMEROS COMPLEJOS


2.1 Forma Cartesiana
Los números complejos se representan generalmente con las letras minúsculas z y w. Todo número
complejo se puede expresar como la suma (algebraica) de una componente real y otra
componente imaginaria y ésta última multiplicada por la unidad imaginaria:
z = a+bi FORMA CARTESIANA

donde a y b ϵ R. Las componentes real e imaginaria de z se designan como: Re(z) = a; Img(z) = b.


Es decir, ambas componentes son números reales; es la forma más utilizada para expresar los
números C.
Se denomina cartesiana porque las componentes real e
imaginaria, se representan en el plano cartesiano, en los ejes
de abscisas y ordenadas, respectivamente. También se puede
considerar que (a, b) son las coordenadas de un punto y así
el número complejo z, es el vector con origen en (0,0) y
extremo en (a,b). Cuando la componente Real o la Imaginaria
son nulas, el número z se denomina imaginario puro o real
puro, respectivamente. Por ejemplo: z = -2+7i → Re(z) = -2 ;
Img(z) = 7; w = -9i → Re(w) = 0; Img(w) = -9
FIGURA 1

Dos números complejos z = a+bi ; w = c+ di son iguales si sus componentes reales e


imaginarias son iguales entre sí: z = w → {ba == cd
2.2 Forma Polar o Trigonométrica
Observando la Fig. 1 es sencillo convertir de coordenadas cartesianas a polares y viceversa:
ρ = √ a2+b 2 ; θ = arctan(b/ a) → x = ρ cos θ ; y = ρ sen θ

MATEMATICA APLICADA - Números Complejos - Funciones Elementales – Cursada 2020/2021. 2


donde ρ se denomina módulo de z y representa la longitud del vector; θ se denomina argumento y
es el ángulo que forma con el semieje positivo de abscisas. Por convención, a los vectores que se
ubican en el primer y segundo cuadrantes, les corresponden argumentos positivos: 0 ≤ θ ≤ π;
mientras que a aquellos ubicados en el tercer y cuarto cuadrantes, les corresponden argumento
negativos: -π < θ < 0. Reemplazando en la forma cartesiana a y b por sus coordenadas polares,
tenemos:

z = ρ (cosθ +i sen θ ) FORMA POLAR O TRIGONOMETRICA

Esta forma Polar o Trigonométrica, se puede abreviar: z = ρ cis(θ ) cis → cosθ + i senθ

Además de estas dos formas que acabamos de ver, existen otras para representar un C, que
veremos más adelante.

Dados dos números Reales a y b, siempre es posible establecer con certeza cuál de ellos es mayor
ó menor; esto se denomina Relación de Orden y permite además representarlos gráficamente en
la recta numérica, donde siempre el número que está a la izquierda, es menor que el de la
derecha. Esta Relación de Orden NO se verifica en el conjunto C, puesto que dados dos números
complejos w y z, no es posible establecer cuál es mayor o menor; solamente es posible comparar
sus módulos (longitud del vector que lo representa) porque son números reales. Puede ayudar a
comprender esta situación, el hecho que así como representamos el conjunto R sobre una recta, el
conjunto C se representa en el plano cartesiano, donde cada número complejo es un punto
perteneciente al plano y su módulo es la distancia al origen.

Conjugado de un número complejo

Sea el número complejo z = a+bi se define su conjugado z̄


como otro número complejo con igual componente real y componente
imaginaria de signo contrario: z̄ = a−bi .

En forma Polar: z = ρ cis(θ )

z̄ = ρ cis(−θ ) = ρ (cos (−θ )+isen (−θ )) = ρ (cos θ −isenθ )

Sea un número complejo y su conjugado: z = x + iy ; z̄ = x−iy

Se plantean la suma, la resta y el producto entre ambos:

MATEMATICA APLICADA - Números Complejos - Funciones Elementales – Cursada 2020/2021. 3


z + z̄ = 2 x ; z− z̄ = 2iy ; z . z̄ = x 2+ y 2

El producto de dos números complejos conjugados, adquiere la forma de una diferencia de


cuadrados, dando por resultado un número real positivo, equivalente al cuadrado del módulo.

3. OPERACIONES BASICAS CON NUMEROS COMPLEJOS


3.1 Sumas, Restas y Productos
Para sumar/restar números complejos, se suman/restan entre sí las componentes real e
imaginaria. Sean: z = a+bi w = c+di → z+w = (a+c )+ i(b+ d) w−z = (c−a )+i(d −b)

Para multiplicar números complejos, se aplica la propiedad distributiva, teniendo especial cuidado
al multiplicar las unidades imaginarias entre sí.
z. w = ac + adi + bci + dbi 2 = (ac−db)+ i(ad+bc)

3.2 Cociente de números complejos en forma cartesiana


Para dividir dos números complejos en forma cartesiana, se multiplica y divide la expresión (para
que no se modifique), por el conjugado del denominador, para conseguir allí un número real:
z a+bi c −di ac+ bci−adi+ bd ac+bd bc−ad
= . = = 2 2 + i 2 2
w c +di c −di 2
c +d 2
c +d c +d

Los productos o cocientes que involucran varios números complejos, se vuelven


tediosos/complicados para resolverlos en forma cartesiana, debido a la aplicación reiterada de la
propiedad distributiva y de las operaciones con los conjugados. Para estos casos es más
conveniente escribir los números en forma polar o trigonométrica.

3.3 Producto/Cociente de números complejos en forma polar


Sean dos números complejos en forma polar: z = ρ cis θ w = σ cis α
z. w = ρ σ (cos θ +isen θ )(cos α +isen α ) = ρ σ (cos θ cos α +i cos θ senα +isen θ cos α −sen θ sen α ) =
ρ σ [(cos θ cos α −sen θ sen α )+i(cos θ sen α +sen θ cos α )] = ρ σ [cos (θ + α ) + isen(θ + α )]
Entonces, para multiplicar dos números complejos, se multiplican sus módulos y se suman sus
argumentos. Esta regla es válida, cualquiera sea la cantidad de factores, es decir cualquiera sea la
cantidad de números que se multipliquen.

MATEMATICA APLICADA - Números Complejos - Funciones Elementales – Cursada 2020/2021. 4


z ρ (cos θ +isen θ ) cos α −isen α
Para el caso del cociente: = . =
w σ (cos α +isen α ) cos α −isen α
ρ cos θ cos α − icosθ sen α + isenθ cos α + sen θ sen α
σ . =
cos2 α +sen 2 α
ρ ρ
σ .[(cos θ cos α +sen θ sen α )+i(sen θ cos α −cos θ sen α )] = σ .[cos (θ −α )+isen(θ −α )]
Para dividir dos números complejos, se dividen sus módulos y se restan sus argumentos. Esta regla
es válida cualquiera sea la cantidad de números complejos involucrados en la operación.

ρ cis θ ρ
ρ cis θ .σ cis α = ρ σ cis (θ +α ) = σ . cis(θ −α )
σ cis α

3.4 Potencias de números complejos


Sea z ϵ C dado en forma polar: z = ρ cis(θ ) para obtener la potencia enésima: w = zn se

aplica la regla del producto que se acaba de demostrar; siendo w el producto de z por sí mismo, n

veces, resulta: w = zn = ( ρ cis θ )n = ρ n cis nθ = ρ n cis (n θ ) para n ϵ R

Para resolver una potencia n-sima, se eleva el módulo al exponente n y se multiplica el argumento
por dicho exponente.

4. IDENTIDAD DE EULER
Utilizando Series de Taylor vamos a demostrar que la expresión cisθ es equivalente a una
exponencial de base e. Recordamos las Series de Taylor de tres funciones:
x2 x3 x4 x5 x6
ex = 1 + x + + + + + + ...........................
2! 3! 4! 5! 6!
x3 x5 x7 x9
senx = x − + − + − .....................
3! 5! 7! 9!

x2 x4 x6 x8
cos = 1 − + − + − .................... .
2! 4! 6! 8!
Reemplazando el exponente x por ix, la Serie de la función Exponencial resulta:
(ix)2 (ix)3 (ix )4 (ix)5 (ix )6 (ix )7
e ix = 1 + ix + + + + + + + ...................... =
2! 3! 4! 5! 6! 7!
x2 ix 3 x4 ix 5 x6 ix 7 x8 ix 9
1 + ix − − + + − − + + −...................... =
2! 3! 4! 5! 6! 7! 8! 9!
x2 x4 x6 x8 x3 x5 x7 x9
(1 − + − + −.......) + i( x − + − + − .....) = cosx +i sen x
2! 4! 6! 8! 3! 5! 7! 9!

MATEMATICA APLICADA - Números Complejos - Funciones Elementales – Cursada 2020/2021. 5


e ix = cos x + i sen x IDENTIDAD DE EULER

Esta maravillosa expresión fue descubierta por el matemático Leonhard Euler en el siglo XVIII y
muestra que cis θ = e iθ , de manera que la forma polar se puede expresar como:
θ θ
z = ρ cis(θ ) = ρ e i → z = ρ ei FORMA EXPONENCIAL

5. FUNCIONES ELEMENTALES
En Cálculo 1 trabajamos con Funciones Reales (de una variable) y definimos como función a una
relación biunívoca entre dos variables, donde a cada valor de la variable independiente le
corresponde un único valor de la variable dependiente; dicho de otra manera: a cada elemento del
dominio le corresponde un sólo elemento de la imagen. Este tipo de funciones se denomina
UNIVALUADAS. Ahora llegó el momento de ampliar nuestro panorama para incorporar las
FUNCIONES MULTIVALUADAS, que son aquellas donde a un valor del dominio le corresponden dos
ó más imágenes diferentes y son frecuentes en el Análisis Complejo. En particular vamos a estudiar
dos funciones con estas características: la raíz enésima y el logaritmo natural, de números
complejos.

5.1 Raíz Enésima Compleja


n
Sea z un número complejo cualquiera, la función raíz n-sima se define como: w = f (z) = √z
donde z es la variable independiente y w la dependiente; n se denomina índice de la raíz y es un
número natural. En el campo de los Reales, si el índice de la raíz es par, el argumento debe ser ≥ 0
y existen dos soluciones; si el índice es impar, siempre existe una única solución. En el campo de
los Complejos, cualquiera sea el valor de n, existen n soluciones diferentes.
Si el argumento de la raíz es: z = ρ cis θ w ϵ C y podemos expresarlo como: w = σ cis α

w = √n z = z1/n = ( ρ cis θ )1/ n = ρ 1/ n (cis θ )1/ n = ρ 1/ n cis( θn ) = σ cis α

Para que se verifique la igualdad ρ 1/ n cis( θ ) = σ cis α es necesario que los módulos y los
n
argumentos de ambos números complejos sean iguales. Al ángulo θ se lo denomina Argumento
Principal y al sumarle un múltiplo entero de 2π, se obtiene el mismo ángulo: θ ≈ θ + 2kπ, donde k ϵ
N0. Así deben verificarse las ecuaciones:

MATEMATICA APLICADA - Números Complejos - Funciones Elementales – Cursada 2020/2021. 6


θ +2 k π n θ +2 k π
ρ 1/ n = σ
n
= α y resulta: √ z = ρ 1/ n cis ( n ) EXPRESION RAIZ ENESIMA

Al asignar a k diferentes valores, se obtienen las diversas soluciones de la raíz n-sima, es decir las
diferentes imágenes.
θ +2 π
k = 0 → Valor Principal: w 0 = ρ 1/ n cis( θ ) k = 1 → w 1 = ρ 1/n cis ( )
n n
θ +4 π θ +6 π
k = 2 → w 2 = ρ 1/n cis ( ) k = 3 → w 3 = ρ 1/n cis ( )
n n
θ +2 π (n−1)
k = n-1 → w n−1 = ρ 1/ n cis ( )
n
θ +2 π n
k = n → w n = ρ 1/ n cis ( )= ρ 1/ n cis( θ + 2 π )
n n

La expresión de wn es idéntica a la de w 0 (igual módulo e igual argumento), es decir que w n repite


la solución ya encontrada con k = 0. Así para encontrar soluciones diferentes, se debe asignar a k
valores desde 0 hasta (n-1); por este motivo esta función es MULTIVALUADA y tiene un número
finito de imágenes distintas para cada elemento del dominio. Al observar los módulos y los
argumentos de cada una de las soluciones, se destacan dos cuestiones:
• Todas las soluciones son números complejos con el mismo módulo; entonces los extremos
de los vectores que las representan se ubican sobre una circunferencia con centro en el
origen y radio ρ1/n.
• Los argumentos de cada solución son diferentes, pero la diferencia entre los argumentos de


w0 y w1 es y es la misma diferencia que entre los argumentos de w 1 y w2; y se repite
n
este mismo valor para la diferencia entre argumentos de todas las soluciones consecutivas;
es decir que los vectores que representan las n soluciones están equiangularmente
espaciados alrededor de la circunferencia y al unir sus extremos se obtienen polígonos
regulares de n lados (n ≥ 3).

5
Ejemplo 1: Obtener analítica y gráficamente las soluciones de: √−1−i
El número complejo z = −1−i es el argumento de esta raíz y en forma polar es:

−3 π −3 π
z = √2 cis ( ) → ρ =√ 2 ; θ = y la forma general de las soluciones será:
4 4

MATEMATICA APLICADA - Números Complejos - Funciones Elementales – Cursada 2020/2021. 7


5 1/5 (−3 π / 4)+2 k π
√−1−i = (√ 2) cis ( 5
) k = 0, 1, 2, 3, 4.

w0 = √ 2 cis ( −3 π )
10
w1 = √ 2 cis ( π )
10
w2 = √ 2 cis ( 13 π )
10
20 4 20

w3 = √ 2 cis ( 2120π )
10
w4 = √ 2 cis ( 2920π )
10

Gráficamente los extremos de los 5 vectores se


ubican sobre una circunferencia de radio (2) 1/10
y entre cada vector y los adyacentes, existe un
ángulo de 720.
Para ubicar los vectores solución, basta con
colocar w0 (Valor Principal) y luego ir
agregando 720 a cada argumento. Al unir los
extremos en este caso, se obtiene un
pentágono regular. En este caso, al ser el
radicando un número complejo con parte
imaginaria no nula, todas las soluciones
también verifican esta condición.

5.2 Logaritmo Natural Complejo


En Análisis de Funciones Reales se ha visto que el dominio de la función y = lnx son los Reales
positivos: (0, ∞); por lo tanto para que exista el logaritmo (cualquiera sea la base, no solamente el
natural) el argumento debe ser x > 0. Sin embrago, como veremos a continuación, la Función

Compleja: w = ln( z ) tiene como dominio todos los Complejos, excepto el 0, es decir: C – {0};
la imagen de esta función es todo el plano C. Expresando el argumento z de la función en forma

exponencial, se tiene: ln(z ) = ln( ρ ei θ ) = ln ρ + ln ( ei θ ) = ln ρ + i θ

ln (z ) = ln ρ +i(θ +2 k π ) k ϵ Enteros EXPRESION GENERAL LOGARITMO NATURAL

Según muestra la expresión anterior, la función Logaritmo Natural de un número complejo es


MULTIVALUADA con Infinitas soluciones, dado que k puede asumir cualquier valor entero (0, ±1,
±2, ±3, ……..) y para cada uno obtendremos un argumento diferente y por ende, una solución

MATEMATICA APLICADA - Números Complejos - Funciones Elementales – Cursada 2020/2021. 8


diferente, de maneras que existen infinitas soluciones, todas con la misma componente Real pero
distintas componentes imaginarias: Re(lnz) = lnρ ; Img(lnz) = θ+2kπ.
w 0= ln ρ +i θ ; w 1= ln ρ +i (θ +2 π ) ; w2= ln ρ +i(θ + 4 π ) ; .........; wn= ln ρ +i(θ +2 n π )

El valor w0 (k = 0) se denomina Valor Principal. Como todas las


soluciones tienen la misma componente real, los extremos de los
infinitos vectores solución, se ubican sobre una recta vertical
cuya ecuación es: x = lnρ. Al asignarle a k diferentes valores
enteros, se obtienen las componentes imaginarias de las
soluciones w0, w1, w2, w3,... etc.

• Si el argumento del logaritmo es un número real positivo , θ = 0, así w0 = lnρ es la única


solución real, las demás (infinitas) soluciones, tienen componente imaginaria y por ende,
son números complejos. Esa única solución real es la que obtenemos cuando analizamos la
función en el campo de los Reales, ya que es la única visible desde “ese punto de vista”.
Pero ahora descubrimos que existen otras infinitas soluciones del logaritmo de un número
real positivo y al pertenecer al conjunto C, no podíamos visualizarlas hasta ahora.
• Si el argumento del logaritmo es un número real negativo, θ = π, las infinitas soluciones
tendrán componente imaginaria no nula, es decir, no hay ni una solución real, motivo por el
cual en el campo de los reales no está definido el logaritmo de un número negativo.
• Si el argumento del logaritmo es un número complejo donde Img(z) ≠ 0, las infinitas
soluciones tienen componente imaginaria no nula y tampoco existen en este caso,
soluciones reales.

Ejemplo 2: a) ln(4) b) ln(-4) c) ln(3+4i)

a) z = 4 → ρ = 4; θ = 0 → ln ( 4) = ln 4+i(2 k π )
k = 0 → w0 = ln4 k = 1 → w1 = ln4 + 2π i k = -1 → w2 = ln4 - 2π i

b) z = -4 → ρ = 4; θ = π → ln(−4) = ln 4+i( π +2 k π )

MATEMATICA APLICADA - Números Complejos - Funciones Elementales – Cursada 2020/2021. 9


w0 = ln4 + π i w1 = ln4 + 3π i w2 = ln4 - π i w3 = ln4 - 3π i

c) z = 3 + 4i → ρ = 5; θ = 0,9273 → ln (3+ 4 i) = ln 5+i(0,9273+2 k π )


w0 = ln5 + i 0,9273 w1= ln5 + i(0,9273+2π) w2 = ln5 + i(0,9273-2π)

Entonces, en el campo de los números complejos, cualquier número distinto de 0, posee infinitos
resultados para el logaritmo y en caso que el número fuese un real positivo, el VP es real. Si el
número es real negativo o complejo, las infinitas soluciones son números complejos con parte
imaginaria no nula.
El estudio de la función f(z) = lnz, también es obra del matemático Leonhard Euler.

5.3 Exponencial Natural Compleja


La función f (z) = e z tiene como dominio todo el plano C y su imagen es: C – {0}. Para
desarrollar esta expresión, es necesario aplicar la identidad de Euler:
f (z) = e z = e x+iy = e x e iy = e x(cosy +i seny ) = e x cosy + i e x seny

f (z) = e z = e x cosy + i e x seny Esta función es Univaluada.

Veamos porqué el 0 no pertenece a la imagen de esta función. Suponiendo que perteneciera,


debería existir un número z que verifique:

MATEMATICA APLICADA - Números Complejos - Funciones Elementales – Cursada 2020/2021. 10


e z = e x cosy + i e x seny = 0+i 0 →
{ e x cosy = 0
e x seny = 0
(1)

En ambas ecuaciones del sistema anterior, aparece la función real y = e x que según
estudiamos en Cálculo 1 tiene como imagen el intervalo (0, ∞), es decir su imagen nunca es cero;
de manera que la única posibilidad que ambas ecuaciones sean nulas es que se cumpla:

{cosy = 0
seny = 0
lo cual no es posible, porque no existe ángulo alguno, cuyo seno y coseno sean

nulos . De esta manera queda demostrado que el sistema de ecuaciones (1) no tiene solución y por

lo tanto, 0 no pertenece al dominio de f (z) = e z .

Ejemplo 3: Resolver w = e −i

z = - i → x = 0; y = -1 → e−i = cos (−1) + isen(−1)=cos 1−isen1 = 0,5403−i 0,84147


Al realizar los cálculos, tener la precaución de trabajar en radianes.

5.4 Seno y Coseno de Números Complejos


Dadas las siguientes expresiones de la identidad de Euler:
e i z = cos z+i senz e−i z = cos z−i senz

Al sumar y restar ambas, se obtienen las expresiones de las funciones Seno y Coseno en variable
compleja:
ei z +e−i z e i z −e−i z
e i z +e −i z= 2 cos z → cos z = ei z−e −iz = 2i sen z → sen z =
2 2i

El dominio de ambas funciones es todo el plano C.


A partir de estas dos funciones, se definen las otras 4 funciones trigonométricas complejas; de la
misma manera que en Variable Real.

Ejemplo 4: Expresar en forma cartesiana: a) cos(2i) b) sen(-2i+6)

i(2i ) −i(2i )
e +e e−2 +e 2
cos (2 i) = = = 3,762 → cos (2 i) = 3,762
2 2

i (6−2i) −i(6−2i)
e −e e 2+6i−e−2−6i e 2 (cos 6 +isen6)−e −2 (cos 6−isen6)
sen (6−2 i) = = = =
2i 2i 2i

MATEMATICA APLICADA - Números Complejos - Funciones Elementales – Cursada 2020/2021. 11


cos 6(e 2−e −2 ) + isen 6(e2 +e−2 ) cos 6 (e 2−e−2) i sen 6(e 2 +e −2 )
= . + =
2i 2i i 2
i (cos 6 (e 2−e −2 )) sen 6 (e 2 +e−2 )
+ = −1,0512 −i 3,4824 → sen (6−2 i) = −1,0512−i 3,4824
−2 2
En el segundo renglón de este último desarrollo, se multiplica y divide el primer término por la unidad
imaginaria, para conseguir un número real en el denominador.

A partir de los resultados, se observa que el coseno de un número imaginario puro es un número
real y que los módulos en ambos ejemplos, resultan superior a uno; a diferencia de lo que sucede
en variable real, donde el módulo del seno y coseno es ≤ 1.

5.5 Exponencial Compleja

Una expresión de la forma: u = z w donde z y w ϵ C, se denomina Exponencial Compleja y para


resolver vamos a aplicar logaritmo natural:
u = z w → ln (u) = ln (z w ) = w ln ( z) = w[ ln ρ + i(θ + 2 k π )] siendo z = ρ cisθ

u = e ln (u) = e w [ln ρ +i(θ +2 k π )] → z w = e w [ln ρ +i(θ + 2k π )]


EXPRESION GENERAL EXPONENCIAL COMPLEJA

Esta función es Multivaluada, con infinitas soluciones, dado que involucra el logaritmo. Al asignar a
la constante k valores enteros, se obtienen las distintas soluciones.

Ejemplo 5: Expresar en forma cartesiana: w = (i)i

w = (i)i → ln (w) = i ln (i)=i [ln 1+i( π + 2 k π )] = i[i ( π +2 k π )] = −( π +2 k π )


2 2 2
−(
π +2k π )
i ln (w) 2
w = (i) = e = e Esta expresión muestra que las infinitas soluciones serán todas
Reales; para k = 0 obtenemos el VP: w0 = 0,207879.

6. REGIONES EN EL PLANO COMPLEJO


Las regiones circulares y anulares en el plano C son de especial interés, debido a sus aplicaciones
en posteriores temas de la Asignatura.
Una circunferencia con centro en el origen y radio R, tiene como expresión: x 2+ y 2=R2
Siendo: z = x+iy su módulo es: ρ =|z| = √ x2 + y 2 = √ R2 = R → |z|= R

MATEMATICA APLICADA - Números Complejos - Funciones Elementales – Cursada 2020/2021. 12


Esta última expresión corresponde a una circunferencia con centro en el origen y radio R.
La expresión: |z|< R corresponde a la región interior a la circunferencia, sin incluir la frontera.

La expresión: |z|> R corresponde a la región exterior a la circunferencia, sin incluir la frontera.

Mientras que: |z|≤ R y |z|≥ R representan la región interior y exterior, incluida la frontera ,

respectivamente.

Si la circunferencia tiene centro en el punto (a, b) y radio R: (x −a)2 +( y−b)2=R2


z−(a +bi) = ( x−a)+i( y−b ) → |z−(a+ib )|=√( x−a )2 +( y −b)2 = √ R 2= R → |z−(a+ib)| = R
Esta última expresión corresponde a una circunferencia con centro desplazado.

Para representar una región anular, con centro en el punto (a, b), con radio interior R 1 y radio

exterior R2, corresponde la expresión: R1 ≤|z−(a+ib)|≤ R 2 , incluyendo ambos bordes.

Mientras que: R1 <|z−(a +i b)|< R 2 representa el interior del anillo solamente.

MATEMATICA APLICADA - Números Complejos - Funciones Elementales – Cursada 2020/2021. 13


FUNCIONES DE VARIABLE COMPLEJA. DERIVADA.

1. FUNCIONES COMPLEJAS
En el estudio de Funciones de Variable Real (FVR) en R 2 y R3, la representación/interpretación
gráfica tiene un rol muy importante, por ejemplo:
• En aquellos valores excluidos del dominio, la función no es continua y su gráfica presenta una AV,
un Salto ó un Hueco.
• La derivada primera, evaluada en un punto, representa la pendiente de la RT a la curva en dicho
punto
• Mediante el signo de la primera y segunda derivada, se puede analizar el crecimiento y la
concavidad, respectivamente.
• En aquellos puntos que presentan RTH la función tiene extremos relativos
• Si para cierto valor de la variable independiente, se presenta un limite indeterminado y una vez
resuelto, arroja como resultado un número real, allí la gráfica tiene un Hueco.
• Mediante una integral definida se puede encontrar el área encerrada por la función y los ejes
coordenados ó entre dos ó más funciones.

Cuestiones como estas, donde la resolución analítica tiene una relación directa con las
interpretaciones gráficas NO ocurren en Funciones de Variable Compleja (FVC), ya que en este caso
NO es posible representar una función de la manera que lo hacemos en R 2 y R3. En R2 tenemos:
y = f (x) donde las variables x e y son unidimensionales, de manera que la función se puede

graficar en dos dimensiones; en R3 tenemos: z = f (x , y ) donde ambas variables independientes

y la dependiente z, son unidimensionales, entonces la gráfica se realiza en tres dimensiones. Pero


en el caso de Funciones de Variable Compleja: w = f ( z) tanto la variable independiente z,

como la dependiente w, son bidimensionales y para representar la función necesitaríamos cuatro


dimensiones; de manera que la representación gráfica como herramienta para analizar funciones
NO está disponible en FVC y cuestiones como la derivada o la integral, tampoco tienen una
interpretación directa o inmediata como en FVR. Si bien existe una manera de representar FVC –
Mapeo o Transformación – es sustancialmente diferente a las gráficas para FVR; este tema lo
veremos posteriormente.
Partiendo del hecho de no contar con una representación gráfica de FVC, los análisis de límites,
continuidad y derivadas, igualmente se realizan de forma analítica – en muchos casos con procesos

MATEMATICA APLICADA - MATEMATICA III – Cursada 2020/2021. María C. Ibarra - 1


muy similares al Análisis Real – pero carecen de una interpretación gráfica inmediata y sencilla. Las
interpretaciones y aplicaciones de FVC se realizan en un contexto físico, cuando la función
representa un fenómeno ó proceso (por ejemplo flujo de fluidos o distribución de temperaturas
bidimensionales; lugar geométrico de impedancias y admitancias en circuitos eléctricos, entre
otras). Para alcanzar este tipo de interpretaciones y apreciar las utilidades de FVC como modelos
matemáticos de diversos fenómenos físicos, en primer lugar es necesario conocer los procesos
analíticos básicos, como análisis de Límites, Continuidad y Derivadas en el Plano Complejo.
Veremos que existen muchas similitudes - al menos desde el punto de vista conceptual – con el
Análisis Real, pero también muchas diferencias, tanto conceptuales como procedimentales. Por
ejemplo, en cuanto a la Derivada, en FVR nos ocupamos de conocer y manejar las técnicas
aplicables en cada caso para encontrar la función derivada y evaluarla en determinados puntos,
para hallar la pendiente del la recta Tangente o para determinar si presenta Puntos Críticos o
Extremos Relativos ó para resolver un problema de Optimización; es decir en FVR se busca la
expresión de la derivada y en las aplicaciones, es fundamental evaluarla en determinados puntos.
En cambio en FVC la mayoría de las veces no estaremos interesados ni siquiera en encontrar la
expresión de la función derivada, menos aún en evaluarla numéricamente, sino que lo más
importante será determinar si existe derivada y en que región del plano es diferenciable la función;
es decir será un análisis más bien conceptual.

2. LIMITES Y CONTINUIDAD EN FVC


En Análisis Complejo, los conceptos de Límite, Continuidad y Derivada, en cuanto a sus
definiciones formales, son muy similares al Análisis Real; pero en lo que respecta a sus
procedimientos analíticos, existen importantes diferencias.
En Funciones de una Variable Real: y = f (x ) para que exista lim f ( x) siendo c ϵ R, es
x→c

necesario que ambos Límites Laterales existan y además sean iguales; porque en este caso
solamente existen dos direcciones a través de las cuales acercarse a c, por izquierda y derecha, en
la recta Real.
En FVC – similarmente a los que sucede con Funciones Reales de dos variables independientes -
para que exista límite en un punto (x, y) es necesario que dicho límite sea el mismo en todas las
direcciones posibles, que en este caso son infinitas.

MATEMATICA APLICADA - MATEMATICA III – Cursada 2020/2021. María C. Ibarra - 2


Sea la FVC: w = f ( z) donde z y w son las

variables independiente y dependiente,


respectivamente. Sea z0 un punto del plano

complejo, para que exista lim f (z) es necesario


z → z0

que dicho límite exista y asuma el mismo valor a


través de los infinitos caminos posibles, que en este
caso, son las infinitas funciones/curvas del plano
que pasan por z0.
Las condiciones de Continuidad en FVC son similares a FVR. Así, siendo w = f(z) y z 0(x0, y0) un punto
del plano complejo, la continuidad de f(z) en el punto, exige tres condiciones:
1) Existencia f(z0) 2) Existencia lim f (z) 3) f (z 0) = lim f ( z)
z→ z0 z→z 0

Pero a diferencia de los que sucede en FVR, donde la falta de cumplimiento de alguna de estas
condiciones genera alguno de los tres tipos de discontinuidades, con su inminente repercusión
gráfica, en FVC la discontinuidad en un punto es meramente conceptual.

3. DERIVADAS EN FVC
f ( z+Δ z)−f (z)
Sea w = f(z); la definición de derivada en el plano complejo es: f I (z) = lim (1)
Δ z→0 Δz

Siendo: Δ z = Δ x +i Δ y el incremento de la variable independiente.

Si la función está dada en términos de z, la derivada se realiza siguiendo las mismas reglas que en
FVR, por ejemplo:
−senz
g (z) = 3 z4 −12 z2− 9 → g I (z) = 12 z3−24 z ; h( z)=ln(cosz ) → h I ( z) =
cosz
También es válida en estos casos la Regla de L’Hopital:
1−cosz senz cosz 1
lim 2
= lim = lim =
z→0 z z → 0 2z z→0 2 2

Pero si f(z) está dada en términos de x e y, habrá que encontrar otra manera de derivar; por
ejemplo: f (z) = z̄ = x−iy ; h(z )= 2 xy 2 − i (x 2 +6 y ) son FVC muy simples, pero no es posible
derivarlas en términos de z. Para estos casos, se utiliza un sistema de Ecuaciones en Derivadas

MATEMATICA APLICADA - MATEMATICA III – Cursada 2020/2021. María C. Ibarra - 3


Parciales, denominado Ecuaciones o Condiciones de Cauchy – Riemann. Siendo la variable
independiente z = x + i y , la función puede expresarse como:
w = f ( z) = f ( x , y ) = u(x , y )+i v (x , y ) (2)

Decimos entonces que x e y son las coordenadas del Plano z que representa el dominio o variable
independiente de la función; mientras que u y v son las coordenadas del Plano w que representa
la imagen o variable dependiente de la función. En la expresión (2), u(x, y) es la componente real
de w mientras que v(x, y) es su componente imaginaria. En las aplicaciones físicas, los Modelos
Matemáticos vienen dados en función de las variables cartesianas x e y, dado que representan
fenómenos bidimensionales; entonces para abordarlos es necesario encontrar la manera de
efectuar la diferenciación en términos de estas variables.

3.1 ECUACIONES DE CAUCHY – RIEMANN


Para que la función f(z) sea derivable, el límite de la expresión (1) debe existir y sus resultados, a
través de cualquiera de los infinitos caminos posibles, deben ser idénticos. Elegimos los dos
caminos más sencillos, que son las rectas paralelas a los ejes coordenados.
f (z) = u (x , y ) + i v (x , y ) f (z+Δ z) = u( x +Δ x , y +Δ y ) + i v (x + Δ x, y + Δ y )

Camino 1: Recta Horizontal → y = constante → Δy = 0 → Δz = Δx


f (x +Δ x , y )−f (x , y ) u( x +Δ x , y ) + i v (x+ Δ x , y)−u (x , y )−iv (x , y)
f I (x , y ) = lim = lim →
Δ x→0 Δx Δ x →0 Δx

u (x +Δ x , y ) −u( x , y ) v (x +Δ x , y ) −v ( x , y) ∂u ∂v
lim + i lim = +i
Δ x →0 Δx Δ x →0 Δx ∂x ∂x

∂u ∂v
f I (x , y ) = +i (3)
∂x ∂x

Camino 2: Recta Vertical → x = constante → Δx= 0 → Δz = i Δy


f ( x , y +Δ y )−f (x , y ) u( x , y + Δ y ) + i v ( x , y +Δ y )−u(x , y )−iv (x , y )
f I (x , y ) = lim = lim →
Δ y →0 iΔ y Δ y→0 iΔ y

u (x , y + Δ y ) −u( x , y) i v (x , y + Δ y) −v (x , y ) ∂u ∂ v
lim .( ) + i lim = −i +
Δ y →0 i Δ y i Δ y →0 i Δ y ∂ y ∂y

∂v ∂u
f I (x , y ) = −i (4)
∂y ∂y

MATEMATICA APLICADA - MATEMATICA III – Cursada 2020/2021. María C. Ibarra - 4


Para que exista la derivada es necesario que las expresiones (3) y (4) sean iguales, entonces por
condición de igualdad de números complejos, resulta:
∂u ∂v ∂v ∂u
= ; = − ECUACIONES DE CAUCHY-RIEMANN
∂x ∂y ∂x ∂y

Si las funciones u(x,y), v(x,y) y sus cuatro derivadas parciales de primer orden son continuas en
determinada región o dominio del Plano Complejo y satisfacen las Condiciones de Cauchy –
Riemann, estas condiciones son necesarias y suficientes para asegurar la derivabilidad de la
función: f(x,y) = u(x,y) + i v(x,y) , en dicha región del Plano Complejo.

De esta manera las Ecuaciones de Cauchy Riemann (ECR) no solamente determinan si la función es
derivable sino también indican en que región del plano existe la derivada. Cuando hablamos de
región/dominio, puede ser todo el plano ó solamente parte de él (por ejemplo un cuadrante, el
interior de una circunferencia, una curva, etc.). Las ECR conforman un sistema, es decir deben
cumplirse ambas simultáneamente.
En caso que estemos interesados en encontrar la derivada, podemos recurrir a las expresiones (3)
ó (4) para hallar la expresión analítica de dicha función.

Ejemplo 1: Determinar en qué región del Plano Complejo son derivables las siguientes funciones:

a . f ( x , y ) = z̄ b .w =|z| 2 c .h( x , y ) = x 2− y 2 − y+i(2 x y + x ) d . f ( x , y ) = x2−x + y + i( y 2−5 y−x )

a) f(x,y) = x – i y → u(x,y) = x; v(x,y) = -y


∂u ∂u ∂v ∂v
=1 ; =0 ; =0 ; = −1 → Aplicando ECR: 1 = −1 ; 0 = 0
∂x ∂y ∂x ∂y

La primera de estas igualdades es un ABSURDO, que no se cumple en ningún punto del plano y la
segunda es una IDENTIDAD, que se cumple en todo el plano. Dado que ECR es un sistema de
ecuaciones, para hallar la solución es necesario hacer la intersección entre ambas, que en este
caso resulta un conjunto vacío, equivale a decir que no existe región del plano donde se verifiquen
ambas simultáneamente, de manera que la función no es derivable en ningún punto del plano .
Siempre que una de las ECR sea un Absurdo, el resultado será que la función no es derivable.
Conclusión: la función es continua en todo el plano pero no es derivable en ningún punto.

MATEMATICA APLICADA - MATEMATICA III – Cursada 2020/2021. María C. Ibarra - 5


b) w = x2 + y2 → u(x,y) = x2 + y2 ; v(x,y) = 0
∂u ∂u ∂v ∂v
=2x ; =2y ; =0 ; = 0 → ECR: 2x = 0 → x = 0 ; 2 y = 0 → y = 0
∂x ∂y ∂x ∂y
La primera de las ECR se cumple para todos los puntos ubicados sobre el eje de ordenadas y la
segunda, se cumple para todos los puntos ubicados sobre el eje de abscisas; la intersección entre
ambos es el origen (0,0), entonces la función es derivable solamente en dicho punto.
Conclusión: la función es continua en todo el plano y derivable solamente en el origen.

c) h(x,y) = x2 - y2- y + i(2xy + x) → u(x,y) = x2 - y2- y ; v(x,y) = 2xy + x


∂u ∂u ∂v ∂v
=2x ; =−2 y −1 ; = 2 y +1 ; = 2 x → ECR: 2 x = 2 x ; 2 y +1 = −(−2 y −1)
∂x ∂y ∂x ∂y
En este caso ambas ECR son identidades, se cumplen en todo el plano complejo, entonces, la
función es derivable en todo el plano.
Para encontrar la expresión de la derivada, elegimos la expresión (3): h I ( x , y) = 2 x+ i(2 y +1)
Si elegimos la expresión (4): h I ( x , y ) = 2 x −i(−2 y −1) = 2 x +i(2 y +1)
Conclusión: la función es continua y derivable en todo el plano.

d) f(x,y) = x2 - x + y + i(y2 – 5y – x) → u(x,y) = x2 - x + y ; v(x,y) = y2 – 5y – x


∂u ∂u ∂v ∂v
= 2 x−1 ; =1 ; = −1 ; = 2 y −5 → ECR: 2 x −1 = 2 y−5 ; 1 = −(−1)
∂x ∂y ∂x ∂y
La primera de estas ecuaciones resulta la recta: y = 2 + x, mientras que la segunda es una identidad
que se verifica en todo el plano, de manera que la intersección es la recta.
Conclusión: la función es continua en todo el plano y derivable solamente en los puntos ubicados
sobre la recta y = x + 2.

Las cuatro funciones analizadas en este ejemplo son sencillas y tienen forma de polinomios en las
variables x e y; resultan continuas en todo el plano. Sin embargo, sólo una de ellas es derivable en
todo el plano, otra solamente sobre una recta, otra únicamente en un punto y otra en ningún
punto. La apariencia algebraica de estas funciones son bastante similares, sin embargo el análisis
de derivabilidad en cada caso arroja resultados sustancialmente diferentes, mostrando que para
FVC hay que analizar cada caso en particular, mediante las ECR.

MATEMATICA APLICADA - MATEMATICA III – Cursada 2020/2021. María C. Ibarra - 6


4. FUNCIONES ANALITICAS

Sea z0 un punto del plano complejo, se define


como VECINDAD de z0, la región del plano
determinada por: |z−z0 | < R siendo R > 0.

Sea la función: f (z) = u (x , y ) + i v (x , y ) y sea z0


un punto del plano complejo en el cual es
derivable, es decir existe f I (z 0) . La función es

ANALITICA en z0 si es derivable en el punto y en


todos los puntos de una vecindad (con radio arbitrario). Así, la condición de Analiticidad es más
exigente que la de Diferenciabilidad, porque no solamente debe ser diferenciable en el punto en
cuestión, sino también en los infinitos puntos interiores a la vecindad. Las funciones analíticas son
muy importante en las aplicaciones físicas, porque cuando una FVC es analítica, sus componentes
u y v satisfacen ciertas condiciones especiales, permitiendo así representar en Variable Compleja,
Campos Vectoriales Bidimensionales, como Campos Eléctricos , Campos de Velocidades, Campos
de Distribución de Temperaturas, entre otros.

En el Ejemplo 1, hemos visto que una FVC puede ser derivable en todo el plano, sobre una
curva/función o solamente en un punto del plano complejo. Analizaremos ahora que sucede con la
Analiticidad en tres de estos casos.

1. Si la función es derivable solamente en un punto del plano, NO será analítica en ningún punto
del plano; ya que por más reducido que sea el radio de la vecindad, indefectiblemente contendrá
puntos del plano donde la función no es diferenciable.
Así la función: w =|z| 2 es continua en todo el plano, derivable solamente en el origen y no es

analítica en ningún punto.

MATEMATICA APLICADA - MATEMATICA III – Cursada 2020/2021. María C. Ibarra - 7


2. Si la función es derivable solamente sobre una
curva/función, no será analítica en ningún punto
del plano, dado que cualquier vecindad abarcará
puntos donde no sea diferenciable. Así, la función:

f (x , y ) = x 2 −x+ y + i( y 2−5 y−x ) es continua en

todos los puntos del plano, derivable en los puntos


ubicados sobre la recta y = x + 2 y no es analítica en
ningún punto.

3. Si una función es derivable en todo el plano, también es analítica en todos los puntos del plano,
dado que alrededor de un punto cualquiera se puede generar un vecindad de radio arbitrario y en
todos los puntos interiores a la vecindad, será derivable. Una FVC derivable y analítica en todo el
plano, se denomina Función Entera.
Así la función: h( x , y) = x 2− y 2 − y+i(2 x y+ x ) es continua, derivable y analítica en todo el plano

complejo. Función Entera.

5. FUNCIONES ARMONICAS
Sea φ(x,y) una función con de dos variables reales, con primeras y segundas derivadas parciales
continuas, en determinado dominio del plano; se denomina FUNCIÓN ARMONICA si satisface la

∂2 ϕ ∂2 ϕ
Ecuación de Laplace: + = 0 → ∇ 2ϕ = 0
∂ x 2 ∂ y2

Cuando una FVC es Analítica en determinada región, sus componentes real e imaginaria, u y v ,
respectivamente, son Funciones Armónicas en esa región. Para demostrarlo, partimos de una
función f(x,y) = u(x,y) + i v(x,y) , analítica en cierto dominio del plano complejo y por lo tanto,
satisface las ECR; si volvemos a derivar la primera de estas ecuaciones respecto a x y la segunda,
respecto a y:
∂u ∂v ∂2 u ∂2 v ∂u ∂v ∂2 u ∂2 v
= → = = − → = − sumando miembro a
∂x ∂y ∂ x2 ∂ y ∂x ∂y ∂x ∂ y2 ∂x∂ y

∂2 u ∂2 u ∂2 v ∂2 v ∂2 u ∂2 u
miembro las ecuaciones: + = − =0 → + =0 (5)
∂ x2 ∂ y2 ∂ y ∂ x ∂x ∂ y ∂ x2 ∂ y2

MATEMATICA APLICADA - MATEMATICA III – Cursada 2020/2021. María C. Ibarra - 8


Nuevamente partimos de las ECR y realizamos las segundas derivadas parciales, pero en este caso
la primer ecuación la derivamos respecto de y, mientras que la segunda derivamos respecto a x.
∂u ∂v ∂2 u ∂2 v ∂u ∂v ∂2 u ∂2 v
= → = − = → − =
∂x ∂y ∂x ∂ y ∂ y2 ∂y ∂x ∂ y ∂x ∂ x2

∂2 u ∂2 u ∂2 v ∂2 v ∂2 v ∂2 v
− = + = 0 → + = 0 (6)
∂ x ∂ y ∂ y ∂x ∂ y2 ∂ x2 ∂ x2 ∂ y2

Con las expresiones (5) y (6) queda demostrado que las componentes real e imaginaria, de una
función analítica, son funciones armónicas, dado que cumplen la Ecuación de Laplace.
Cuando dos funciones armónicas, satisfacen las ECR, se denominan ARMONICAS CONJUGADAS. De
manera que siendo f(x,y) = u(x,y) + i v(x,y) una función analítica, las funciones u y v son armónicas
y además entre ellas, son armónicas conjugadas. Así, al disponer un par de funciones de dos
variables reales que son entre sí armónicas conjugadas, como las componentes real e imaginaria
de una FVC, la función así generada, será analítica.

Ejemplo 2: Ya hemos determinado que la función: h( x , y) = x 2− y 2 − y+i(2 x y+ x) es entera, o


sea, analítica en todo el plano. Ahora comprobaremos que sus componentes real e imaginaria son
armónicas.
∂u ∂2 u ∂u ∂2 u
u( x , y) = x 2− y 2 − y ; =2x → = 2 ; = −2 y −1 → = −2 → ∇ 2u = 0
∂x ∂ x2 ∂y ∂ y2
∂v ∂2 v ∂v ∂2 v
v ( x , y ) = 2 x y +x ; = 2 y +1 → = 0 ; = 2 x → =0 → ∇2v = 0
∂x ∂ x2 ∂y ∂ y2
No solo son armónicas cada una de estas funciones, sino que además satisfacen las ECR de manera

que u( x , y) = x 2− y 2 − y es la armónica conjugada de v ( x , y ) = 2 x y +x y viceversa.

5.1 FUNCIONES ARMONICAS CONJUGADAS


Dada una función de dos variables reales que sea armónica, es posible encontrar su armónica
conjugada, de manera que ambas determinen una función compleja analítica. Para encontrar la
armónica conjugada se recurre a ECR.
Sea u(x,y) una función armónica, para encontrar su armónica conjugada, v(x,y), de manera que
f(x,y) = u(x,y) + i v(x,y) sea analítica en todo el plano complejo, partimos de la primera ECR:
∂u ∂ v
=
∂x ∂y
→ ∫ dv = ∫ ( ∂u
∂x
) dy → v (x , y ) = ϕ (x , y) + C (x)

MATEMATICA APLICADA - MATEMATICA III – Cursada 2020/2021. María C. Ibarra - 9


Para determinar v(x,y) aún falta encontrar C(x) y lo haremos utilizando la segunda ECR:
∂u ∂v ∂u ∂ ϕ dC dC ∂u ∂ ϕ ∂u ∂ ϕ
=− → = −[ + ] → =− − → C (x ) = ∫ (− − ) dx
∂y ∂x ∂y ∂x dx dx ∂y ∂x ∂y ∂x

El orden en que se utilicen la ECR es indistinto, llegándose siempre al mismo resultado. Las FVC
analíticas son muy frecuentes en las aplicaciones físicas de Campos Vectoriales Bidimensionales y
sus componentes real e imaginaria tienen interpretaciones físicas específicas, en cada caso.

Curvas Ortogonales
Las funciones armónicas conjugadas, son Curvas Ortogonales, es decir que se cortan ó intersectan
perpendicularmente. Para demostrar esta propiedad se utilizan ECR.
Sean u y v dos funciones armónicas conjugadas, si igualamos a una constante cada una de ellas,
encontraremos las correspondientes Curvas de Nivel: u( x , y) = c v (x , y ) = k cykϵR

Diferenciando, podemos despejar dy/dx, que representa la pendiente de la Recta Tangente a cada
curva:
∂u ∂v
∂u ∂u dy dy ∂x ∂ v ∂ v dy dy ∂x
du= + =0 → =− (7) dv= + =0 → =− (8)
∂ x ∂ y dx dx ∂u ∂ x ∂ y dx dx ∂v
∂y ∂y
∂v ∂u
dy ∂x ∂y
En la expresión (8), aplicamos ECR: =− = (9)
dx ∂v ∂u
∂y ∂x
Las expresiones (7) y (9) representan las pendientes de las Rectas Tangentes a las funciones u y v,
respectivamente, haciendo el producto comprobamos que son perpendiculares:
∂u ∂u
∂x ∂ y
− . = −1 CONDICION CURVAS ORTOGONALES
∂u ∂u
∂ y ∂x

Ejemplo 3: Sea ϕ ( x , y) = −4 x y . Comprobar que es armónica, hallar su armónica conjugada y

expresar la función analítica, donde φ sea su componente real. Luego elegir un punto del plano y
comprobar que en dicho punto las curvas son ortogonales.
∂ϕ ∂2 ϕ ∂ϕ ∂2 ϕ
= −4 y ; = 0 ; = −4 x ; = 0 → φ es armónica.
∂x ∂ x2 ∂y ∂ y2

MATEMATICA APLICADA - MATEMATICA III – Cursada 2020/2021. María C. Ibarra - 10


Para hallar la armónica conjugada, denominaremos u a φ, de manera que coincida con la

∂u ∂v
nomenclatura de ECR: = −4 y = → v(x , y ) =−2 y2 + C (x) , ahora hallamos C(x):
∂x ∂y
∂u ∂v
= −4 x =− → −4 x = −[ C I (x )] → C (x ) = 2 x 2 + k → v ( x , y ) = −2 y 2 + 2 x 2
∂y ∂x
La constante de integración k ϵ R, en este caso se le asignamos el valor 0.

Así la función analítica buscada es: f (x , y ) = −4 x y + i (−2 y 2 + 2 x 2)

Elegimos el punto (4,2) y determinamos las curvas de nivel de ambas funciones que pasan por
dichos puntos:
8
u(4 ,2) = −32 → −4 x y = −32 → y = (10)
x

v (4 ,2) = 24 → −2 y 2 +2 x 2 = 24 → x 2 − y 2 = 12 (11)

Las hipérbolas dadas por (10) y (11) tienen en común el punto (4, 2) y las pendientes de las RT
vienen dadas por:
8 1 x
yI = − 2
→ yI (4) = m1 =− ; yI = → y I (4 ,2) = m2 = 2 → perpendiculares
x 2 y

Las expresiones u( x , y) = c ; v (x , y ) = k representan dos familias infinitas de curvas y al elegir

un punto del plano, estamos asignando un valor específico a las constantes c y k, de manera que
seleccionamos una de las infinitas curvas posibles, de cada una de ambas familias; las únicas que
pasan por ese punto.

MATEMATICA APLICADA - MATEMATICA III – Cursada 2020/2021. María C. Ibarra - 11


ANALISIS DE CAMPOS VECTORIALES CON FUNCIONES COMPLEJAS

1. Representación de Campos Vectoriales mediante FVC

El Campo Vectorial bidimensional: F(x,y) = P(x,y) î + Q(x,y) ĵ , se puede expresar como una FVC:
f (x , y ) = P(x , y ) + i Q( x , y ) (1)

Este Campo Vectorial Bidimensional (CVB) asigna un vector a cada punto del plano, equivalente a
la imagen de la función compleja en dicho punto; siendo así una posible representación gráfica de
la función. Los CV tienen importantes aplicaciones en:
• Flujo de Fluidos, donde el CV representa el Campo de Velocidades; siendo P(x,y) y Q(x,y)
las componentes del Vector Velocidad, tangentes en cada punto a la trayectoria del fluido.
• Distribución de Temperaturas, donde el CV representa la Densidad de Flujo de Calor y el
vector en cada punto es tangente a la trayectoria ó Lineas de Corriente/Flujo.
• Campos Eléctricos, donde el CV representa la Densidad de Flujo Eléctrico y el vector en
cada punto es tangente a las Líneas de Fuerza del Campo Eléctrico.

En todos los casos, trabajamos con CVB, de manera que suponemos que la distribución del
Flujo/Campo es idéntica en todos los planos paralelos al xy, y la magnitud depende solamente de
la posición del punto.

Ejemplo 1 : Sea el Campo Vectorial: F( z ) = z̄ = x − iy

F(1, 1) = (1, -1) F(2,-1) = (2, 1)


F(0, -1) = (0, 1) F(-1, -2) = (-1, 2)
Si elegimos algunos puntos del plano y
representamos el vector que le asigna el
Campo Vectorial, con inicio en el punto,
obtenemos la representación de la Figura 1.

Figura 1

MATEMATICA APLICADA - MATEMATICA III – Cursada 2020/2021. MCIbarra - 1


Si utilizamos un programa para representar los
vectores, como en la Figura 2, la gráfica resulta
más agradable a la vista, dado que la longitud
de los vectores está normalizada para lograrlo.
Si este CVB representa el Flujo de un Fluido sobre una
placa: P(x, y) = Vx ; Q(x,y) = Vy; siendo r(t) = x(t) î + y(t) ĵ
una parametrización de la trayectoria del flujo, se
debe cumplir: Vx = xI (t) ; Vy = yI (t).
dx dy
= x ; = −y (2)
dt dt

Figura 2

La solución del Sistema de Ecuaciones Diferenciales (2) será la familia de funciones g(x,y) = c, que
representa la trayectoria del fluido en el plano y para distintos valores de la constante, se obtienen
las diferentes curvas de nivel que conforman esta familia de curvas.
dy y
= − → ln y = −ln x + C → lny + ln x = C → yx = c → yx = c
dx x
Esta familia de hipérbolas representa las trayectorias del fluido y en cada punto, el vector
determinado por f(x, y) = x – i y, es tangente a la curva.
Para comprobarlo elegimos algunos puntos: A(1, 1); B(1, -1); C(-1, -2):
F(1, 1) = (1, -1) ; y.x = 1 → y = 1/x → yI = -1/x2 → yI (1, 1) = -1
F(1, -1) = (1, 1) ; y.x = -1 → y = -1/x → yI = 1/x2 → yI (1, -1) = 1
F(-1, -2) = (-1, 2) ; y.x = 2 → y = 2/x → yI = -2/x2 → yI (-1, -2) = -2
Se observa en cada caso, que la pendiente de la RT a la curva coincide con la pendiente del vector
velocidad en el punto.

MATEMATICA APLICADA - MATEMATICA III – Cursada 2020/2021. MCIbarra - 2


2. Curvas Ortogonales. Vector Gradiente

Siendo g(x,y) una función escalar con dos variables


independientes, el Vector Gradiente se define como:
∂g ∂g
∇g = ( , ) y en cada punto del plano es
∂x ∂ y

perpendicular a la curva de nivel g(x,y) = k que pasa por


dicho punto; y por lo tanto es normal al vector tangente a
la curva. Lo podemos comprobar diferenciando g(x,y) = k
y utilizando el Producto Escalar:

∂ g dx ∂ g dy ∂ g ∂ g dx dy
+ =0 → ( , ).( , ) = 0 → ∇ g . r I (t) = 0 → ∇ g .r I (t ) = 0 (3)
∂ x dt ∂ y dt ∂x ∂y dt dt

Siendo rI(t) la derivada de la curva en forma paramétrica: r(t) = x(t) î + y(t) ĵ; coincidente con el
vector T. El resultado de la expresión (3) comprueba la condición de perpendicularidad entre los
vectores.
Dado que las componentes u y v de una función compleja analítica son familias de curvas
ortogonales, podemos concluir entonces, que la dirección de v coincide con el Gradiente de u.

3. Campos Gradientes o Conservativos

Sea el CVB: F(x,y) = P(x,y) î + Q(x,y) ĵ , si sus componentes P y Q, se pueden expresar como el
gradiente de una función escalar φ(x,y), se denomina Campo Gradiente:
∂ϕ ∂ϕ
P(x , y ) = ; Q( x, y) = → F = ∇ϕ (4) DEFINICION CAMPO GRADIENTE
∂x ∂y

En un Campo Gradiente el trabajo necesario para mover una partícula desde un punto a otro,
solamente depende de las posiciones inicial y final y no de la trayectoria, debido al Principio de
Conservación de la la Energía ( energía cinética + energía potencial = constante ), motivo por el cual
también se denominan Campos Conservativos. Los Campos Gradientes se presentan en varios
fenómenos físicos/naturales, como electromagnetismo, gravitación, flujo de fluidos y distribución
de temperaturas en régimen estable.

MATEMATICA APLICADA - MATEMATICA III – Cursada 2020/2021. MCIbarra - 3


En forma algebraica, la manera de determinar si un CV es Conservativo, consiste en determinar si
existe la Función Potencial φ(x,y), cuyas derivadas parciales en x e y, generen las componentes
respectivas del CV.

Ejemplo 2: Determinar si los CV son Conservativos: a) f(x,y) = x – i y b) h(x,y) = (x - y) + i (x+y)

a) Este CV fue analizado en el ejemplo anterior; ahora veremos si es un Campo Gradiente.


∂ϕ x2 ∂ϕ y2
= x → ϕ (x , y ) = + C( y ) ; = − y → − y = C I ( y ) → C ( y ) =− + k
∂x 2 ∂y 2

x2 − y 2
ϕ ( x , y) = Esta es la Función Potencial o simplemente, Potencial, del CV que resulta ser
2

un Campo Gradiente. Haciendo φ(x,y) = constante, se obtienen las curvas de nivel, llamadas
Curvas Equipotenciales, dado que los infinitos puntos ubicados sobre cada una de ellas, presentan
el mismo valor de potencial.
∂ϕ x2 ∂ϕ
b) = x − y → ϕ (x , y ) = − y x + C( y ) ; = x + y → x + y = −x + C I ( y ) →
∂x 2 ∂y
C I ( y ) = 2 x + y → No es posible encontrar una Función Potencial, de manera que el CV no es
Conservativo.

3.1. Campos Gradientes. Función Potencial Armónica

Si la divergencia de un Campo Gradiente es nula, la Función Potencial φ(x,y) es una Función


Armónica; para demostrarlo, expresaremos el CV como FVC:
∂ϕ ∂ϕ ∂ϕ ∂ϕ ∂2 ϕ ∂ 2 ϕ
f (x , y ) = +i → ∇ . f = ( ∂ , ∂ ).( , )= + =0 → ∇ 2ϕ = 0 (5)
∂x ∂y ∂x ∂ y ∂x ∂ y ∂ x2 ∂ y 2
La función armónica conjugada de φ(x,y) la designaremos como ψ(x, y); sus curvas de nivel
representan las Líneas de Corriente ó Líneas de Flujo y son las trayectorias que sigue el fluido ó las
líneas de campo. Así, las curvas de nivel de ambas familias son perpendiculares y en cada punto
del plano, la dirección de ψ coincide con el gradiente de φ, mostrando que el flujo del
fluido/campo, se produce en la dirección de máxima variación del potencial. La FVC analítica
formada por ambas funciones, se denomina Potencial Complejo:
Ω(x , y ) = ϕ ( x , y) + i ψ ( x , y) (6) POTENCIAL COMPLEJO

MATEMATICA APLICADA - MATEMATICA III – Cursada 2020/2021. MCIbarra - 4


4. Aplicaciones Físicas

En todos los casos, analizaremos el proceso en una placa bidimensional, paralela al plano xy y
supondremos que las condiciones y características dependen solamente de estas coordenadas y
son independientes de la coordenada perpendicular a dicho plano.

4.1 Distribución de Temperatura en Régimen Estable

Sea una placa paralela al plano xy (ver Figura 3) donde se produce la conducción de calor, bajo las
siguientes condiciones:
- Las características del material y del proceso son idénticas en todos los planos paralelos al xy y solamente
dependen de la posición del punto en R 2
- El proceso se efectúa en estado estacionario, independiente del tiempo
- Las caras laterales de la placa son aislantes, no pueden absorber ni generar calor

Figura 3 - Fuente: Variable Compleja con Aplicaciones. 2da. Ed. A. David Wunsch

La intensidad del calor en cada punto del interior del material viene dada por un CVB denominado
Densidad de Flujo de Calor: Q(x,y) = Qx (x,y) î + Qy (x,y) ĵ ; si este campo es conservativo, existe
una función potencial φ(x,y) que representa la temperatura en cada punto del material conductor
y las componentes del vector flujo de calor, son proporcionales a la variación de temperatura:
∂ϕ ∂ϕ
−k . = Q x ; −k . = Q y ; Q = −k ∇ ϕ k: conductividad térmica del material
∂x ∂y

En estado estacionario y con las paredes laterales del material conductor aisladas, el flujo neto de
calor en un elemento de volumen es nulo, es decir que la densidad de líneas de flujo que ingresan
coinciden con las que salen, no habiendo generación ni absorción de calor en el interior del
material. De manera que la divergencia del campo es nula: ∇ .Q = 0 , al igual que el flujo neto.

MATEMATICA APLICADA - MATEMATICA III – Cursada 2020/2021. MCIbarra - 5


Si la divergencia es > 0 implica que se genera calor, indicando la presencia de fuentes y si es < 0
implica que se absorbe calor, indicando la presencia de sumideros.
Planteando el producto escalar:
∂ϕ ∂ϕ ∂2 ϕ ∂2 ϕ ∂2 ϕ ∂2 ϕ
∇ .Q = ( ∂ , ∂ ).(−k , −k ) = −k − k =−k ( + )= 0 → ∇ 2 ϕ =0
∂x ∂x ∂x ∂y ∂ x2 ∂ y2 ∂ x2 ∂ y2

Se comprueba que la función potencial, que representa la temperatura, es armónica y entonces


su armónica conjugada ψ(x,y), representa las Líneas de Corriente o de Flujo, es decir las
trayectorias que sigue el flujo de calor en el interior del conductor. Las curvas de nivel de la función
potencial , son las Isotermas; de manera que en cada punto del plano, el flujo de calor se dirige en
dirección perpendicular a las isotermas, como indica la Figura 4.

Figura 4 - Fuente: Variable Compleja con Aplicaciones. 2da Ed. - A. David Wunsch

4.2 Flujo de Fluidos

Consideremos el flujo de un fluido en un plano paralelo al xy, que cumple las siguientes
condiciones:
- El flujo es estacionario y la velocidad en cada punto, depende solamente de la posición.
- El fluido es incompresible e irrotacional, es decir es un fluido ideal.

La velocidad en cada punto se representa mediante el Campo de Velocidades: V(x,y) = Vx î + Vy ĵ


Si este CV es un campo gradiente, existe una función escalar φ(x,y), tal que:
∂ϕ ∂ϕ
Vx = ; Vy= ; V =∇ϕ
∂x ∂y
∂ϕ ∂ ϕ
y si además la divergencia es nula, φ es armónica: ∇ .V = ( ∂ , ∂ ).( , ) = ∇2 ϕ = 0
∂x ∂ y ∂x ∂ y
y en ese caso, el rotor del CV - que se define mediante el producto vectorial ∇ x V - resulta el

vector nulo, indicando que no existen remolinos.

MATEMATICA APLICADA - MATEMATICA III – Cursada 2020/2021. MCIbarra - 6


Un flujo cuya divergencia es nula se denomina incompresible; siendo nulo el flujo neto que
atraviesa una unidad de volumen, es decir que en su interior, no existen fuentes ni sumideros de
fluido. Un fluido incompresible e irrotacional, se denomina Fluido Ideal.
La función ψ(x,y) – armónica conjugada de φ – representa las trayectorias del fluido y su dirección
en cada punto del plano, es perpendicular a la función potencial. Las curvas φ = ctte, se denominan
equipotenciales y las curvas ψ =ctte, son las Líneas de Corriente o de Flujo y en cada punto, el
vector velocidad, es tangente a estas líneas.

FIGURA 5

4.3 Campos Eléctricos

En una configuración de cargas electrostáticas, el Campo Eléctrico viene dado por:


E(x,y) = Ex î + Ey ĵ ; como es un campo conservativo, existe una función escalar φ(x,y), que

∂ϕ ∂ϕ
corresponde al potencial del campo, siendo: E = −∇ ϕ → E x =− ; Ey =−
∂x ∂y

En cada punto, el vector E(x,y) es perpendicular a la curva equipotencial que pasa por dicho punto
y coincide con la dirección del flujo eléctrico, es decir con las Líneas de Campo o Líneas de Fuerza;
que se dirigen en dirección del Gradiente del Potencial, así el flujo eléctrico se produce en la
dirección donde el potencial disminuye con máxima rapidez.

MATEMATICA APLICADA - MATEMATICA III – Cursada 2020/2021. MCIbarra - 7


Figura 6. Fuente: Física Universitaria. Vol 2. 12da Ed.- H. Young – R. Freedman.

En una región libre en su interior de cargas eléctricas, la divergencia del Campo Eléctrico es nula, y
el potencial es una función armónica, en tal caso las Líneas de Fuerza del Campo o trayectoria del
flujo, se pueden obtener mediante las curvas de nivel de ψ(x,y) , armónica conjugada del
potencial.

En el cuadro siguiente se resumen las interpretaciones físicas de las componentes del Potencial
Complejo: Ω(x , y ) = ϕ ( x , y) + i ψ ( x , y) , para las aplicaciones desarrolladas:

Ω φ ψ
Distribución de Temperaturas Potencial Complejo Temperatura Líneas de flujo.
de Temperaturas Curvas nivel: Isotermas Trayectoria flujo de calor
Flujo de Fluidos Potencial Complejo Potencial de Velocidad Lineas de flujo ó de
de Velocidad corriente.
Trayectoria fluido.
Campos Eléctricos Potencial Potencial eléctrico Líneas de Campo
Electrostático Eléctrico o Líneas de
Complejo Fuerza

MATEMATICA APLICADA - MATEMATICA III – Cursada 2020/2021. MCIbarra - 8


Ejemplo 3: Anteriormente hemos trabajado con el CVB: F( z ) = z̄ = x − iy y comprobamos

x2 − y 2
que es un Campo Gradiente con potencial: ϕ ( x , y) = y también determinamos que las
2

líneas de flujo vienen dadas por la familia de hipérbolas: yx = c

Utilizando el hecho que φ es armónica, vamos a encontrar las trayectorias del fluido mediante su
armónica conjugada y comprobaremos el resultado.
∂ϕ ∂ψ ∂ψ ∂ϕ ∂ψ
= → x= → ψ (x , y ) = x y + C( x ) ; =− → − y =− [ y + C I ( x)] → C = ctte
∂x ∂y ∂y ∂y ∂x
entonces: ψ (x , y ) = x y ; algunas de las curvas de esta familia, están representadas en la

Figura 2, con trazo azul.

Ejemplo 4: Sea el campo de velocidades dado por: f (x , y ) = (x + y) + i( x− y ) Comprobar que es

un campo gradiente, determinar las trayectorias del flujo y verificar la ortogonalidad de dichas
trayectorias con la curva equipotencial que pasa por el punto cuyo vector velocidad es (0, 4).

∂ϕ x2 ∂ϕ y2
x+ y= → ϕ (x , y ) = + y x + C ( y ) ; x− y = = x + C I ( y ) → C ( y ) =− + C
∂x 2 ∂y 2
x2 y2
ϕ (x , y) = + yx − → campo conservativo
2 2
∂2 ϕ ∂2 ϕ
= 1 ; =−1 → φ es armónica → fluido incompresible
∂ x2 ∂ y2
Para hallar la armónica conjugada aplicamos ECR:
∂ψ y2 ∂ψ x2
= x + y → ψ ( x , y) = x y + + C( x) ; − = x − y → −( y +C I ( x)) → C (x ) = − + C
∂y 2 ∂x 2
y2 x2
ψ (x , y ) = x y + − → trayectoria del flujo
2 2
V = (0 , 4) → P (2 ,−2) → ϕ (2 ,−2) = −4 ; ψ (2 ,−2) = −4
Las curvas de nivel de ambas familias, que pasan por el punto (2, -2) con sus respectivas
pendientes, son:
x2 y2
ϕ= + yx− = −4 → y I (2 ,−2) = 0 → RTH
2 2
y2 x2
ψ =x y+ − = −4 → y I (2,−2)= ∞ → RTV
2 2

MATEMATICA APLICADA - MATEMATICA III – Cursada 2020/2021. MCIbarra - 9


En la Figura 8 se representa el campo vectorial de velocidades y en trazo azul, algunas curvas
equipotenciales.

Figura 7 Figura 8

MATEMATICA APLICADA - MATEMATICA III – Cursada 2020/2021. MCIbarra - 10


MAPEO EN EL PLANO COMPLEJO

La representación gráfica de Funciones de una Variable Compleja (FVC), requiere de


procedimientos significativamente diferentes, respecto de los utilizados para graficar Funciones de
Variable Real; el MAPEO o TRANSFORMACION es una de estas estrategias y para realizarlo son
necesarios dos elementos:
1- La Ley del Mapeo o Transformación.
2 – El elemento al cual se le aplicará el Mapeo.
La Ley del Mapeo es la función w = f(z) que se representará, mediante su aplicación al
elemento/objeto que se pretende mapear; el cual puede ser un punto, una curva/función ó una
región del plano (finita o infinita).
Trabajaremos con tres tipos de funciones:
- Funciones lineales: w = a z +b a, b ϵ C → Mapeos/ Transformaciones Lineales

- Función cuadrática: w = z2 → Mapeo/ Transformación Cuadrática

a
- Funciones racionales: w= a, b, d ϵ C → Mapeos/ Transformaciones Inversas
b z+ d

Es necesario encontrar las componentes u y v de la ley de mapeo: f(z) = u(x,y) + iv(x,y) y trabajar
analíticamente para expresar la región/elemento a mapear en términos de las variables u-v.
Dependiendo de la Ley y del elemento a mapear, este proceso puede tener distintos niveles de
complejidad algebraica.
Cuando se mapean dos curvas/funciones con puntos en común, estos se denominan PUNTOS
FRONTERA y son importantes al momento de verificar la transformación, dado que el
transformado de un Punto Frontera en el plano z, debe ser también Punto Frontera en el plano w.
Es decir, si las curvas C 1 y C2 tienen en común el punto L, cuando se efectúa el mapeo, se obtienen
C1’ y C2’en el plano w, y ambas se intersectan en L’, siendo éste último el transformado del punto L.

FIGURA 1 – Puntos Frontera

MATEMATICA APLICADA - MATEMATICA III – Cursada 2020/2021. MCIbarra - 1


1. Mapeos Lineales
Los mapeos lineales son los más sencillos, no solamente por la simplicidad de las expresiones
algebraicas que involucran, sino principalmente porque es el único tipo de mapeo que mantiene la
forma original del elemento/región mapeada; es decir una recta se transforma en otra recta, una
circunferencia en otra circunferencia, un anillo en otro anillo y así sucesivamente. Los únicos
cambios que se pueden producir son de posición (rotación y/o traslación) y de tamaño (aumento/
disminución); pero se mantiene la forma.
Las constantes de la función: w = a z +b son las que determinan las modificaciones de posición
y tamaño. La constante b produce traslación en ambos ejes coordenados; mientras que a genera
cambios de tamaño (según |a| sea mayo ó menor a la unidad) y también rotación (según sea el
argumento de este número complejo).

Ejemplo 1: Aplicar la Ley: w = 2 z−i+ 2 al interior del cuadrado unitario de vértices:


(0,0); (0, 1); (1,1) y (1, 0), indicado en la Figura 2.

En primer lugar trabajamos la ley de mapeo, para encontrar u(x,y) y v(x,y); y así poder transformar
el cuadrado unitario desde el plano z (x-y) al plano w (u-v).
w = 2 z−i+ 2 = 2(x +iy)−i+2 = (2 x +2)+i(2 y−1) → u ( x , y) = 2 x+2 ; v( x , y) = 2 y−1 (1)
Dado que este es un mapeo lineal, sabemos que la región transformada también será el interior de
un cuadrado, por lo tanto podemos simplemente mapear los cuatro vértices de acuerdo a la
expresión (1) y luego unirlos, para obtener la región en el plano w:

O (0,0) → O ' (2 ,−1) A(0,1) → A' (2,1) B(1,1) → B ' (4,1) C(1,0) → C ' (4 ,−1)

El cuadrado del plano w, respecto al del plano z, tiene modificaciones en cuanto a tamaño y
posición:
- La longitud de los lados se ha duplicado, debido a la constante 2 que multiplica a z en la ley de
mapeo; pero no ha sido rotado, dado que este número se representa mediante un vector de
argumento nulo.
- La región se ha trasladado dos unidades horizontalmente a la derecha y una unidad verticalmente
hacia abajo, debido a la constante (2-i), que es el término independiente de la Ley.

MATEMATICA APLICADA - MATEMATICA III – Cursada 2020/2021. MCIbarra - 2


FIGURA 2 – Plano z FIGURA 3 – Plano w

Ejemplo 2: Otra manera de mapear el cuadrado unitario del ejemplo anterior, es considerar la
región limitada por las cuatro rectas: y = 0; y = 1; x = 0; x = 1. En este caso se mapea una a una las
rectas y la región en el plano w será la encerrada por las transformadas de las cuatro rectas. Para
realizar este mapeo se utiliza la expresión (1):
y = 0 → u = 2 x+ 2 ; v = −1 y = 1 → u = 2 x +2 ; v = 1
x = 0 → u = 2 ; v = 2 y−1 x = 1 → u = 4 ; v = 2 y −1
Se obtienen así las rectas: v = 1 ; v =−1 ; u = 2 ; u = 4 que limitan el cuadrado en el

plano w. Los puntos de intersección de dichas rectas son O’, A’, B’ y C’ ; son los PUNTOS FRONTERA.

Para comprobar la transformación realizada se pueden elegir algunos Puntos de Prueba, interiores
o exteriores. Todo punto interior a la región original en el plano x-y se debe mapear en un punto
interior a la región transformada en el plano u-v; y todo punto exterior a la región del plano x-y se
debe mapear en un punto externo de la región transformada en el plano u-v.
Por ejemplo si elegimos los Puntos M(½, ½) y N(2,2); interior y exterior, respectivamente, al
cuadrado unitario del plano z, se mapean en M’(3, 0) y N’(6,3), interior y exterior, respectivamente,
al cuadrado del plano w.

En este caso, no es necesario mapear las rectas que limitan la región, es suficiente con mapear los
Puntos Frontera y luego unirlos para determinar la región transformada; pero cuando la ley de

MATEMATICA APLICADA - MATEMATICA III – Cursada 2020/2021. MCIbarra - 3


transformación no es lineal, se debe realizar el mapeo de las curvas/funciones que limitan la
región, dado que cambian significativamente la forma al efectuar el mapeo.

2. Mapeo Cuadrático
Sea la Ley de Mapeo: w = z2 → u( x, y) = x 2− y 2 ; v ( x , y) = 2 x y

Sea el cuadrado unitario de la Figura 2, la región a mapear. En este caso debemos mapear una a
una, las cuatro rectas que encierran la región.
x = 0 → u (x , y ) = − y 2 ; v ( x, y ) = 0 → semieje u (-)

y = 0 → u (x , y ) = x 2 ; v ( x , y ) = 0 → semieje u (+)
v2
x = 1 → u( x , y) = 1− y 2 ; v ( x , y) = 2 y → u = 1 −
4
v2
y = 1 → u( x , y ) = x2−1 ; v ( x , y) = 2 x → u = −1
4
Las cuatro rectas en el plano x-y, e mapean en dos semirrectas y dos parábolas, en el plano u-v.

O(0,0) → O' (0,0) A(0,1) → A ' (−1,0)

B (1,1) → B ' (0,2) C(1,0) → C ' (1,0)

Se observa en este caso la deformación del cuadrado


unitario, dado que estamos frente a un mapeo no
lineal. A pesar de no mantenerse la forma del recinto
original, los Puntos Frontera en el plano z, se mapean
en los Puntos Frontera del plano w.

FIGURA 4 - Mapeo Cuadrático

Como una forma de verificar el proceso, se toman dos puntos, uno interior y otro exterior a la
región original en el plano z: M(½, ½) → M’(0, ½) N(2,2) → N’(0, 8)

MATEMATICA APLICADA - MATEMATICA III – Cursada 2020/2021. MCIbarra - 4


3. Mapeo Inverso Puro
1
La Ley de Transformación: w= se denomina Mapeo Inverso Puro y tiene aplicaciones físicas
z

que veremos más adelante; en las cuales es necesario aplicar esta Ley de Mapeo a rectas y
circunferencias. Las rectas y circunferencias en el plano x-y , se representan mediante la Ecuación

General: A (x 2 + y 2) + B x + C y + D = 0 (2)

Recordemos: z + z̄ = 2 x ; z − z̄ = 2 i y ; z . z̄ = x 2 + y 2 (3)

Utilizando (3) la Ecuación General (2) puede expresarse de la siguiente manera:


z+ z̄ z− z̄ 1
A (z . z̄) + B( ) + C( ) + D = 0 ; y ahora aplicamos la Ley de Mapeo Inversa: z=
2 2i w
1 1 B 1 1 C 1 1 A B w̄+ w C w̄−w
A( . )+ ( + )+ ( − )+ D=0 → + .( ) + .( ) + D = 0 (4)
w w̄ 2 w w̄ 2 i w w̄ w . w̄ 2 w . w̄ 2 i w . w̄

El equivalente de las expresiones (3) para el plano u-v es:


w + w̄ = 2 u ; w − w̄ = 2 i v ; w . w̄ = u 2 + v 2

Así, la expresión (4) resulta:


A B 2u C −2 i v
2 2
+ .[ 2 2 ] + .[ 2 2 ] + D = 0 → multiplicamos ambos miembros por (u2+v2)
u +v 2 u +v 2i u +v

A + B u − C v + D(u2 +v 2 ) = 0 (5)

La expresión (5) es la Ecuación General de rectas y circunferencias en el plano u-v; mostrando así
que bajo el Mapeo Inverso, toda recta del plano x-y se convierte en otra recta o en una
circunferencia en el plano u-v; y también que toda circunferencia del plano x-y se convierte en otra
circunferencia o en una recta del plano u-v.

Si analizamos esta Ley de Mapeo como una función, vemos que su dominio son todos los
complejos, excepto z = 0, dado que en dicho valor se anula el denominador y la función, no tiene
imagen en ese punto, es decir no está definida. Aquellos puntos del plano donde una función
racional no está definida, es decir los ceros del denominador, se denominan SINGULARIDADES.
Una función racional compleja, no es continua ni derivable, en sus singularidades y no es analítica
en ninguna región que contenga alguna de dichas singularidades. Analizando qué sucede cuando
intentamos mapear las singularidades, al anularse el denominador, la imagen tiende a infinito, es
decir la singularidad “se mapea” o “se transforma” en el infinito, mostrando así que la región en el

MATEMATICA APLICADA - MATEMATICA III – Cursada 2020/2021. MCIbarra - 5


plano u-v será una región infinita, siempre que la región original en el plano x-y contenga alguna de
las singularidades de la Ley de Mapeo. En el caso del Mapeo Inverso Puro, la única singularidad es
el origen, y toda región del plano x-y que contenga (en su interior o en su frontera/borde) este
punto, se transformará en una región infinita (no acotada).

Ejemplo 3: Aplicar la Ley: w = 1/z al cuadrado unitario de la Figura 2.


Utilizando las expresiones (2) y (3) haremos el mapeo de cada una de las cuatro rectas que limitan
la región.
y = 0 → A = 0 ; B = 0 ;C = 1 ; D = 0 → −v = 0 → v = 0 → el eje x se mapea en el eje u.
x = 0 → A = 0 ;B = 1 ;C = 0 ; D = 0 → u = 0 → el eje y se mapea en el eje v.
12 1
y − 1 = 0 → A = 0 ; B = 0 ; C = 1 ; D = −1 → −v −(u2 +v 2 ) = 0 → u2 + (v + ) = → la recta
2 4
y = 1 se transforma en la circunferencia de radio ½ y centro (0, - ½)
12 1
x − 1 = 0 → A = 0 ; B = 1 ;C = 0 ; D = −1 → u−(u2 +v2 ) = 0 → v 2 + (u− ) = → la recta
2 4
x = 1 se transforma en la circunferencia de radio ½ y centro (½, 0).
Vamos a verificar los Puntos Frontera y los Puntos de Prueba:
1 1 1−i 1 1
A (0 ,1) → w = =−i → A ' (0 ,−1) B (1 ,1) → w = = → B' ( , − )
i 1+i 2 2 2
1 1 1 1
C(1 ,0) → w = = 1 → C ' (1,0) M( , ) → w = = 1−i → M ' (1 ,−1)
1 2 2 0,5 + 0,5 i
1 2−2 i 1 1
N (2 , 2) → w = = → N'( , − )
2+ 2i 8 4 4

FIGURA 5 FIGURA 6

MATEMATICA APLICADA - MATEMATICA III – Cursada 2020/2021. MCIbarra - 6


Así, la región finita (acotada) limitada por el cuadrado unitario en el plano z, se ha transformado en
una región infinita (no acotada) en el plano w, siendo sus bordes las dos circunferencias y los ejes
coordenados, como se muestra en la Figura 6. De los cuatro Puntos Frontera del plano x-y, tres
siguen siendo Puntos Frontera en el plano u-v, mientras que el origen, al ser la singularidad, se ha
ido al infinito. Aún en estos casos, donde pasamos de una región finita a otra infinita, mediante la
transformación, los puntos interiores/exteriores de la región original, se mantienen como
interiores/exteriores en la región transformada.

3.1. Alternativa Algebraica para el Mapeo Inverso Puro


En lugar de trabajar con las Ecuaciones Generales de rectas y circunferencias (2) y (5), se puede
utilizar la Ley de Mapeo en forma cartesiana y mediante pasos algebraicos, transformar las
regiones/curvas desde el plano x-y al plano u-v.
1 1 1 x −i y x−i y x −y
w = = = . = 2 2 → u( x, y) = 2 2
; v (x , y ) = 2 2 (6)
z x+ i y x +i y x −i y x +y x +y x +y

La expresión (6) es la Ley de Mapeo Inverso Puro, en forma cartesiana. Para aplicar la
transformación, es necesario operar algebraicamente las curvas/funciones del plano z, que se
encuentran en términos de las variables x-y y llevarlas a expresiones en el plano w, en términos de
las variables u-v. Hay varias maneras de trabajar algebraicamente, una de ellas es la que se
muestra a continuación:
1. Se obtiene la Ley de Mapeo en forma cartesiana. (Esto se ha hecho en (6))
2. Mediante pasos algebraicos se expresan x e y, en términos de u y v, es decir: x(u,v) e y(u,v)
3. Se reemplazan en estas expresiones, las funciones/curvas a mapear, para dejarlas en términos
de u y v.
x −y u −y −y
A partir de (6): x2+ y2 = = → x = −y. → v = 2
= 2 2

u v v 2 u 2 2 v +u
y 2+ y y ( 2 )
v v
−y . v2 −v −v u
v = → 1 = → y = → x =
y (u 2 +v 2)
2
y (u 2 + v 2 ) u 2+ y 2 u + v2
2

u −v
x= 2 2
; y= 2 2 (7) → Paso 2
u +v u +v

El próximo paso es aplicar estas identidades , a las curvas/funciones de la región a mapear.

MATEMATICA APLICADA - MATEMATICA III – Cursada 2020/2021. MCIbarra - 7


Ejemplo 4: Al cuadrado unitario le aplicaremos el mapeo inverso, pero ahora utilizando esta otra
variante, y comprobaremos que el resultado es idéntico al obtenido mediante las Ecuaciones
Generales de rectas y circunferencias.

−v
y=0 → 0= → v=0 → eje x se transforma en eje u
u2 +v2
u
x=0 → 0= → u=0 → eje y se transforma en eje v
u +v 2
2

u 2 2 1 2 2 1
x=1 → 1= → u + v −u = 0 → (u− ) +v = → x = 1 se mapea en circunferencia
u2 +v 2 2 4

Radio: ½ y C( ½, 0)
−v 2 2 1 2 2 1
y =1 → 1= → u +v + v = 0 → (v + ) +u = → y = 1 se mapea en circunferencia
u2 + v2 2 4

R: ½ y C(0, - ½)

3.2. Aplicación al Análisis de Circuitos


En un circuito serie RLC, la Impedancia es: Z = R+i( X L−X C )

1
y la Admitancia es su recíproca: Y=
Z

FIGURA 7 – Circuito RLC

De manera que la Admitancia resulta de aplicar el Mapeo Inverso Puro a la Impedancia; mientras Z
se ubica en el plano x-y, Y lo hace en el plano u-v.
V 1
Por la Ley de Ohm: I = = V. = V. Y → La Intensidad de Corriente es directamente
Z Z

proporcional a la Admitancia y el vector que representa esta última, salvo por una cuestión de
escala, también representa a la Intensidad de Corriente del circuito.

Si R, L y C son parámetros constantes, la Impedancia es un número complejo, que se representa


con un vector y la Admitancia, será otro número complejo, con argumento opuesto:
XT
Z = R+ X T → θ = arctan( )
R

1 R X −X T
Y = = 2 2 − i 2 T 2 → ϕ = arctan( ) = −θ
Z R +X T R + XT R

MATEMATICA APLICADA - MATEMATICA III – Cursada 2020/2021. MCIbarra - 8


En el caso que uno de los parámetros R, X L ó XC sean variables, la Impedancia y la Admitancia se
representan como lugares geométricos en los planos z y w, respectivamente. Analizaremos dos
casos, según sea la Resistencia o la Reactancia, variables.

Caso 1: R y XL constantes; 0 ≤ XC ≤ X1
En este caso, el lugar geométrico de la Impedancia es un segmento de la recta vertical: x = R

Para hallar el lugar geométrico de la Admitancia, aplicamos la Ley de mapeo Inversa Pura y
utilizaremos directamente las Ecuaciones Generales (2) y (5).
u
x − R = 0 → A = 0 ; B = 1 ;C = 0 ; D = −R → −R (u 2 +v 2)+u = 0 → u2 + v 2 − =0 →
R

1 2 1
(u− ) + v2 = → circunferencia que pasa por el origen, con centro sobre eje u.
2R 4 R2

El lugar geométrico de la Admitancia, en el plano u-v, corresponde a un arco de circunferencia,


ubicado entre el primer y cuarto cuadrantes. Los Puntos Frontera A y B del plano z, se mapean en
A’ y B’; siendo los argumentos de los vectores O’A’ y O’B’, opuestos a los argumentos de OA y OB,
respectivamente.
El vector OR corresponde a la Impedancia mínima y se mapea en el vector O’R’ que corresponde a
la Admitancia máxima y por ende, a la máxima Intensidad de Corriente.
El vector OB corresponde a la Impedancia máxima y se transforma en el vector O’B’ que representa
la mínima Admitancia y por lo tanto, la mínima Intensidad de Corriente.

FIGURA 8 – Impedancia FIGURA 9 - Admitancia

MATEMATICA APLICADA - MATEMATICA III – Cursada 2020/2021. MCIbarra - 9


Caso 2: XL y XC constantes; 0 ≤ R ≤ R1
Este caso lo desarrollaremos mediante un ejemplo.

Ejemplo 5: Determinar analítica y gráficamente el lugar geométrico de la Impedancia y la


Admitancia de un circuito serie con: XL = 20 Ω; XC = 10 Ω; 0 ≤ R ≤ 20 Ω

En este caso, el lugar geométrico de la Impedancia, en el plano z, es un segmento de la recta


horizontal: y = 10 → y−10 = 0 → A = 0 ; B = 0 ;C = 1 ; D = −10 → −10(u 2 +v 2)−v = 0 →

v 1 2 1 2 1 2 1 2
u2 + v 2 + = 0 → u 2 + (v + ) = ( ) → u2 + (v + ) =( )
10 20 20 20 20

La Admitancia tiene como lugar geométrico un arco de circunferencia que pasa por el origen, radio
1/20, Centro (0, -1/20).
Comprobación de Puntos Frontera:
1 1
A (0 ,10) → w = = − i → A ' (0 ;−0,10 )
10i 10

1 20−i 10 1
B (20 ,10) → w = = → B' (0,04 ;−0,02) → (0,04)2 +(−0,02+ 0,05)2 =
20+ i10 500 400

FIGURA 10 – Impedancia Figura 11 - Admitancia

MATEMATICA APLICADA - MATEMATICA III – Cursada 2020/2021. MCIbarra - 10


El vector OA representa la Impedancia mínima y se mapea en el vector O’A’ que corresponde a la
Admitancia e Intensidad de Corriente máximas; mientras que el vector OB representa la máxima
Impedancia y se mapea en el O’B’ correspondiente a la Admitancia e Intensidad de Corriente,
mínimas.

Ejemplo 6: Determinar analítica y gráficamente la transformada de la región indicada en la Figura


12, bajo la ley w = 1/z.
x 2 + y 2 = 1 → A = 1; B = C = 0 ; D =−1 → −(u2 + v2 )+ 1 = 0 → u 2 +v 2 = 1
1
x 2 +( y−1)2 = 1 → x 2 + y 2 −2 y + 1 = 1 → A = 1 ; B = 0 ; C = −2 ; D = 0 → 2 v +1 = 0 → v = −
2

1
u2 +v 2 = 1 ; v =−
2

De manera que región transformada en el plano u-v es finita y está limitada por una circunferencia
y una recta horizontal, cuyos puntos de intersección A’ y B’ corresponden al mapeo de los Puntos
Frontera A y B. Como Punto de Prueba se ha seleccionado E(0, 3/2) cuya transformación
corresponde a E’(0, -2/3), ambos interiores a sus respectivas regiones.

FIGURA 12 - Plano z FIGURA 13 – Plano w

Otra característica del Mapeo Inverso Puro es que “invierte” la distancia de cada punto respecto al
origen; es decir si en el plano z un punto se encuentra a L unidades de distancia del origen, su
mapeado en el plano w, se encuentra a 1/L unidades de distancia del origen. Así todos los puntos
del plano z que se encuentren a una unidad de distancia del (0,0) se transformarán en puntos del

MATEMATICA APLICADA - MATEMATICA III – Cursada 2020/2021. MCIbarra - 11


plano w, que también estén a esa misma distancia del origen, motivo por el cual la circunferencia:
x2 + y2 = 1 , se mapea en: u2 + v2 = 1.
En este último ejemplo, el punto C se encuentra a 2 unidades de distancia del origen y C’ está a ½
unidades de distancia del origen; mientras que el punto E está a 3/2 unidades distancia y E’ a ⅔
unidades del (0,0). En general podemos decir, que cuanto más cercano al origen del plano z se
encuentre un punto, su mapeado en el plano w, más lejos estará del (0,0).
En la Fig 12 se observa que el punto C es el más alejado del origen de la región original, mientras
que en la Fig 13 se muestra que C’ es el punto más próximo al origen de toda la región
transformada.

4. Mapeo Inverso General


Cuando la Ley de Mapeo se corresponda con una función racional de la forma:
b
w= b, d, e: constantes complejas
dz + e

se denomina Mapeo Inverso General, Mapeo Inverso No puro o simplemente Mapeo Inverso. Para
trabajar algebraicamente este tipo de mapeos, aplicaremos el procedimiento de tres pasos
descrito en el la Sección 3.1.

Este tipo de mapeo puede generar regiones transformadas infinitas o finitas, dependiendo de la
región original y de la ubicación de la singularidad de la Ley, es decir del cero del denominador, que
en la expresión general se encontraría en: z = -e/d y corresponde al único punto del plano z, cuyo
transformado se corresponde con el infinito.

• Si se mapea una región finita que contiene la singularidad, resulta en una región infinita
• Si se mapea una región finita que no contiene la singularidad, resulta en otra región
también acotada.

Es importante, antes de iniciar el trabajo algebraico, detectar la ubicación de la singularidad,


respecto de la región a mapear y ubicar ambas en un gráfico en el plano x-y. Decir que una región
“contiene” la singularidad, significa que se ubica en su interior ó en el borde/frontera de la región.

MATEMATICA APLICADA - MATEMATICA III – Cursada 2020/2021. MCIbarra - 12


Ejemplo 7: Determinar analítica y gráficamente la transformada de la región ubicada en el primer
cuadrante, limitada por el eje de abscisas, la circunferencia de radio unitario con centro en (1, 0) y

i
la recta: y = 2-x; bajo la Ley: w=
z−1

Paso 1. Encontrar las componentes u y v de la Ley.


i i ( x−1)−iy i (x−1) y
w = = . = +
x+i y −1 ( x−1)+ iy ( x−1)−iy 2
( x−1) + y 2
( x−1)2+ y2
y x −1
u( x , y) = 2 2
; v(x , y ) =
(x −1) + y ( x−1)2 + y 2

Paso 2. Encontrar x(u,v) e y(u,v) mediante las expresiones encontradas en el paso anterior.
y x−1 u x−1 x −1
= → y = (x −1) → v = → v = →
u v v 2 u 2
2
2
2 v +u
2
( x−1) + 2 ( x−1) (x−1) ( 2 )
v v
v2 v u v u
v = 2 2
→ x−1 = 2 2 → y = . 2 2 → y = 2 2
( x−1) (u +v ) u +v v u +v u +v
v u
x−1 = 22
; y = 2
u +v u + v2

Paso 3. Aplicar la ley de Mapeo a las curvas que limitan la región a transformar.
y = 0 → u = 0 → eje de abscisas en z se mapea en eje de ordenadas en w
v2 u2 u2 + v2
(x −1)2 + y 2 = 1 → + = 1 → = 1 → u2 + v 2 = 1 → circunferencia
(u2 + v 2 )2 (u2 + v2 )2 (u 2 +v 2 )2

del plano z se mapea en circunferencia en plano w


u v u u2 + v 2 −v 1 2 1 2 1
y = 2 −x → = 2 − [1+ ] → = → (u− ) + (v − )= →
u 2+v 2 u2 +v 2 u 2 +v 2 u 2 +v 2 2 2 2

recta del plano z se mapea en circunferencia del plano w

Dado que la singularidad de la Ley está incluida en la región a mapear, el resultado del mapeo será
una región infinita y el eje v, así como las dos circunferencias, serán los bordes/fronteras de dicha
región.

MATEMATICA APLICADA - MATEMATICA III – Cursada 2020/2021. MCIbarra - 13


FIGURA 14 – Región original FIGURA 15 – Región Mapeada

Para finalizar, verificamos los Puntos Frontera:


i i i
A (0,0) → w = = = −i → A ' (0 ,−1) B(1,1) → w = = 1 → B' (1,0)
z−1 −1 1+ i−1
i
C(2,0) → w = = i → C ' (0,1)
2−1

En el plano w existen dos regiones infinitas, limitadas por el eje v y las dos circunferencias, para
determinar cuál de los semiplanos corresponde a la región mapeada, se pueden tomar Puntos de

i
Prueba. Por ejemplo: P(1 , 1/2) → w = = 2 → P ' (2,0) ; vemos así que el punto P
i
1+ −1
2
interior a la región original, se mapea en P’ que se ubica en el semiplano u > 0, mostrando que este
se corresponde con la región transformada.

MATEMATICA APLICADA - MATEMATICA III – Cursada 2020/2021. MCIbarra - 14


INTEGRALES EN EL PLANO COMPLEJO

1. Introducción
En este Curso trabajaremos con Integrales de Línea en el Plano Complejo, cuyas trayectorias de
integración consisten en Contornos Cerrados Simples.

Figura 1 – CONTORNOS CERRADOS (No Simples) Figura 2 – CONTORNOS CERRADOS SIMPLES

La resolución de integrales en Variable Real y en Variable Compleja, son sustancialmente


diferentes, dado que en VR es necesario encontrar la Primitiva, aplicando las Técnicas apropiadas y
luego evaluar la integral; mientras que en VC directamente se evalúa la integral, es decir se
determina su valor/resultado numérico, sin necesidad de hallar la Primitiva; mediante la aplicación
de Teoremas / Fórmulas, que vamos a desarrollar. Al momento de resolver una integral compleja,
es fundamental determinar la ubicación de las singularidades del integrando y analizar si dichos
puntos son interiores o exteriores al recinto de integración C .
Vamos a trabajar con combinaciones de funciones enteras y racionales; las funciones enteras no
tienen singularidades, mientras que las racionales presentan singularidades en los ceros del
denominador.

Sea la integral compleja: ∮ f (z) dz C: Recinto o Curva de integración


C

Se considera positivo el sentido antihorario al recorrer C y negativo, el sentido horario. Al cambiar


el sentido de recorrido, cambia el signo de la integral.
Al expresar f(z) y dz en términos de x-y, resulta:

∮ f (z) dz = ∮ (u( x , y ) + i v ( x , y))(dx + i dy) = ∮ (u dx + iu dy + i v dx − v dy)


C C C

∮ f (z) dz = ∮ (u dx − v dy) + i ∮ (v dx + u dy) (1)


C C C

MATEMATICA APLICADA - MATEMATICA III – Cursada 2020/2021. MCIbarra - 1


2. Teorema de Cauchy – Goursat
El Teorema establece: Sea C un recinto cerrado simple y f(z) una función analítica en el interior y

sobre C, entonces: ∮ f (z) dz = 0 Teorema de Cauchy – Goursat (TCG)


C

Es decir, si f(z) no presenta singularidades en el interior del contorno C ni sobre él, la integral es
nula. Para demostrar este teorema, se utiliza el Teorema de Green, que establece una equivalencia
entre una integral de Línea sobre un contorno cerrado simple C y una integral doble en el área
encerrada por C:

∮ (P (x , y ) dx + Q(x , y ) dy) = ∬ ( δδ Qx − δδ Py ) dx dy Teorema de Green


C R

Según (1) podemos expresar la integral compleja en sus componentes real e imaginaria, cada una
de ellas en función de u y v, a las cuales podemos aplicarles el Teorema de Green; además si f(z) es
analítica en la región R limitada por el contorno C, debe satisfacer ECR:

∮ f (z) dz = ∮ (u dx − v dy) + i ∮ (v dx + u dy) = ∬ (− δδ vx − δδ uy )dx dy + i ∬ ( δδ ux − δδ vy )dx dy = 0


C C C R R

• Mediante la aplicación de este Teorema, si f(z) es una función entera, cualquiera sea el
recinto de integración, la integral es nula.
• En caso que f(z) presente singularidades, pero éstas sean exteriores al recinto, la integral es
nula.

Ejemplo 1: Determinar en cuál/es de las siguientes integrales corresponde aplicar TCG.


4 z−2 i ez cos z
a) ∮ dz b) ∮ dz c) ∮ dz
| z |=2 z2 +9 | z|=1 ( z2 −4)3 C
2
z ( z+2)

INTEGRAL a. INTEGRAL b. INTEGRAL c.

MATEMATICA APLICADA - MATEMATICA III – Cursada 2020/2021. MCIbarra - 2


4 z−2 i 4 z−2 i
a. ∮ 2
dz = ∮ dz = 0 TCG
| z |=2 z +9 | z |=2 (z+3 i)(z−3 i)

ez ez
b. ∮ dz = ∮ dz = 0 TCG
| z|=1 ( z2 −4)3 3
| z |= 1 (z−2) ( z+2)
3

En las dos integrales anteriores, las singularidades se encuentran afuera del recinto de integración,
de manera que resultan nulas, por aplicación del TCG.
En cambio, la tercera integral, presenta dos singularidades y una de ellas se encuentra en el
interior del contorno de integración, por lo tanto no es posible aplicar TCG y para resolverla
precisamos de otro teorema, que veremos más adelante.

3. Principio de Deformación de Contornos


El Principio establece que: Siendo C1 y C2 dos contornos cerrados simples; si f(z) es analítica sobre
C1, sobre C2 y en la región comprendida entre ambos, entonces:

∮ f (z) dz = ∮ f ( z) dz PRINCIPIO DE DEFORMACION DE CONTORNOS


C1 C2

Estableciendo así, que las integrales de línea complejas, alrededor de ambos contornos son
idénticas, si la función no presenta singularidades sobre los contornos ni en la región comprendida
entre ellos. Para demostrarlo, consideremos los contornos C 1 y C2, de manera que C2 esté incluido
en C1 y f(z) sea analítica sobre ambos contornos y en la región encerrada entre ellos.

Figura 3 – DEFORMACIÓN DE CONTORNOS

MATEMATICA APLICADA - MATEMATICA III – Cursada 2020/2021. MCIbarra - 3


Si efectuamos los cortes transversales AB y CD, obtenemos dos regiones limitadas por los

contornos: Cs y Ci, y por aplicación del TCG: ∮ f (z) dz = 0 ; ∮ f (z) dz = 0


Cs Ci

Planteo alrededor Cs :
A B C D
∫ f (z)dz + ∫ f ( z)dz + ∫ f (z)dz + ∫ f (z)dz = 0 (2)
D A B C

Planteo alrededor Ci :
D C B A
∫ f (z)dz + ∫ f ( z)dz + ∫ f (z)dz + ∫ f (z)dz = 0 (3)
A D C B

Sumando (2) y (3), resulta:


A B C D D C B A
∫ f (z) dz + ∫ f ( z) dz + ∫ f (z) dz + ∫ f ( z) dz+∫ f (z) dz + ∫ f ( z) dz + ∫ f (z) dz + ∫ f ( z) dz = 0
D A B C A D C B

A C D B

∫ f (z) dz + ∫ f ( z) dz + ∫ f (z) dz + ∫ f ( z) dz = 0
D B A C

∮ f ( z) dz − ∮ f ( z) dz = 0 → ∮ f (z) dz = ∮ f ( z) dz
C1 C2 C1 C2

Se concluye entonces, que el resultado de una integral de línea compleja en un contorno cerrado
simple, no depende de la forma del contorno, sino de la ubicación de las singularidades respecto a
la región limitada por el contorno. Considerando dos contornos simples cerrados, si no existen
singularidades sobre ellos ni en la región comprendida entre ambos, el resultado de la integral será
exactamente el mismo, cualquiera fuese la forma geométrica de ambos. Dado que no es relevante
la forma del contorno, en la mayoría de los casos, se utilizan circunferencias, ya que representan
los contornos cerrados simples más sencillos. Cuando ambos contornos son circunferencias, la
región comprendida entre ellos, se denomina región anular.
En la Figura 3, si imaginamos que los contornos son elásticos (flexibles), podemos deformar C 1
hasta hacerlo coincidir con C2 sin tocar ni pasar por sobre ninguna singularidad, dado que la región
anular comprendida entre ellos, está libre de singularidades; de la misma manera podemos
deformar C2 hasta que coincida con C1 , sin tocar ninguna singularidad; de aquí la denominación de
este Principio.
Sea una función f(z) que presenta singularidades en z 0 y z1 – según indica Figura 4 – si analizamos la
integral alrededor de los tres contornos indicados, por el Principio de Deformación de Contornos,

MATEMATICA APLICADA - MATEMATICA III – Cursada 2020/2021. MCIbarra - 4


se verifica: ∮ f (z) dz = ∮ f ( z) dz , porque el polígono y la circunferencia, pueden deformarse
C1 C3

hasta coincidir, sin pasar por sobre las singularidades; sin embargo la circunferencia y la elipse,
para coincidir una con otra, deberían pasar por sobre z 0 y lo mismo ocurre entre el polígono y la
elipse.

Figura 4 – DEFORMACIÓN DE CONTORNOS

4. Resolución de la Integral ∮ 1n dz
C z

Siendo n ϵ N y C : |z| = R, la singularidad de f(z) (z = 0) se ubica en el interior del contorno, de


manera que no es posible aplicar TCG y habrá que resolver la integral; para lo cual expresamos z en
forma exponencial: z = R e iθ → dz = R i e i θ d θ

2π 2π 2π 2π
1 1 R i e iθ i dθ i dθ
∮ n
dz = ∫ iθ n
R i e iθ d θ = ∫ n i nθ
d θ = n−1 ∫ (i nθ −iθ ) = n−1 ∫ i θ (n−1)
|z|=R z 0 (Re ) 0 R e R 0 e R 0 e

du 1 du −1
i θ (n−1) = u → d θ =
i (n−1)
→ n−1 ∫
R (n−1) e u
= n−1
R (n−1)
e−u

1 iθ (1−n) 1
n−1
e = n−1 [ cis(2 π (1−n))−cis 0]
R (1−n) R (1−n)

1 1
n−1
[cos(2 π (1−n))+i sen(2 π (1−n))−(cos 0+i sen 0)] = n−1 [1−1] = 0
R (1−n) R (1−n)
En este desarrollo, el argumento [2π (1-n)] es un múltiplo entero de 2π, resultando el coseno igual a 1 y el
seno, nulo.
El resultado anterior es válido para todo número natural, excepto n = 1; en este caso resulta:
2π 2π
1 1
∮ z
dz = ∫ iθ
R i e iθ d θ = i ∫ d θ = i θ = 2 π i
|z |= R 0 Re 0

MATEMATICA APLICADA - MATEMATICA III – Cursada 2020/2021. MCIbarra - 5



|z|= R
1
zn
dz =
{ 20π i n=1
n≠ 1
(4)

El desarrollo anterior, también es válido cuando la singularidad es un punto cualquiera del plano,
z0, en cuyo caso el contorno de integración será una circunferencia con centro en z 0 y radio R:


| z −z0 | = R
1
(z−z 0 )n
dz =
{ 20π i n=1
n≠1
(5)

5. Fórmula de la Integral de Cauchy


El TCG permite evaluar integrales complejas en casos donde el integrando es una función analítica
sobre el recinto y en su interior, pero si la singularidad se encuentra en el interior del recinto, no es
aplicable y para estos casos, debemos aplicar la Fórmula de la Integral de Cauchy que
desarrollamos a continuación.

Sea f(z) una función analítica sobre C y en su interior y sea C 0 una circunferencia de centro z 0 y
radio r , según indica la Figura 5. Mediante la Fórmula de Cauchy es posible evaluar integrales que
contienen singularidades en el interior del contorno de integración, y resulta según esta Fórmula:

f (z)
∮ z−z0
dz = 2 π i f (z0 ) FORMULA DE INTEGRAL DE CAUCHY
C

Según la Fórmula de Cauchy, el resultado de la integral


es el producto de la constante (2π i) multiplicada por la
componente analítica del integrando, evaluada en la
singularidad. A continuación se realiza la demostración.

Figura 5

5. 1 Desigualdad ML
Para demostrar la Fórmula de la Integral de Cauchy, será necesario utilizar esta Desigualdad, que
establece una cota superior para una integral de línea compleja:

MATEMATICA APLICADA - MATEMATICA III – Cursada 2020/2021. MCIbarra - 6


|∫ f (z) dz | ≤ M . L
C
DESIGUALDAD ML

Siendo M una cota superior de f(z) en la trayectoria de integración: |f(z)| ≤ M; y L es la longitud de


la curva de integración. De esta manera es posible determinar un valor máximo (número real
positivo), que no será superado por la integral, sin necesidad de resolverla.

5. 2. Demostración de la Fórmula de Cauchy


f (z)
La integral: ∮ z−z0
dz presenta una singularidad en el interior de C 0 , pero la región
C

comprendida entre ambos contornos está libre de singularidades, de manera que podemos aplicar

f (z) f ( z)
el Principio de Deformación de Contornos: ∮ z−z0
dz = ∮ z−z 0
dz ; siendo f(z) analítica en
C C0

este dominio, f(z0) está definida; sumando y restando esta imagen, resulta:
f ( z) f ( z) − f ( z0 ) + f (z 0 ) f (z) − f ( z0 ) f ( z0 )
∮ z−z 0
dz = ∮ z−z 0
dz = ∮ z−z 0
dz + ∮ z−z0
dz (6)
C0 C0 C0 C0

Resultan así dos integrales en el lado derecho de (6); aplicando la Desigualdad ML al integrando de
la primera de estas integrales:

| f ( z) − f ( z0 )
z−z 0
dz ≤| |f ( z) − f ( z0 )| | |
|z−z 0|
dz ≤ ε 2π r = 2π ε → 0 →
r

C0
f (z) − f (z0 )
z−z0
dz = 0

Para el desarrollo anterior, siendo f(z) analítica en el dominio considerado, de manera que es
continua y por lo tanto, es posible hacer tender z → z0 , y acercarse a la singularidad tanto como se
desee; cuanto más próximos estén z y z 0, menor será la diferencia entre las imágenes de f(z) para
ambos puntos y siendo ϵ un infinitésimo, podemos expresar: f(z) – f(z0) → ϵ; además: z – z0 = r,
siendo C0 : | z – z0| = r, resultando la cota superior M = (ϵ/r) y L es el perímetro de C 0. Así, la
expresión (2πϵ) resulta la cota superior para la integral y dado que ϵ → 0, la integral también

f ( z) − f (z 0 )
tiende a cero y podemos asumir que: ∮ z−z0
dz = 0
C0

Ahora debemos resolver la segunda integral que aparece en (6), para lo cual directamente

f (z ) dz
aplicamos la expresión (5): ∮ z−zo dz = f (z o ) ∮
z−zo
= 2 π i f (z o ) ; así reemplazando en (6):
C0 o C0

MATEMATICA APLICADA - MATEMATICA III – Cursada 2020/2021. MCIbarra - 7


f ( z) f ( z) − f ( z0 ) f (z 0 ) f (z)
∮ z−z 0
dz = ∮ z−z 0
dz + ∮ z−z0
dz = 2 π i f ( z0 ) → ∮ z− z0
dz = 2 π i f (z0 ) (7)
C0 C0 C0 C

De esta manera, tenemos una herramienta para evaluar integrales complejas sobre trayectorias
cerradas, cuando la singularidad se encuentra en el interior del recinto. El procedimiento consiste
en evaluar la componente analítica del integrando en la singularidad y multiplicar por (2πi).

2
Ejemplo 2: Evaluar la integral: ∮ zz2+3
+4
i
dz siendo C: |z+3i| = 2
C

En primer lugar, graficamos el recinto de integración y ubicamos las singularidades:

Siendo el integrando una función racional, las


singularidades son los ceros del denominador:
z2 +4 = 0 → z = ±2i → z 2 +4 = (z+2 i) ( z−2 i)
Una de las singularidades se encuentra en el
interior de C, de manera que corresponde
aplicar la Fórmula de Cauchy. Con centro en la
singularidad -2i, generamos un contorno C0 con
un radio apropiado, de manera que sea interior a C
y factoreamos el denominador, para detectar f(z):

z2 +3 i z2 + 3i z2 +3 i dz z 2+ 3 i
∮ 2 dz = ∮ ( z−2 i)( z+2 i) dz = ∮( )
z−2 i z+2 i
→ f ( z) =
z−2 i
; z0 = −2 i
C z +4 C C0

Luego a la componente analítica del integrando f(z), debemos evaluarla en z0:


(−2i )2 + 3 i −4+3 i
f (z 0 ) = = ; ahora aplicamos la Fórmula de Cauchy:
−2 i−2i −4 i
2 2
∮ z 2+3 i dz = 2 π i (
−4 +3 i
−4 i
) = π (4−3 i)
2
→ ∮ z 2+3 i dz = π (4−3 i)
2
C z +4 C z +4

5. 3. Generalización de la Fórmula de Cauchy para dos o más singularidades


El desarrollo anterior, de la Fórmula de Cauchy, está planteado para el caso en que una
singularidad se encuentre en el interior del recinto de integración, pero esta Fórmula se puede
extender a un número cualquiera de singularidades ubicadas dentro del recinto, simplemente

MATEMATICA APLICADA - MATEMATICA III – Cursada 2020/2021. MCIbarra - 8


sumando las integrales que correspondan a cada una de ellas. Realizaremos la demostración para
el caso en que hayan dos singularidades en el interior del recinto, y este mismo procedimiento se
puede extender a n singularidades.
f (z)
Sea la integral: ∮ ( z−z ) ( z−z ) dz ; si ambas singularidades se ubican en el interior de C,
C 1 2

encerramos cada una de ellas con una circunferencia centrada en la singularidad y elegimos un
radio conveniente de manera que ambos recintos C 1 y C2, no tengan puntos en común y sean
interiores a C, como indica la Figura 6. Si realizamos tres cortes transversales AB, DE y FG,
obtenemos dos contornos cerrados simples libres de singularidades, donde es posible aplicar TCG.

Figura 6 - FORMULA DE CAUCHY PARA DOS SINGULARIDADES

En el desarrollo siguiente designaremos como g(z) al integrando, por razones de simplicidad.


Planteamos TCG para el recinto superior:
A B D E F G

∫ g( z) dz + ∫ g( z) dz + ∫ g( z) + ∫ g( z) dz + ∫ g( z) dz + ∫ g( z) dz = 0
G A B D E F

Planteamos TCG para el recinto inferior:


G F E D B A
∫ g( z) dz + ∫ g (z) dz + ∫ g (z) + ∫ g( z) dz + ∫ g (z) dz + ∫ g (z) dz = 0
A G F E D B

Sumando las dos expresiones, se cancelan las integrales de línea que recorren los cortes
transversales en sentidos opuestos, y resulta:
A D F G E B
∫ g (z) dz+∫ g (z) dz+∫ g (z) dz+∫ g (z) dz+∫ g (z) dz+∫ g (z) dz = 0
G B E A F D

∮ g (z) dz − ∮ g (z) dz − ∮ g (z) dz = 0 → ∮ g( z) dz = ∮ g( z) dz + ∮ g(z) dz


C C1 C2 C C1 C2

∮ g (z) dz = ∮ g (z) dz + ∮ g (z) dz (8)


C C1 C2

MATEMATICA APLICADA - MATEMATICA III – Cursada 2020/2021. MCIbarra - 9


Si en (8) reemplazamos g(z) por su expresión original, resulta:
f ( z) f (z) f (z) f (z) dz f ( z) dz
∮ ( z−z )( z−z ) dz = ∮ (z−z )( z−z ) dz + ∮ (z−z )(z−z ) dz = ∮ [ z−z ] +∮[ ]
z−z1 C z−z 1 z−z2
C 1 2 C1 1 2 C2 1 2 C1 2 2

f ( z) f ( z1 ) f (z 2)
∮ ( z−z )( z−z ) dz = 2π i [
z 1−z2
] + 2π i [
z2 −z 1
]
C 1 2

1 f ( z) f ( z1 ) f ( z2 )
2π i
∮ ( z−z )(z−z ) dz =
z 1 −z2
+
z 2 −z1
C 1 2

Así, para evaluar una integral con dos singularidades en el interior del recinto, se aplica la Fórmula
de Cauchy a cada una de la singularidades, independientemente de la otra, y luego se suman
ambos resultados parciales.

La Fórmula de Cauchy se puede extender a n singularidades, mediante la suma de las integrales

correspondientes alrededor de cada una de ellas. Sea la integral: ∮ g( z) dz con n singularidades


C

en el interior de C, según indica la Figura 7; replicando el procedimiento anterior, llegamos a la


generalización de la Fórmula de Cauchy:

∮ g (z) dz = ∮ g( z) dz + ∮ g( z) dz + ∮ g( z) dz + ............... + ∮ g (z) dz (9)


C C1 C2 C3 Cn

Figura 7 - GENERALIZACION FORMULA CAUCHY

2
Ejemplo 3: Evaluar la integral:
1
2π i
∮ z 2+3 i dz siendo C: |z| = 3
C z +4

Las singularidades son z = ± 2i y ambas se encuentran en el interior de C. En primer lugar debemos


factorear el denominador y realizar el gráfico correspondiente.

MATEMATICA APLICADA - MATEMATICA III – Cursada 2020/2021. MCIbarra - 10


Aplicando el resultado obtenido en (8):
1 z2 +3 i z 2+3 i
2π i
∮ 2 dz = 2 1π i ∮ (z−2 i) (z+ 2i) dz
C z +4 C

1 z2 +3 i dz 1 z 2+ 3 i dz
2π i
∮[ ]
z+ 2i z−2i
+
2π i
∮[ ]
z−2 i z+2 i
C1 C2

(2 i)2 + 3 i (−2 i)2 + 3 i −4 +3 i −4+3i


+ = − =0
4i −4 i 4i 4i

1 z 2 +3 i
2π i
∮ 2 dz = 0 ; este resultado muestra que el
C z +4

TCG no es recíproco, es decir, no necesariamente cuando el resultado de una integral de línea


compleja, sobre una trayectoria cerrada es nula, significa que la función sea analítica en el recinto
de integración.
Para evaluar este tipo de integrales, hay que analizar cuál es la singularidad encerrada por cada
contorno y expresar el integrando como el producto de dos factores:
- el primero de ellos, debe representar la componente analítica del integrando
- y el segundo término, es un cociente entre el diferencial y el binomio que representa la
singularidad
Luego, para aplicar la Fórmula de Cauchy, se evalúa la parte analítica en la singularidad.
Este procedimiento se aplica a cada una de las singularidades contenidas en el recinto.

cos z
Ejemplo 4: Evaluar la integral: ∮ dz ; siendo C el recinto poligonal indicado en el gráfico .
C z4 −81

En primer lugar, factoreamos el denominador:


z 4 −81 = (z 2−9) (z2 + 9) = (z−3) (z+ 3) (z−3 i) (z+ 3 i)
De las 4 singularidades, 2 se encuentran en el
interior de C y son las que debemos tener en cuenta
para resolver la integral. El contorno C1 contiene a
z = -3; mientras que C2 contiene a z = -3i.
cos z cos z
∮ dz = ∮ ( z−3)(z+ 3)(z−3 dz →
C z4 −81 C i)( z+3 i)

MATEMATICA APLICADA - MATEMATICA III – Cursada 2020/2021. MCIbarra - 11


cos z dz cos z dz cos(−3) cos(−3 i)
∮ [ ( z2 +9)(z−3) ] z+3 +∮ [ (z 2−9)(z−3 i) ] z+3 i
= 2π i[ +
(9+9)(−3−3) (−9−9)(−3i−3 i)
] →
C1 C2

cos (−3) cos(−3 i)


2π i [ + ] = π [cos(−3 i) − i cos(−3)]
−108 108 i 54

cos z
∮ dz = π [cos (−3 i) − i cos(−3)]
C z4 −81 54

6. Fórmula Extendida de Cauchy


La Fórmula de Cauchy se aplica cuando las singularidades interiores al recinto de integración son
simples, es decir con orden de multiplicidad 1; en caso que la singularidad sea de orden 2 ó
superior, es necesario recurrir a la Fórmula Extendida de Cauchy y para obtener su expresión,
partimos de la Fórmula de Cauchy (7) y utilizamos un artificio matemático, que consiste en derivar
ambos lados de la igualdad respecto a z0:
f (z) d f (z) d
∮ z−z0
dz = 2 π i f (z 0 ) → [∮
dz0 C z−z 0
dz] =
dz0
[2 π i f (z 0 )]
C

d f ( z) d 1 1
∮ dz [
z−z 0
dz] = 2 π i f I (z 0 ) → ∮ f ( z)dz . dz [
z−z0
] = ∮ f (z) dz .
(z−z0 )2
= 2 π i f I (z0 )
C 0 C 0 C

f (z)
∮ (z− z )2 dz = 2 π i f I (z 0 ) (10) → con este resultado, tenemos una Fórmula para resolver una
C 0

integral con singularidad de orden de multiplicidad 2.

Derivando la expresión (10) respecto a z0:


d f (z) d d f (z)
[∮ 2
dz ] = [2 π i f I (z 0 )] → ∮dz [ dz ] = 2 π i f II ( z0 )
d z0 C (z−z 0 ) d z0 C 0 (z−z 0 )2

2
∮ f (z)dz . = 2 π i f II (z 0)
C ( z−z0 )3

f (z) 2 π i II
∮ (z−z )3 dz =
2
f ( z0 ) (11) → con este resultado, tenemos una Fórmula para resolver una
C 0

integral con singularidad de orden de multiplicidad 3.

Derivando la expresión (11) respecto a z0:

MATEMATICA APLICADA - MATEMATICA III – Cursada 2020/2021. MCIbarra - 12


d f (z) d 2 π i II d f ( z) 2 π i III
[∮ 3
dz ] = [ f ( z0 )] → ∮dz [ 3
dz] = f (z 0)
d z0 C (z−z 0 ) d z0 2 C 0 (z−z0 ) 2

3 2 π i II
∮ f ( z) dz . 4
=
2
f (z 0)
C (z−z 0 )

f (z) 2 π i III
∮ (z−z )4 dz =
2. 3
f ( z0 ) (12) → con este resultado, tenemos una Fórmula para resolver una
C 0

integral con singularidad de orden de multiplicidad 4.

f (z) 2 π i IV
Y si volvemos a derivar (12), obtendríamos: ∮ (z−z )5 dz =
2. 3. 4
f (z 0 )
C 0

Así, es posible observar el siguiente patrón en las Fórmulas encontradas:


- El orden de la derivada es inferior en una unidad, al orden de multiplicidad de la singularidad.
- En el denominador del lado derecho, aparece el factorial del número natural coincidente con el
orden de la derivada.

De esta manera, podemos deducir la Fórmula General, para una singularidad de multiplicidad n:
f (z) 2 π i (n−1)
∮ (z−z )n dz =
(n−1)!
f ( z0 ) FORMULA EXTENDIDA CAUCHY (13)
C 0

Si bien la Fórmula Extendida se plantea para singularidades de orden ≥ 2, también es válida


cuando n = 1 y en tal caso se convierte en la Fórmula de Cauchy, siendo esta última un caso
particular de la Fórmula Extendida.
Para aplicar (13), en primer lugar debemos factorear el denominador y ubicar las singularidades en
el mismo gráfico que el recinto C; luego determinar cuál es la parte analítica del integrando y
derivar (n-1) veces, después evaluar la derivada en la singularidad y dividir por (n-1)!.
La Fórmula Extendida se puede aplicar también cuando hay más de una singularidad múltiple en el
interior del recinto, analizando cada una independientemente de las demás y luego sumando los
resultados parciales. También es posible evaluar singularidades múltiples y simples, en el mismo
recinto; sumar las integrales correspondientes a cada singularidad y luego evaluar la integral
completa, sumando los resultados parciales.

MATEMATICA APLICADA - MATEMATICA III – Cursada 2020/2021. MCIbarra - 13


1−z
Ejemplo 5: Evaluar la integral: ∮ dz siendo C : |z-2i| = 3
C z ( z2 +16)2
3

z3 (z 2 +16)2 = z 3 (z+4 i)2 (z−4 i)2 → presenta 3 singularidades y 2 son interiores a C; z = 0 es de


orden de multiplicidad 3 y z = 4i es de orden 2.

1−z 1−z
∮ dz = ∮ dz =.
C z ( z2 +16)2
3
C z ( z+4 i)2 ( z−4 i)2
3

1−z dz 1−z dz
.= ∮ + ∮
C1 z (z+4 i) ( z−4 i)2
3 2 2 2 3
C 2 ( z +16) z

La componente analítica del integrando, para C1

1−z
es: f (z) = y como es de orden de
z ( z+4 i)2
3

multiplicidad n = 2, hay que obtener la primera


derivada y evaluarla en z = 4i.

1−z
Para C2 la componente analítica es: g ( z) =
(z 2 +16)2

y en este caso n = 3, de manera que hay que hallar la segunda derivada y evaluarla en z = 0.

1−z dz 1−z dz df 1 d2 g
∮ + ∮ = 2 π i { [ ] + [ ] } =.
C1 z ( z+4 i) (z−4 i)2
3 2
C2 (z 2+16)2 z3 dz z = 4 i 2 ! dz 2 z = 0

1 3i 1 −1 −1 3i
2 π i [( − )+ ( )] = 2 π i ( − )
4096 4096 2 ! 1024 4096 4096
1−z πi
∮ 3 2 2
dz = −
2048
(1+3 i)
C z ( z +16)

MATEMATICA APLICADA - MATEMATICA III – Cursada 2020/2021. MCIbarra - 14


ECUACIONES DIFERENCIALES

1. Introducción. Clasificación

En Cálculo 1 hemos trabajado con Ecuaciones Diferenciales Lineales (EDL) de primer orden; en este
Curso abordaremos las EDL de orden superior, en sus aspectos más generales y desarrollaremos
algunos métodos básicos para resolverlas. Existe una gran cantidad y variedad de ED, desde las
simples y básicas hasta las más complejas; también es muy amplio el abanico de métodos para su
resolución; existen algunos métodos generales que resuelven varios tipos de ED, algunos más
específicos que se aplican a determinadas ED e incluso hay ED que no tienen solución analítica y en
estos casos, se debe recurrir a Método Numéricos. El estudio de las ED es una de las ramas más
interesantes y extensas del Cálculo o Análisis Matemático; por las numerosas aplicaciones que
presentan, en Física, Química, Biología, Medicina, y otras ciencias; donde constituyen los Modelos
Matemáticos que representan procesos/sistemas de distinta índole y con diferentes niveles de
complejidad, en función del tipo y cantidad de variables involucradas.

Sea la expresión general de una ED de enésimo orden:


(n) (n−1) (n−2)
an y + an−1 y + an−2 y + ............. + a 1 y I + a 0 y = g( x) (1)

En términos generales, las ED se clasifican en las siguientes categorías.

1. Según el orden.
El orden de la ED está determinado por la derivada de mayor orden que aparece en su expresión.

2. Según el número de Variables Independientes.


• Ordinarias: cuando existe una sola variable independiente
• Parciales: cuando existen dos ó más variables independientes
La ED (1) es Ordinaria.

3. Según la Linealidad
Ecuaciones Diferenciales Lineales (EDL) son aquellas donde:
I. La ED es lineal en la variable dependiente y en sus derivadas

MATEMATICA APLICADA - MATEMATICA III – Cursada 2020/2021. MCIbarra - 1


II. Los coeficientes a1, a2, …, an y g(x) son constantes o funciones de la variable independiente.
Ejemplos ED No Lineales:
1 d3 y d4 x dx
+ xy =4 y II − 3 y I + e y = 0 + 3t − x 2 = 2 + cos t
y dx3 dt 4 dt

4. Homogéneas y No Homogéneas
Ecuaciones Diferenciales Homogéneas (EDH) son aquellas donde g(x) = 0.

En esta Unidad vamos a trabajar con ED Lineales, Ordinarias, Homogéneas y No Homogéneas y


más adelante, abordaremos las Ecuaciones Diferenciales Parciales.

2. Ecuaciones Diferenciales Homogéneas

La siguiente es la expresión general de una Ecuación Diferencial Homogénea (EDH) de orden n:

(n) (n−1) (n−2)


an y + an−1 y + an−2 y + ............. + a 1 y I + a 0 y = 0 (2)

2.1 Principio de Superposición de Soluciones


Este Principio establece: siendo y1, y2, …, yk soluciones de una EDH, entonces la combinación

k
lineal: c 1 y 1 + c 2 y 2 + c 3 y 3 + .............. + c k y k = ∑ c i yi también es solución.
i=1

c1, c2, ……., ck : constantes

Vamos a demostrar este Principio para el caso de una EDH de segundo orden.

d2 y dy
Sea la ED: a2 2
+ a1 + a0 y = 0 y sean y1(x) e y2 (x) las soluciones; vamos a comprobar
dx dx

que la función: c 1 y 1 + c2 y 2 = f ( x) también es solución de la ED.

c 1 y I1 + c 2 y 2I = f I (x ) ; c 1 y 1II + c 2 y II2 = f II (x )

a2 [ c 1 y II1 + c 2 y II2 ] + a1 [ c 1 y1I + c 2 y 2I ] + a 0 [c 1 y 1 + c 2 y2 ] =.

.= c 1 [ a2 yII1 + a 1 y 1I + a 0 y 1 ] + c 2 [a 2 y2II + a1 y I2 + a 0 y 2 ] = 0

Esta demostración se extiende a EDH de orden n.

MATEMATICA APLICADA - MATEMATICA III – Cursada 2020/2021. MCIbarra - 2


El Principio de Superposición establece que todo múltiplo constante de una solución de una EDH
es también solución, así como también lo son las combinaciones lineales de dos o más soluciones.
Toda EDLH tiene como Solución: y = 0, que se denomina Solución Trivial.

2.2 Conjunto Fundamental de Soluciones


Si y1, y2, ………, yn son n soluciones linealmente independientes de una EDLH de orden n,
conforman un Conjunto Fundamental de Soluciones y cualquier otra solución de la ED se puede
expresar como Combinación Lineal de este Conjunto Fundamental. Las funciones componentes del
Conjunto Fundamental deben ser linealmente independientes, es decir, que ninguna de ellas
puede ser expresada como Combinación Lineal de las demás. Una manera de comprobar si un
conjunto de funciones es linealmente independiente (ó no), es mediante el Wronskiano, que
consiste en un determinante n x n, en el cual se involucran las funciones en cuestión y sus
derivadas hasta el orden (n-1):

[ ]
y1 y2 ..... yn
yI1 y2I ..... y In
W =
..... ..... ..... .....
(n−1) (n−1) (n−1)
y1 y2 ..... yn

Si este determinante resulta ≠ 0 entonces, las funciones: y1, y2, ………, yn son linealmente
independientes y constituyen un Conjunto Fundamental de Soluciones para la EDLH y cualquier
otra solución se puede expresar como Combinación Lineal de este Conjunto Base. Se denomina
Solución general de la ED a la Combinación Lineal del Conjunto Fundamental de Soluciones:

n
y = c 1 y1 + c 2 y 2 + c3 y 3 + ................. + c n y n = ∑ c i y i SOLUCION GENERAL EDLH (3)
i=1

Las funciones que constituyen el Conjunto Fundamental para las EDLH, son de tres tipos:
• Polinomios
• Exponenciales , de base natural y exponente lineal: eβx
• sen(βx) ; cos(βx)
• Sumas algebraicas y/o productos de estas funciones.

MATEMATICA APLICADA - MATEMATICA III – Cursada 2020/2021. MCIbarra - 3


2.3 EDLH de Segundo Orden
Vamos a desarrollar las EDLH de segundo orden, con coeficientes constantes. El interés particular
en estas ED se debe a que diversos sistemas/ procesos físicos que estudiaremos posteriormente,
se modelan mediante estas ED.

d2 y dy
Sea la EDLH: a2 2
+ a1 + a0 y = 0 (4)
dx dx

Encontraremos las soluciones y1 e y2, que conforman un Conjunto Fundamental para esta ED.

En primer lugar vamos a plantear y resolver la EDLH con coeficientes constantes, de 1er orden:
b
dy dy b dy b b − x
a + by = 0 → = − y → = − dx → ln y = − x + C → y = C1 e a
dx dx a y a a

Podemos proponer esta función exponencial como solución de la EDLH de 2do orden:
b
m =− → y = e m x → y I = m e m x ; y II = m2 e m x ; reemplazando en (4):
a

a 2 m2 e m x + a1 me m x + a0 e m x = 0 → e m x (a2 m2 + a1 m+ a0 ) = 0 (5)

La única manera que se verifique (5) es que sea nulo el término entre paréntesis, el cual se

denomina Polinomio Característico ó Ecuación Característica de la ED: P(x ) = a2 m 2 + a1 m+ a0 ;

en este caso el Polinomio es de segundo grado, porque corresponde a una ED de segundo orden;
dado que el grado del Polinomio Característico coincide con el orden de la ED.
a 2 m2 + a 1 m+ a 0 = 0 → pueden presentarse tres situaciones:

Caso I. Dos raíces reales diferentes; m1 ≠ m2


m1 x m2 x
Conjunto Fundamental de Soluciones: y1 = e ; y2 = e
m1 x m2 x
Solución General: y = c1 e + c2 e SOLUCION GENERAL - CASO I

Caso II. Una raíz real doble; m1 = m2 = m


Esta única raíz genera una sola solución: y1 = em x ; el Conjunto Fundamental debe tener dos

componentes y entonces debemos encontrar otra solución Linealmente Independiente de esta; lo


cual se consigue multiplicando por la variable independiente la función y1; entonces: y2 = x em x

MATEMATICA APLICADA - MATEMATICA III – Cursada 2020/2021. MCIbarra - 4


Conjunto Fundamental de Soluciones: y1 = em x ; y 2 = x em x

Solución General: y = c 1 em x + c 2 x em x SOLUCION GENERAL - CASO II

Caso III. Un par de raíces complejas conjugadas: m1 = α +i β ; m2 = α −i β


(α +i β ) x ( α −i β ) x
Conjunto Fundamental de Soluciones: y1 = e ; y2 = e

Mediante trabajo algebraico, se busca que las funciones solución, queden expresadas en términos
de Variable Real, mediante la Identidad de Euler.
De acuerdo al Principio de Superposición, toda combinación lineal de y 1 e y2 será también solución:
( α +i β )x (α −i β )x
y = c1 e + c2 e = c 1 eα x e i β x + c 2 eα x e −i β x =.

.= c 1 e α x [cos β x + i sen β x ] + c 2 eα x [cos β x − i sen β x ]

asignando a las constantes: c1 = c2 = 1; resulta:


y = eα x [cos β x + i sen β x] + e α x [ cos β x − i sen β x ] = 2 e α x cos β x (6)

asignando a las constantes: c1 = 1; c2 = -1; resulta:


y = eα x [cos β x + i sen β x] − e α x [ cos β x − i sen β x ] = 2 i e α x sen β x (7)

Las funciones (6) y (7) son Linealmente Independientes y constituyen un Conjunto Fundamental;
por lo tanto su Combinación Lineal es la Solución General: y = c 1 e α x cos β x + c 2 e α x sen β x

y = eα x (c 1 cos β x + c 2 sen β x ) SOLUCION GENERAL - CASO III

Ejemplo 1: Hallar la Solución General de las siguientes EDLH.


a) y II − 3 y I + 2 y = 0

Ecuación Característica: m² – 3 m + 2 = 0 → x1 = 1 ; x2 = 2
Solución General: y = c1 e x + c 2 e2 x

d2x dx
b) 2
− 10 + 25 x = 0
dt dt

Ecuación Característica: m² – 10 m + 25 = 0 → t1 = t2 = 5
Solución General: x = c 1 e 5t + c 2 t e 5t

MATEMATICA APLICADA - MATEMATICA III – Cursada 2020/2021. MCIbarra - 5


d2 y dy
c) 2 2
−3 +4y=0
dt dt

Ecuación Característica: 2m² – 3 m + 4= 0 → t 1, 2 =


3
±i
√ 23 → α = 3 ; β = √23
4 4 4 4

Solución General: y = e0.75 t [c 1 cos


√23 t + c sen √ 23 t ]
2
4 4

2.4 Problemas con Valores Iniciales o de Frontera


En la Solución General de una ED existen constantes arbitrarias también llamadas parámetros y
para determinar su valor es necesario contar con Condiciones Iniciales o Condiciones de Frontera;
es decir datos adicionales a la ED, que permitan encontrar una solución única, la cual consiste en
una función/curva que satisface la ED y también cumple las condiciones iniciales/frontera. Dado
que la Solución General de la EDLH de enésimo orden, consta de n parámetros arbitrarios, se
requieren n condiciones iniciales. Estos datos adicionales generalmente consisten en puntos
pertenecientes a la curva solución y/o a sus sucesivas derivadas. En el caso de una EDLH de 2do
orden se requieren dos condiciones, que consisten en puntos por los que pasa la curva solución ó
el valor de la pendiente en determinados puntos:
y(x0) = a ; yI(x0) = b y(x0) = a ; y(x1) = b yI(x0) = a ; yI(x1) = b

Cuando todas las condiciones se especifican para el mismo valor de la variable independiente, se
denomina Problema de Valor Inicial (PVI); mientras que cuando se especifican en distintos valores
de la variable independiente, se denomina Problema de Valor en la Frontera (PVF).

d2 y dy
Ejemplo 2: Resolver el PVI: 2
− 10 + 25 y = 0 ; y (0) = −1 ; y I (0) = 4
dt dt

y = c 1 e 5t + c 2 t e 5t → y I = 5 c1 e5 t + c 2 e5t + 5 c 2 t e 5t
A partir de la Solución General:
y (0) = c 1 =−1 y I (0) = 5 (−1) + c2 = 4 → c 2 = 9

La solución del PVI es: y (t) =− e 5 t + 9 t e5t ; se observa que esta solución, a diferencia de la general,

no presenta parámetros arbitrarios.

En la Figura 1 se indican algunas de las funciones solución de la ED, para diferentes valores de las
constantes; la curva en color verde corresponde a la solución del PVI.

MATEMATICA APLICADA - MATEMATICA III – Cursada 2020/2021. MCIbarra - 6


FIGURA 1

2.5 Ejercicios Propuestos


1. Una EDLH con coeficientes constantes, presenta un Polinomio cúbico como ecuación
característica y dos de sus raíces son: m1 = -1/2; m2 = 3 + i. Encontrar la ED.
d4 y
2. Resolver: 4
+ y = 0 , teniendo en cuenta que: m4 + 1 = (m2+1)2 – 2m2
dx

3. Ecuaciones Diferenciales No Homogéneas

Sea la expresión general de una EDL No Homogénea


(n) (n−1) (n−2)
an y + an−1 y + an−2 y + ............. + a 1 y I + a 0 y = g( x )

donde g(x) ≠ 0.

De acuerdo al Principio de Superposición, se puede descomponer esta ED en una componente


Homogénea y otra No Homogénea; se resuelven ambas y luego la solución general se encuentra
mediante la combinación de las dos soluciones.
• En primer lugar se debe resolver la EDH Asociada, haciendo g(x) = 0, y hallar la solución y H,
mediante combinación lineal de las soluciones fundamentales.
• Luego se debe encontrar una Solución Particular yP que corresponde a la componente No
Homogénea. Para determinar esta solución particular, existen métodos específicos.

MATEMATICA APLICADA - MATEMATICA III – Cursada 2020/2021. MCIbarra - 7


3.1 Principio de Superposición . Solución General
Sea una EDL no Homogénea, de enésimo orden; si y1, y2, ….., yn son soluciones de la EDH Asociada e
yp es una Solución Particular, la Solución General viene dada por la Combinación Lineal:

y = c 1 y1 + c2 y2 + c 3 y 3 + .............. + cn y n + y P = y H + y P (8)

Vamos a comprobar el Principio de Superposición para una ED de 2do orden:


d2 y dy
a2 + a1 + a 0 y = g( x) (9)
dx2 dx

d2 y dy
Sean y1 e y2, las soluciones de la EDH Asociada: a2 + a1 + a 0 y ( x) = 0 , entonces:
dx 2 dx

y H = c 1 y 1 + c2 y 2

Y sea yP una Solución Particular, entonces: a 2 y IIP + a 1 y IP + a 0 y P = g( x)

De acuerdo a este Principio, la Solución general viene dada por: y = c 1 y1 + c2 y2 + y P

a2 [ c 1 y II1 + c 2 y II2 + y IIP ] + a1 [c 1 y I1 + c 2 y I2 + y IP ] + a 0 [ c 1 y 1 + c 2 y 2 + y P ] =.

.= c 1 [ a2 y II1 + a 1 y 1I + a 0 y 1 ] + c 2 [a 2 y II2 + a1 y 2I + a 0 y 2 ] + [a 2 y IIP + a1 y IP + a 0 y P ] = g( x)

a 2 y IIP + a1 y IP + a0 y P = g( x)

A continuación, desarrollamos dos métodos analíticos para encontrar una solución particular de la
ED No Homogénea: Coeficientes Indeterminados y Variación de Parámetros.

3.2 Método de los Coeficientes Indeterminados


Este método se utiliza para hallar la Solución Particular, y P, cuando g(x) está compuesta por las
funciones siguientes:
Polinomios - Exponenciales de base natural y exponente lineal (e βx) - sen (βx) - cos (βx); como
también sumas algebraicas y/o productos de estas funciones elementales.

Las derivadas de estas funciones son de la misma familia y resultan en sumas algebraicas y/o
productos de funciones del mismo tipo. Este método consiste en proponer una yP con idéntica
estructura o muy similar a g(x), de manera que en ambos miembros de (1) aparezcan el mismo tipo

MATEMATICA APLICADA - MATEMATICA III – Cursada 2020/2021. MCIbarra - 8


de funciones, permitiendo así igualar términos semejantes y de esta manera encontrar los
Coeficientes (constantes) de la yP. Pero antes de proponer una yP es necesario hallar yH, dado que
la Solución General debe estar formada por funciones linealmente independientes. En caso que
g(x) contenga funciones que también formen parte de y H, la yP se construirá multiplicando esos
bloque básicos (funciones elementales) por potencias naturales de la variable independiente (x n),
eligiendo el mínimo exponente que consiga la independencia lineal entre las funciones ( de manera
similar al Caso II de la EDLH, donde se repite la raíz ).
Para comprobar la independencia lineal entre dos ó más funciones, se puede utilizar el
Wronskiano; pero en la mayoría de los casos no será necesario – si las funciones involucradas
pertenecen a alguna de las tres familias enumeradas más arriba – si tenemos en cuenta que:

• Los polinomios de diferentes grados, son linealmente independientes


• Las exponenciales de base natural y exponente lineal, son linealmente independientes si los
coeficientes de los exponentes, son distintos
• Las funciones seno y coseno, son linealmente independientes si los argumentos son
distintos.

Este Método para encontrar una solución particular, yP, se puede resumir en los siguientes pasos:
I. Encontrar la solución de la EDH Asociada, yH (sección 2.3)
II. Si g(x) es linealmente independiente de y H, se propone una yP con la misma estructura que g(x).
Mientras que si g(x) no es linealmente independiente de y H, se propone una yP con estructura
similar a g(x) pero multiplicada por xn (suponiendo que x es variable independiente ), siendo n el menor
número natural que permita la independencia lineal entre las funciones.
III. Se reemplaza la solución particular y sus derivadas en la ED y se obtienen los Coeficientes

(constantes) de la yP.

IV. Se encuentra la Solución General: y = yH + yP; en el caso que corresponda a un PVI ó PVF se
determinan los valores de las constantes de la solución general y se encuentra “la solución”.

En el siguiente cuadro se indican algunos ejemplos de y P a proponer, según la forma analítica de


g(x):

MATEMATICA APLICADA - MATEMATICA III – Cursada 2020/2021. MCIbarra - 9


g(x) yP g(x) yP

3 x2+5 x−1 A x 2+B x +C 2 x senx ( A x+ B) sen x+(C x+D )cosx

x3 −4 x 2 A x 3+B x2+ C x + D ( x2−3) e x ( A x 2 +B x +C) e x

cos (3 x) A cos (3 x)+ B sen (3 x) e x /2 cos(5 x) A e x /2 cos (5 x)+B e x/ 2 sen (5 x)

e−2 x A e−2 x x2 e 2 x sen(4 x) ( A x 2 +B x +C) e2 x cos(4 x)+( D x 2 +E x+ F )e2 x sen(4 x)

A, B, C, …..: constantes
CUADRO 1 - Propuesta de Soluciones Particulares

Ejemplo 3: Resolver el PVI: y II + 4 y I + 4 y = (3+x ) e−2 x y (0) = 2 ; y I (0) = 5

Paso 1. Resolvemos la EDH Asociada


EDH Asociada: y II + 4 y I + 4 y = 0 → m² + 4 m + 4 = 0 → m = -2 (raíz doble, caso II)
Solución Homogénea: y H = c1 e−2x + c 2 x e −2 x

Paso 2. Buscamos una propuesta para la solución particular


Siendo g(x) el producto entre un polinomio de 1er grado y la exponencial e -2x ; para proponer una
solución particular hay que tener en cuenta que las funciones elementales e-2x; x e-2x, aparecen
ambas en yH, entonces para conseguir una yp linealmente independiente, se propone:

y P = ( A x3 + B x 2 ) e−2x → yIP = (3 A x 2 + 2 B x) e −2 x + ( A x 3 + B x 2)(−2 e −2 x)

y IP = e−2 x (3 A x 2 + 2 B x − 2 A x 3 − 2 B x 2 )

y IIP = −2 e−2x (3 A x 2 + 2 B x − 2 A x3 − 2 B x 2 ) + e−2 x (6 A x + 2 B − 6 A x 2 − 4 B x)

y IIP = e −2 x (−12 A x2 + 4 A x 3 − 8 B x + 4 B x2 + 6 A x + 2 B)

Paso 3. Reemplazamos yP y sus derivadas en la ED

e−2x (−12 A x 2 + 4 A x3 − 8 B x + 4 B x 2 + 6 A x + 2 B) + 4 e −2 x (3 A x 2 + 2 B x − 2 A x 3 − 2 B x 2)+.

+ 4 ( A x 3 + B x 2) e −2 x = e−2 x (6 A x + 2 B) = (x +3) e −2 x

Igualamos los dos últimos términos, para encontrar los Coeficientes de y P:

MATEMATICA APLICADA - MATEMATICA III – Cursada 2020/2021. MCIbarra - 10


{62 AB == 31 →→ BA == 3/1/26 →
1
6
3
y P = ( x3 + x 2 ) e−2 x
2
(observar que yP no contiene constantes arbitrarias)

Paso 4. Encontramos la solución general y mediante las condiciones iniciales, determinamos la


solución del PVI.
1 3
Solución General ED: y = c 1 e−2x + c 2 x e−2 x + ( x 3 + x2) e −2 x → y (0) = c 1 = 2
6 2

1 x3 3
yi = −4 e −2 x + c2 e−2 x − 2 c 2 x e−2 x + ( x 2 + 3 x) e−2 x − 2 e−2 x ( + x 2 )
2 6 2

y I (0 ) = −4 + c 2 = 5 → c 2 = 9

1 3
Solución PVI: y = 2 e−2 x + 9 x e−2 x + ( x 3 + x 2 ) e−2 x
6 2

3.3 Método de Variación de Parámetros


En aquellas ED no Homogéneas, donde g(x) está compuesta por funciones que no sean Polinomios,
Exponenciales, Seno, Coseno y sumas algebraicas y/o productos de éstas, el método de los
Coeficientes Indeterminados no es apropiado para encontrar una Solución Particular; en estos casos
dicha solución se puede hallar mediante el Método de Variación de Parámetros ( este método también
permite encontrar una solución particular cuando g(x) está formada por las tres funciones elementales
nombradas).

d2 y dy
Sea la ED de 2do orden: a2 2
+ a1 + a 0 y = g (x) , dividiendo ambos miembros por el
dx dx

coeficiente a2, se obtiene la denominada forma estándar de la ED:

d2 y dy
2
+ a + b y = f (x ) (10)
dx dx

En primer lugar se resuelve la EDH Asociada y se encuentran las soluciones: y 1 e y2.

Luego, se propone como Solución Particular: y P = u1 ( x) y1 ( x) + u2 ( x) y 2 ( x ) ; siendo u1 y u2

funciones a determinar.

MATEMATICA APLICADA - MATEMATICA III – Cursada 2020/2021. MCIbarra - 11


yIP = u1I y 1 + u 1 y I1 + u 2I y 2 + u2 y I2
→ reemplazando en (10)
y IIP = u1II y1 + u1I y 1I + u1I y1I + u1 y 1II + u II2 y2 + u I2 y2I + u I2 y2I + u 2 y II2

u1II y 1 + u1I y 1I + u1I y 1I + u1 y 1II + u II2 y 2 + u I2 y 2I + u I2 y 2I + u 2 y II2 +



+ a [u I1 y 1 + u 1 y 1I + u I2 y 2 + u 2 y 2I ] + b [u1 y 1 + u2 y 2 ] = f ( x )

u1 [ y II1 + a y 1I + by 1 ] + u 2 [ y2II + a y 2I + b y 2 ] + u II1 y 1 + u1I y I1 + u1I y I1 +

+ u II2 y 2 + u I2 y2I + u I2 y2I + a [u I1 y 1 + u I2 y 2 ] = f (x )

teniendo en cuenta que las expresiones entre corchetes, ubicadas en los dos primeros términos son nulas y
haciendo algunos arreglos algebraicos, resulta:
u1II y 1 + u1I y 1I + u1I y 1I + u2II y2 + u I2 y 2I + u I2 y2I + a [u I1 y 1 + u I2 y2 ] =.

.= a [ u1I y1 + u2I
d
y 2 ]+ (u1I y 1 + u2I y 2) + u1I y I1 + u2I y I2 = f ( x )

{ u1I y 1 + u I2 y 2 = 0
u1I y I1 + u I2 y I2 = f (x )
(11)

dx

De este sistema de ecuaciones encontramos uI1 y uI2; de manera que integrando podemos hallar
las funciones u1 y u2 y así determinar una solución particular de la ED.
Para resolver (11) se pueden utilizar distintas estrategias; entre las cuales se encuentra la Regla de
Cramer, que resulta especialmente sencilla en este caso:

| | | | | |
y1 y2 0 y2 y1 0 W1 W2
W= W1 = W2 = → u1I = ; u I2 = (12)
yI1 y I2 f ( x) y2I y I1 f ( x) W W

Una vez encontradas ambas funciones, es posible plantear la Solución General: y = y H + yP


y = c1 y1 + c 2 y2 + u 1 y 1 + u 2 y2

ex
Ejemplo 4: Resolver la ED: y II − 2 y I + y =
1 + x2

Paso 1. Resolución EDH Asociada


m² – 2 m + 1 = (m-1)2 = 0 → y1 = e x ; y 2 = x ex

Paso 2. Propuesta Solución Particular


y P = u1 e x + u 2 x ex → utilizando (12) hallamos uI1 y uI2

MATEMATICA APLICADA - MATEMATICA III – Cursada 2020/2021. MCIbarra - 12


| | | |
0 x ex ex 0
ex
W= x
e | x ex
e x ( x +1) |
= e2 x ; W 1 = ex
1+ x 2
e x ( x +1)
=−
x e2 x
1+ x 2
; W2 = x
e
ex =
1+ x2
e2 x
1+ x 2

x 1 1
u1I =− 2
→ u1 = − ln (1+ x2 ) ; u I2 = → u 2 = arctan x
1+x 2 1+x 2
ex
yP = − ln (1+ x 2 ) + x e x tan−1 x
2

Paso 3. Solución General

ex
y = c1 ex + c2 x e x − ln (1+ x2) + x e x tan−1 x
2

4. Modelo Matemático de un Circuito RLC

Una de las aplicaciones de las EDL de 2do orden consiste en la


modelización matemática de un circuto serie RLC ( siendo constantes
los tres parámetros).
De acuerdo a la Ley de Kirchhoff: E(t) = V L + V R + V C

di 1 dq
VL = L ; V R = R i (t ) ; V C = q (t ) ; i(t ) =
dt C dt

d2 q dq 1
E(t) = L 2
+R + q(t ) MODELO MATEMATICO CIRCUITO SERIE RLC (13)
dt dt C

Esta EDL de 2do orden, con coeficientes constantes, tiene como variable dependiente la carga del
capacitor, la cual varía en el tiempo según la función q(t), que es la solución de la ED. Se convierte
en un PVI con la incorporación de dos condiciones iniciales, que generalmente consisten en la
carga del capacitor y la intensidad de corriente, en el instante t = 0.
1
L q II (t ) + R q I (t ) + q (t) = E(t) q(0) = q 0 ; i(0) = i0 → PVI (solución única)
C

MATEMATICA APLICADA - MATEMATICA III – Cursada 2020/2021. MCIbarra - 13


En los casos en que la ED representa un fenómeno/proceso físico, a la función g(x) de la expresión
(1) se le denomina Función de Entrada, Alimentación o Excitación del Sistema; mientras que a la
solución de la ED se designa como Función de Salida o Respuesta del Sistema. La Función de
Entrada y las Condiciones Iniciales/Frontera determinan la Salida.
En el caso del circuito RLC, la fem de alimentación E(t) es la Entrada del Sistema y la función q(t)
corresponde a la Respuesta/Salida.

Ejemplo 5: Un circuito serie con R = 2 Ω; L = 1 hy; C = 0,25 F; es alimentado con V(t) = 50 cost V;
siendo nulas la carga del capacitor y la corriente inicial; encontrar q(t).
q II (t) + 2 q I (t ) + 4 q(t ) = 50 cos t q (0) = 0 ; i(0) = 0.

Tenemos una EDL No Homogénea y un par de condiciones iniciales, que conforman un PVI. Por la
forma que tiene la función de entrada, es posible encontrar una solución particular mediante el
método de Coeficientes Indeterminados.

Raíces del Polinomio Característico de la Homogénea Asociada: m =−1 ± i √3


Soluciones de EDH: q 1 = e−t cos( √ 3 t) q 2 = e −t sen ( √3 t )

De acuerdo a la forma de la Función de Entrada de la ED, la Solución particular ,q P, , debe estar


conformada por una suma algebraica de sent y cost; las cuales son linealmente independientes de
las funciones que aparecen como solución de la Homogénea Asociada, entonces se propone:
q P = A cos t + B sen t

q IP =− A sen t + B cos t q IIP = −A cos t − B sen t

−A cos t − B sen t + 2 (− A sen t + B cos t ) + 4 ( A cost +B sent) = 50 cost

{3 3AB+−2 B2 A==500 → A=
150
; B=
100
13 13

150 100
Solución General: q (t ) = c 1 e −t cos ( √3 t ) + c 2 e−t sen (√ 3 t ) + cos t + sen t
13 13
150 250
Aplicando las Condiciones Iniciales, se obtiene: c1 = − ; c2 =−
13 13 √ 3

MATEMATICA APLICADA - MATEMATICA III – Cursada 2020/2021. MCIbarra - 14


150 −t 250 −t 150 100
Solución PVI: q (t ) =− e cos ( √3 t ) − e sen( √ 3 t ) + cos t + sen t
13 13 √ 3 13 13

FIGURA 2 – Carga del Capacitor FIGURA 3 – Intensidad de Corriente

La función solución del PVI representa la variación de Carga del Capacitor en función del tiempo,
q(t); mientras que su primer derivada es la Intensidad de Corriente en el circuito. En este caso,
ambas magnitudes tienen forma senoidal y constan de dos componentes:
- Componente Transitoria: constituyen los términos de la función que contienen e -t, expresión que
tiende a cero para valores relativamente grandes de la variable independiente.
- Componente Permanente/Estable: constituyen los términos de la función que contienen
solamente senos y cosenos.

MATEMATICA APLICADA - MATEMATICA III – Cursada 2020/2021. MCIbarra - 15


TRANSFORMADA DE LAPLACE

1. Presentación. Definición.

La Transformada de Laplace es un Operador Integral, que se aplica a una función f(t), en el


intervalo [0, ∞), mediante la siguiente expresión:

L[ f (t)] = ∫ e−st f (t ) dt DEFINICION TRANSFORMADA DE LAPLACE
0

La expresión e-st se denomina núcleo de la transformación e incorpora una variable ficticia s, que a
los efectos de la integral se toma como una constante, pero se convierte en la variable de la
función transformada, es la variable de la Transformada de Laplace (TL): L[ f (t)] = F (s)

Para que exista la TL , es necesario que la integral impropia converja, para lo cual, la función f(t)
debe cumplir dos condiciones:

1. Continua o seccionalmente continua en el intervalo [0, ∞); equivale a decir que para t ≥ 0, la
función no debe presentar discontinuidades infinitas; este tipo de funciones se llama también,
continua a tramos.

2. Orden Exponencial, lo que significa que para valores suficientemente grandes de la variable
independiente (t > T) la función f crece más lentamente que la exponencial Mect, donde M y c son
constantes positivas. Si f(t) es de Orden Exponencial, su gráfica debe encontrarse por debajo de la
curva Mect para valores suficientemente grandes de t; analíticamente se puede corroborar esta

f (t )
condición si se verifica: lim = 0 donde M y c se seleccionan convenientemente . Las
t →∞ M e ct

funciones polinómicas, las exponenciales de base natural y exponente lineal, senos y cosenos, son
algunos ejemplos de funciones de orden exponencial. Para comprobarlo analíticamente,
planteamos los límites correspondientes:
t 2 +t 2 t +1 2 e 4t 1
lim t
= lim t
= lim t = 0 lim 5t
= lim t = 0
t →∞ e t →∞ e t →∞ e t →∞ e t →∞ e

en el primer caso: M = c = 1 y en el segundo caso: M = 1 , c = 5

MATEMATICA APLICADA - MATEMATICA III – Cursada 2020/2021. MCIbarra - 1


FIGURA 1 – Seccionalmente Continua FIGURA 2 - Orden Exponencial
Fuente: Ecuaciones Diferenciales – 8va. Ed. – Autores: D. Zill ; W. Wright

f (t ) = 4 t es un ejemplo de una función que no es de orden exponencial y donde no es posible

4t 4 t
hallar la TL: lim t
= lim ( ) = ∞
t →∞ e t →∞ e

Existen algunas funciones que no son seccionalmente continuas ni de orden exponencial y aún así
presentan TL; de manera que estas dos condiciones son suficientes pero no necesarias para la
existencia de TL.

2. Transformada por definición de Funciones Elementales.


Aplicando la definición vamos a encontrar las TL de algunas funciones básicas o elementales, como
polinomios, exponenciales, senos y cosenos; y así obtendremos una Tabla de Transformadas.
Estas funciones básicas son las que aparecen con más frecuencia en la resolución de EDL y como
nuestro objetivo es emplear las TL para resolver ED/PVI/PVF, nos interesan específicamente.
Hallar la TL por definición, implica resolver la integral impropia, que en la mayoría de los casos, se
realiza mediante la técnica de integración Por Partes.

2.1 Función Constante f (t ) = k


∞ ∞
k k ∞
L[k ] = ∫ k e−st dt ; u = −st ; du = −s dt → − ∫ e u du = − e (−st ) |0 =.
0
s 0
s

k k k
.= − [e−∞−1] = → L [k ] =
s s s

MATEMATICA APLICADA - MATEMATICA III – Cursada 2020/2021. MCIbarra - 2


2.2 Función Identidad f (t ) = t

e−st
L[t ] = ∫ t e−st dt ; u = t ; dv = e−st dt → du = dt ; v = − s
0

| |
∞ ∞ ∞ ∞
e−st e−st e−st 1 −st ∞
∫t e −st
dt = −t
s 0
+∫
s
dt = −t
s 0

s2
e |0
0 0

El primer término del lado derecho de la última ecuación es nulo, por la condición de Orden
Exponencial que cumple f(t) = t, según se comprueba mediante la Regla de L’Hopital:
t 1
lim (t e−st ) = lim = lim = 0 ; así resulta:
t →∞ t →∞ e st t →∞ s est

e−st ∞
∫ t e−st dt = − 2
|0 = − 12 [e−∞−1] = 12 → L [t ] = 12
0 s s s s

2.3 Función Cuadrática f (t ) = t 2



e−st
L[t 2 ] = ∫ t 2 e−st dt ; u = t 2 ; dv = e−st dt → du = 2t dt ; v =−
s
0

t 2 −st e −st t2 2
∫ t 2 e−st dt = −
s
e + ∫ 2t
s
dt = − e −st +
s s
∫ t e−st dt

| |
∞ ∞ ∞ ∞
t2 2 t2 2
∫t 2
e −st
dt = − e −st
s
+ ∫ t e −st dt = − e−st
s 0 s
+
s
L [t ]
0 0 0

El primer término del lado derecho de la última igualdad es nulo, por condición de Orden
Exponencial de la función f(t) = t2:
t2 2t 2
lim st
= lim st
= lim 2 st = 0 , resulta así:
t →∞ e t →∞ s e t→∞ s e

2 2 1 2 2
∫ t 2 e−st dt =
s
L [t ] = ( 2 ) = 3 → L [t 2 ] = 3
s s
0 s s

2.4 Función Cúbica f (t ) = t 3



e−st
L[t 3 ] = ∫ t 3 e−st dt ; u = t 3 ; dv = e−st dt → du = 3 t 2 dt ; v = − s
0

e−st 3 3
∫ t 3 e−st dt = − s
t +
s
∫ t 2 e−st dt

MATEMATICA APLICADA - MATEMATICA III – Cursada 2020/2021. MCIbarra - 3


|
∞ ∞ ∞
−e −st 3 3 3 3 2 6 6
∫t 3 −st
e dt =
s
t +
s
∫ t 2 e−st dt =
s
L[t 2 ] = ( 2 ) = 3 → L[t 3 ] = 3
s s
0 0 0 s s

Las TL de potencias naturales de la variable independiente f (t ) = t n tienen una expresión

n!
recurrente, que se puede generalizar como: L[t n ] =
sn+1

2.5 Función Seno f (t ) = sen(k t ) kϵR


e −st
L[ sen(kt )] = ∫ sen (k t) e−st dt ; u = sen(kt ) ; du = k cos (k t )dt ; dv = e −st dt ; v = −
s
0

sen(k t) −st k
∫ sen(k t) e−st dt =−
s
e + ∫ e −st cos (k t) dt
s

e−st
u = cos (k t ) ; du = −k sen (k t) dt ; dv = e−st dt ; v =−
s

est k
∫ e−st cos (k t) dt = −
s
cos(k t ) − ∫ sen (k t) e−st dt
s

sen (kt ) −st k −e st k


∫ sen (k t) e−st dt = −
s
e + [
s s
cos(kt ) − ∫ sen (kt ) e−st dt ] =.
s

sen (kt ) −st k k2


∫ sen (kt ) e−st dt = −
s
e − 2 e−st cos (kt ) − 2 ∫ sen(k t ) e−st dt
s s

k2 sen (k t) −st k
2 ∫
[1+ ] sen(k t ) e−st dt = − e − 2 e−st cos(kt )
s s s

|
∞ ∞
k2 −sen (kt ) −st k
[1+ 2 ] ∫ sen(kt ) e−st dt = e − 2 e−st cos(kt )
s 0 s s 0

|

k2 −sen (k t) −st k −st ∞
[1+ 2
] L[ sen(k t )] = e − 2
e cos(k t ) |0
s s 0 s

Aquí nuevamente debemos tener en cuenta que las funciones sen(kt) y cos(kt) son de Orden
Exponencial, de manera que el primer término del lado derecho de la última ecuación, resulta nulo
y entonces:

MATEMATICA APLICADA - MATEMATICA III – Cursada 2020/2021. MCIbarra - 4


k
2
k k k s2 k k
[1+ 2 ] L[ sen(kt )] =− 2 [0−1] = 2 → L[ sen(kt)] = 2 2
= 2 2
→ L [sen (kt )] = 2
s s s s +k s +k s + k2
s2

Mediante un procedimiento similar al que se acaba de realizar, es posible obtener la TL de la

s
función f(t) = cos(kt), resultando: L[cos (kt )] =
s +k 2
2

2.6 Función Exponencial Natural f (t ) = e k t kϵR


∞ ∞
L[e k t ] = ∫ e k t e−st dt = ∫ e−t (s−k) dt → u = −t (s−k) ; du = −(s−k)dt
0 0

−1 −1 −t (s−k )
∫ e−t (s−k ) dt =
s−k
∫ e u du =
s−k
e

−1 −t (s −k ) |∞ −1 −∞ 1 1
L[ e k t ] = e 0 = [ e −1] = → L[e k t ] =
s−k s−k s−k s−k

3. Transformada por Tablas


Con los resultados anteriores, podemos elaborar una Tabla de TL para las funciones
elementales/básicas:

f(t) F(s) f(t) F(s)


k k/s (k ϵ R) sen(kt) k/(s2 + k2)

t 1/s2 cos(kt) s/(s2 + k2)

t2 2/s3 ekt 1/(s-k)

tn n!/sn+1 (n ϵ N)

TABLA 1 : TRANSFORMADA DE LAPLACE PARA FUNCIONES ELEMENTALES

Es importante destacar que las TL de estas funciones básicas, son funciones racionales;
observamos así que este Operador, convierte tanto las funciones algebraicas como las
trascendentes, en racionales (siendo el grado del numerador inferior al del denominador) .

MATEMATICA APLICADA - MATEMATICA III – Cursada 2020/2021. MCIbarra - 5


La TL es un Operador Lineal, es decir que la TL de una combinación lineal de funciones, equivale a
la combinación lineal de las TL de cada una de estas funciones; sean A y B constantes reales:

∞ ∞ ∞
L[ A f (t )+ B g(t)] = ∫ [ A f (t )+B g (t )] e−st dt = A∫ f (t ) e−st dt + B∫ f (t ) e−st dt
0 0 0

L[ A f (t )+ B g(t)] = A L[ f (t)] + B L[ g(t )] → L[ A f (t )+B g(t)] = A F(s) + B G(s)

Aplicando la propiedad de linealidad de la TL y utilizando la Tabla de transformadas, podemos


determinar la TL de las funciones básicas.

Ejemplo 1: Hallar las TL (por tabla) de las siguientes funciones:

1
a) f (t ) = −4 sen(2 t ) + b cos (−t ) + e −4 t
5
1 −4t −8 bs 1
F(s) = L[−4 sen (2 t )] + L[b cos (−t)] + L [ e ] = 2 + 2 +
5 s +4 s +1 5 (s+4 )

b) h(t ) = t (t 3 − 3 )2 + π
7 ! 6. 4 ! 9 π
h(t ) = t (t 6 − 6 t 3 + 9) + π = t 7 − 6 t 4 + 9 t + π → H (s) = − 5 + 2+
s8 s s s

4. Transformada Inversa
Hasta aquí hemos visto como hallar F(s), conociendo f(t); pero también es necesario que podamos
realizar el proceso inverso, es decir, dada F(s) determinar la función f(t) a la cual corresponde esa
TL. Este proceso inverso se denomina Transformación Inversa o Antitransformada y se designa con
el símbolo L-1.

L[ f (t)] = F (s) → L−1 [F (s)] = f (t )

4.1 Transformada Inversa Inmediata


Si la función F(s) tiene una forma algebraica coincidente con alguna de las funciones elementales
que figuran en la Tabla 1, para hallar su Transformada Inversa, simplemente observamos en la
tabla a qué función corresponde la F(s) que tenemos. También puede ser necesario “acomodar” la

MATEMATICA APLICADA - MATEMATICA III – Cursada 2020/2021. MCIbarra - 6


expresión de manera que coincida con alguna de las funciones elementales, en tal caso lo que se
debe hacer es agregarle algunas constantes, como veremos en los ejemplos siguientes.

Ejemplo 2: Determinar a qué funciones corresponden las siguientes TL:

−3 16 1 5
a) G (s) = 7
+ 5 − 3+ 2 → observamos que todos los términos de G(s) se corresponden
s s s s
con la TL de potencias naturales de la variable independiente y para hallar g(t) debemos trabajar
con las constantes para conseguir en el numerador el factorial correspondiente:
−3 6 ! 16 4 ! 1 2! 5 −3 6 ! 16 4 ! 1 2! 1
G(s) = 7
( )+ 5 ( )− 3 ( )+ 2 = [ 7] + [ 5]− [ 3] +5[ 2]
s 6! s 4! s 2! s 6! s 4! s 2! s s
3 6 16 4 1 2
L−1 [G(s)] = g(t ) = − t + t − t +5 t
6! 4! 2!

3s−8
b) H (s) = → observamos que el denominador tiene la estructura de la transformada
s2 + 25
del seno/coseno, pero el numerador no coincide exactamente con ninguna de estas funciones.
3s−8 3s 8 5 s 8 5
H (s) = 2
= 2 − 2 ( ) = 3[ 2 ]− [ 2 ]
s + 25 s + 25 s + 25 5 s + 25 5 s + 25
8
L−1 [ H (s)] = h (t) = 3 cos (5 t) − sen (5 t)
5

4.2 Descomposición en Fracciones Simples


Cuando la estructura algebraica de F(s) no coincide directamente con alguna de las funciones
elementales, es necesario recurrir a la descomposición en Fracciones Simples de la función
transformada, lo cual siempre es posible, dado que son funciones racionales.
Para este proceso recordar lo visto en Cálculo 1, respecto a los tres casos posibles; raíces reales simples,
reales múltiples y complejas.

2s −1
Ejemplo 3: Hallar f(t) siendo: F(s) =
(s − s2) (s2 + 3)
4

MATEMATICA APLICADA - MATEMATICA III – Cursada 2020/2021. MCIbarra - 7


2s − 1 2s − 1 A B C D E s+ G
4 2 2
= 2 2
= + 2+ + + 2
(s − s ) (s + 3) s (s − 1)(s+1) (s + 3) s s s−1 s+1 s +3

F (s) =
A B
+ 2 +
C
+
D s
+ E [ 2 ]+
G √3
[ 2 ]
s s s−1 s+1 s +3 √3 s + 3
G
L−1 [ F(s)] = f (t ) = A + B t + C et + D e−t + E cos( √ 3 t) + sen( √ 3 t)
√3
donde A, B, C, D, E, G ϵ R

5. Transformadas de Derivadas
En este Curso estamos interesados en utilizar la TL para la resolución de EDL con coeficientes
constantes; de manera que necesitaremos encontrar las TL de las sucesivas derivadas de una
función. Hallaremos la TL por definición de las primeras derivadas y observaremos que la
expresión de las TL tienen una estructura algebraica recurrente, entonces podremos generalizar la
TL a una derivada de enésimo orden.
Si f(t) es una función seccionalmente continua y de orden exponencial, sus derivadas también
verifican ambas condiciones.

5.1 TL de Derivada 1er Orden



I
L[ f (t)] = ∫ f I (t) e−st dt → u = e−st ; du = −s e−st dt ; dv = f I (t ) dt ; v = f (t )
0

∞ ∞

∫f I −st
(t ) e dt = e −st
f (t ) | + s ∫ f (t)e −st dt = −f (0) + s L[ f (t )] = s F(s)−f (0 )
0
0 0

L[ f I (t)] = s F (s)−f (0)

donde f(0) es Condición Inicial.

5.2 TL de Derivada 2do Orden



L[ f II (t )] = ∫ f II (t ) e−st dt → u = e −st ; du = −s e−st dt ; dv = f II (t) dt ; v = f I (t )
0

∞ ∞

∫ f II (t ) e−st dt = e−st f I (t ) |0 + s∫ f I (t) e−st dt = −f I (0) + s L[ f I (t)]
0 0

L[ f II (t )] = s [ s F (s) − f (0)] − f I (0) = s 2 F (s) − s f (0) − f I (0)

MATEMATICA APLICADA - MATEMATICA III – Cursada 2020/2021. MCIbarra - 8


L[ f II (t )] = s2 F (s) − s f (0) − f I (0)

donde f(0) y fI(0) son Condiciones Iniciales

5.3 TL de Derivada 3er Orden



L[ f III (t )] = ∫ f III (t) e−st dt → u = e −st ; du = −s e−st dt ; dv = f III (t) dt ; v = f II (t)
0

∞ ∞

∫ f III (t) e−st dt = e−st f II (t ) |0 + s∫ f II (t )e−st dt = −f II (0) + s L[ f II (t )]
0 0

L[ f III (t )] = s [ s2 F (s) − s f (0) − f I (0)] − f II (0) = s3 F (s) − s2 f (0) − s f I (0) − f II (0)

L[ f III (t )] = s3 F(s) − s 2 f (0) − s f I (0) − f II (0)

5.4 TL de Derivada n-sima


Observando las expresiones anteriores, podemos extraer ciertos patrones recurrentes:
• La TL de la derivada de orden n, es un polinomio de grado n, en la variable s.
• Solamente el término de mayor grado del polinomio es positivo y está multiplicado por
la TL de la función original.
• Todos los demás términos son negativos y contienen las Condiciones Iniciales.

La expresión general para la TL de una derivada de enésimo orden es:


(n) (n−2) (n−1)
L[ f (t )] = s n F (s) − sn−1 f (0 ) − s n−2 f I (0 ) − s n−3 f II (0) − ............. − s f (0) − f (0)

Para encontrar la TL de una derivada de n-simo orden, se necesitan n condiciones iniciales, todas evaluadas
en t = 0.

Podemos observar que la aplicación de TL para resolver una ED requiere de Condiciones Iniciales,
es decir lo que se resuelve es un PVI y como tal, tiene única solución. A diferencia de los métodos
anteriormente vistos para resolver PVI, donde era necesario encontrar la solución de la EDH
Asociada, luego la solución particular y luego aplicar las Condiciones Iniciales; mediante las TL
todos estos pasos se realizan simultáneamente y la función obtenida es directamente la solución
del PVI. Esta es una de las ventajas de la TL, dado que no requiere resolver la EDH Asociada ni
tampoco proponer una solución particular, pero como contraparte, requiere de estrategias
algebraicas en ocasiones, un tanto extensas.

MATEMATICA APLICADA - MATEMATICA III – Cursada 2020/2021. MCIbarra - 9


La resolución de un PVI mediante TL puede resumirse en dos pasos:
1. Aplicar TL a la ED y encontrar F(s), que corresponde a la TL de la función solución f(t). En esta
instancia se incorporan las Condiciones Iniciales. La función F(s) es racional y así transforma una
Ecuación Diferencial en variable t, en una Ecuación Algebraica en variable s.
2. Aplicar Transformación Inversa a F(s) para hallar f(t).
3. Para corroborar el resultado, se verifican las Condiciones Iniciales.

Ejemplo 4:
d3 y d2 y dy
Resolver el PVI: 3
+ 2 3
− − 2 y = sen(3 t ) y (0) = 0 ; y I (0) = 0 ; y II (0) = 1
dt dt dt

3
s3 Y (s)−s2 y (0)−s y I (0)− y II (0) + 2[ s2 Y ( s)−s y (0)−y I (0)] − [ sY (s)− y (0)] − 2 Y (s) = 2
s +9

3
s3 Y (s) −1 + 2 s2 Y (s) − s Y (s) − 2 Y (s) = 2
s +9

3 s2 +12
Y ( s) [s3 + 2 s2 − s − 2] = + 1 =
s2 +9 s2+9

s2 + 12
Y ( s) =
(s2 +9) (s−1) (s+1) (s+2)

A B C Ds + E s2 + 12
Y (s) = + + + 2 = 2
s−1 s+1 s+2 s +9 ( s + 9) ( s−1) (s+1) ( s+2)

13 −13 16 3 −3
A= B= C= D= E=
60 20 39 130 65

La función racional Y(s) representa la TL de la solución y(t): L[y(t)] = Y(s); para encontrar y(t) se
aplica Transformada Inversa; se observa que las tres primeras fracciones corresponden a
exponenciales y la última fracción a seno/coseno.

E
L−1 [Y (s)] = y (t ) = A e t + B e−t + C e−2t + D cos(3 t ) + sen (3 t )
3
13 t 13 −t 16 −2t 3 1
y (t) = e − e + e + cos(3 t ) − sen (3 t )
60 20 39 130 65

MATEMATICA APLICADA - MATEMATICA III – Cursada 2020/2021. MCIbarra - 10


La función y(t) es la solución del PVI, incluye la componente homogénea, la no homogénea y
también las condiciones iniciales (observar que no contiene parámetros arbitrarios ).

Como último paso, se verifican las condiciones iniciales:


13 13 16 3
y (0) = − + + = 0
60 20 39 130

13 t 13 −t 32 −2t 9 3
y I (t ) = e + e − e − sen(3 t) − cos (3 t )
60 20 39 130 65

13 13 32 3
y I (0 ) = + − − = 0
60 20 39 65

13 t 13 −t 64 −2t 27 9
y II (t) = e − e + e − cos (3 t ) + sen(3 t)
60 20 39 130 65

13 13 64 27
y II (0) = − + − = 1
60 20 39 130

6. Primer Teorema del desplazamiento


En la resolución de ED es frecuente la presencia de productos entre exponenciales y polinomios,
senos o cosenos; por ejemplo: cos (b t ) ea t ; sen(bt ) e−at ; t 3 eat ;en estos casos, encontrar la TL
por definición generalmente implica extensos desarrollos algebraicos. El Primer Teorema del
desplazamiento permite encontrar la TL de este tipo de funciones de una manera sencilla.
Este teorema establece que para encontrar la TL del producto de una función f(t) por una
exponencial ea t ,, debemos encontrar la TL de f(t) y desplazar en a unidades la variable s:

L[e a t f (t )] = F(s−a) siendo L[ f (t)] = F (s)

FIGURA 3 - Desplazamiento en variable s

Fuente: Ecuaciones Diferenciales – 8va. Ed. – Autores: D. Zill ; W. Wright

MATEMATICA APLICADA - MATEMATICA III – Cursada 2020/2021. MCIbarra - 11


Demostración
∞ ∞
L[e a t f (t )] = ∫ eat f (t) e−s t dt = ∫ f (t ) e−t (s−a) dt = F (s−a) donde a ϵ R
0 0

Este teorema permite hallar la TL del producto de una función f(t) por una exponencial, en dos
pasos: primero encontrar la TL de la función f ( sin tener en cuenta la exponencial) y luego desplazar
todas las s de F(s) tantas unidades como corresponda al exponente de la exponencial. Así, cuando
una TL tiene la variable desplazada, ya sabremos que proviene del producto de dos funciones, una
de las cuales, es una exponencial natural.

Para aplicar la Transformación Inversa, tenemos la expresión:

L−1 [ F(s−a)] = f (t ) e a t siendo L−1 [ F(s)] = f (t ) ; en este caso primero se encuentra la

Transformada Inversa de F(s) (sin tener en cuenta el desplazamiento ) y luego se multiplica por la
exponencial. Para realizar este proceso inverso, todas las s que aparecen en la TL deben tener el
mismo desplazamiento; de no ser así, habrá que trabajar algebraicamente la expresión hasta
conseguirlo y recién después efectuar la antitransformada.

Ejemplo 5: Hallar f(t) ó F(s), según corresponda.

t
a) f (t ) = (e −t + e 3 t ) 2 cos ( )
2
t t s
f (t ) = (e −2 t + 2 e2 t + e6 t ) cos ( ) → L[cos ( )] =
2 2 1
s2 +
4
s+2 s−2 s−6
L[ f (t)] = + 2 +
1 1 1
(s+2)2 + (s−2)2 + (s−6)2 +
4 4 4

(s+1)2
b) F(s) =
(s+2)4

[(s+ 2)−1 ] 2 (s+2) 2 − 2(s+2) + 1 1 2 1


F(s) = 4
= 4
= 2
− 3
+ ; una vez que
(s+2) (s+2) ( s+ 2) (s+2) (s+2) 4

conseguimos el mismo desplazamiento en todas las s , como cálculo auxiliar, suponemos que no existen los
desplazamientos, entonces tendríamos:

MATEMATICA APLICADA - MATEMATICA III – Cursada 2020/2021. MCIbarra - 12


1 2 1 1 3 t3
G(s) = 2
− 3+ 4 → g (t) = t − t 2 + t → L−1 [ F( s)] = e −2 t [t − t 2 + ]
s s s 3! 6

7. Segundo Teorema del desplazamiento


Con frecuencia es necesario aplicar TL a una función por tramos, en tal caso es posible aplicar la
definición y plantear una integral por cada tramo; por ejemplo:

{
f 1 (t) 0≤t≤a a b ∞
f (t ) = f 2 (t) a <t≤b → L[ f (t )] = ∫ f 1 (t ) e−st
dt + ∫ f 2 (t ) e−s t dt + ∫ f 3 (t) e−s t dt
0 a b
f 3 (t ) t >b

Resolver por definición la TL de una función con varios tramos, puede convertirse en un proceso
extenso; pero existe una manera más sencilla de transformar este tipo de funciones , mediante la
aplicación directa del Segundo Teorema del desplazamiento; para cuyo desarrollo se necesita una
función especial, que se presenta a continuación.

7.1 Función Escalón Unitario


La Función Escalón Unitario, también conocida como Función de Heaviside, es una función por

tramos muy sencilla, definida de la siguiente manera: h(t ) = { 10 t <0


t≥0
, en este caso el

escalón se ubica en t = 0. Con frecuencia el escalón está desplazado en t = a, en tal caso la

expresión por tramos es: h(t−a) = { 10 t <a


t≥a

FIGURA 4 -Escalón Unitario en t = 0 FIGURA 5 -Escalón Unitario en t = a

MATEMATICA APLICADA - MATEMATICA III – Cursada 2020/2021. MCIbarra - 13


Cuando la Función Escalón cambia su imagen de cero a uno, decimos que se activa; así puede
activarse en cualquier instante t ≥ 0, según como esté definida. También con frecuencia, la imagen
no es la unidad, sino cualquier otra constante.
La particularidad que presenta la Función Escalón Unitario, es que al multiplicar a cualquier otra
función f(t), la anula para valores de t inferiores al instante de activación, y a partir del instante que
se activa, no produce cambio alguno en f(t). A continuación analizaremos gráficamente el efecto de
la función Escalón Unitario sobre una función f(t).
En la Figura 6 se representa la función f(t), que inicia en t = 0. Al multiplicarla por h(t-a) resulta la

función por tramos: f (t ) h(t−a) =


{ f (t0 ) t<a
t≥a
, que se representa en la Figura 7. El efecto del

Escalón Unitario al multiplicar la función, es anularla para t < a y activarla a partir del instante
t =a , posteriormente se mantiene intacta, dado que se multiplica por la unidad.

FIGURA 6 - f(t) FIGURA 7 - f(t). h(t – a)

Siendo b > a, al multiplicar f(t) por el Escalón Unitario h(t–b), se obtiene una función, nula para
t < b y coincidente con f(t) a partir del instante t = b, su expresión es:

f (t ) h(t−b ) =
{ f (t0 ) t<b
t≥b
; si a esta función se multiplica por (-1) se consigue su opuesta:

−f (t ) h(t−b ) =
{ −f0 (t ) t<b
t≥b
; ambas se representan en la Figura 8.

MATEMATICA APLICADA - MATEMATICA III – Cursada 2020/2021. MCIbarra - 14


Si a la función f (t ) h(t−a) se le suma −f (t ) h(t−b ) , la función resultante se anula a partir

del instante t = b (ver Figura 9), podemos decir también que se desactiva a partir de ese instante.
La expresión algebraica es: f (t ) h (t −a) − f (t ) h(t−b ) = f (t ) [h(t−a) − h (t −b)]

De esta manera se consigue un tramo de la función original: f (t ) [h(t−a) − h(t−b )] , que se

activa en t = a y se desactiva en t = b; indicado en la Figura 10.

FIGURA 8 - Funciones Opuestas FIGURA 9

FIGURA 10 - f(t) [h(t-a) – h(t-b)]

Entonces, mediante la aplicación del Escalón Unitario, es posible generar funciones por tramos,
multiplicando la función original por la diferencia entre el escalón en el instante en que se activa y

MATEMATICA APLICADA - MATEMATICA III – Cursada 2020/2021. MCIbarra - 15


el escalón en el instante que se desactiva; y además de esta manera se dispone de otra forma
algebraica de expresar funciones por tramos.

{
f 1 (t) 0≤t≤a
Por ejemplo, la función: f (t ) = f 2 (t) a <t≤b se puede expresar como:
f 3 (t ) t >b

f (t ) = f 1 (t) [h (t ) − h (t −a)] + f 2 (t) [h (t −a) − h (t −b)] + f 3 (t ) h(t−b)

También podemos partir de una función por tramos, dada en términos de la Función Escalón:
g (t) = g 1 (t ) + g 2 (t ) h(t−a) + g3 (t ) h(t−b) donde 0 < a < b

{
g 1 (t) 0≤ t < a
y expresarla de la siguiente forma: g (t) = g1 (t ) + g 2 (t ) a≤t <b
g 1 (t ) + g2 (t ) + g 3 (t) t ≥b

7.2 Enunciado del Segundo Teorema del desplazamiento


El Primer Teorema plantea un desplazamiento en la variable s, mientras que el Segundo Teorema
establece un desplazamiento de la variable t y se aplica para encontrar la TL de funciones por
tramos; este Teorema establece:

L[ f (t−a ) h(t−a)] = F (s) e−a s siendo L[ f (t)] = F (s)

Según esta expresión, la TL de una función que se activa en t = a, se obtiene multiplicando la TL de


la función original por la exponencial e −as ; siendo a > 0.

Demostración
∞ a ∞
L[ f (t−a )h(t−a)] = ∫ f (t−a ) h(t−a) e−s t dt = ∫ f (t−a) h (t −a) e−st dt + ∫ f (t−a) h (t −a) e−s t dt
0 0 a


L [f (t−a)h (t −a)] = ∫ f (t−a) e−st dt → u = t −a ; du = dt
a

∞ ∞ ∞
∫ f (t−a ) e−s t dt = ∫ f (u) e−s (u+a) du = e−a s ∫ f (u) e−s u du = e−a s F(s)
a 0 0

MATEMATICA APLICADA - MATEMATICA III – Cursada 2020/2021. MCIbarra - 16


Para aplicar este teorema, los desplazamientos de la variable independiente en la función y en el
escalón, deben ser iguales; si esto no sucede, es necesario agregar las constantes apropiadas y
recién una vez que se haya conseguido los desplazamientos correctos, aplicar la TL. También hay
que tener en cuenta que este trabajo algebraico ( para acomodar los desplazamientos) se debe
realizar en la función f(t), no en el escalón.
Otra cuestión importante a observar, es que al aplicar este teorema, la función transformada ya no
es racional, sino que aparece un término exponencial en la variable s; de manera que cuando se
tiene una TL en la cual figura una expresión del tipo e −as , se reconoce que proviene de una

función por tramos, con escalón en t = a.

Ejemplo 6: Hallar la TL de la siguiente función, mediante el 2do Teorema del desplazamiento.

f (t ) =
{ t2
6−t
t≤2
2<t ≤6

En primer lugar, expresamos f(t) en términos del Escalón Unitario:


f (t) = t 2 [h(t ) − h (t −2)] + (6−t ) [h (t −2) − h (t −6)]

f (t ) = t 2 − t 2 h (t −2) + 6 h(t −2)−6 h (t −6) − t h(t −2) + t h (t −6)

Agregamos las constantes necesarias para conseguir que en cada término, el desplazamiento de la
variable t coincida con el del escalón:
f (t ) = t 2 − (t − 2 + 2)2 h(t−2) + 6 h (t −2)−6 h (t −6) − (t − 2 + 2) h (t −2) + (t − 6 + 6) h(t−6)

f (t ) = t 2 − [(t −2) + 2]2 h (t −2) + 6 h(t−2) − 6 h(t−6) − (t−2) h(t−2) − 2 h(t−2) +.

.+ (t −6) h(t−6) + 6 h(t−6)

f (t ) = t 2 − (t−2)2 h (t −2) − 5 (t−2) h(t−2) + (t−6) h (t −6)

Una vez conseguidos los desplazamientos correctos, aplicamos el Teorema para hallar la TL:
2 2 5 1
F(s) = 3
− 3 e −2 s − 2 e−2s + 2 e−6 s
s s s s

Observar que los coeficientes de los exponentes coinciden con los instantes donde se producen los cambios
de tramos en la función.

MATEMATICA APLICADA - MATEMATICA III – Cursada 2020/2021. MCIbarra - 17


7.3 Transformación Inversa
Para aplicar la Transformada Inversa de acuerdo Segundo Teorema, se utiliza:

L−1 [e−a s F (s)] = f (t − a ) h(t − a)

En primer lugar se debe reconocer la función f(t), por observación de F(s) y luego desplazar la
variable t y escalonar la función, multiplicándola por el Escalón Unitario.

2 2 5 1
Ejemplo 7: Hallar f(t), siendo: F(s) = 3
− 3 e−2 s − 2 e−2s + 2 e −6 s
s s s s

Observando la estructura algebraica de F(s) se puede adelantar que proviene de una función por
tramos, con puntos de corte en t = 2 y t = 6. El primer término corresponde a la transformada de la
función elemental g(t) = t2 ; en los demás términos donde aparece la exponencial, podemos
realizar cálculos auxiliares, suponiendo que no estuviesen las exponenciales y así encontrar las
funciones base; luego desplazar las t y escalonar:
2
L−1 [ ] = t 2 → (t −2)2 h(t−2)
s3

5
L−1 [ ] = 5t → 5 (t −2) h(t−2)
s2

1
L−1 [ ] = t → (t−6) h (t −6)
s2

L−1 [ F( s)] = t 2 − (t −2)2 h(t−2) − 5 (t−2) h(t−2) + (t−6) h (t −6)

A los efectos de graficar una función por tramos, es conveniente expresarla de manera que se
expliciten los dominios de cada tramo y sus expresiones analíticas. Para esto es necesario observar
en que instante inician y finalizan cada uno de los tramos. En este ejemplo, tenemos 4 términos, el
primero de ellos inicia en t = 0, el segundo y el tercero en t = 2 y el último tramo en t = 6; cada
tramo se va agregando al anterior y así podemos armar la función:

{ {
t2 0≤t <2 t2 0≤t <2
f (t ) = t −(t−2)2 −5(t−2)
2
2≤t <6 → f (t ) = −t +6 2≤t< 6
2 2
t −(t−2) −5(t−2)+(t−6) t≥6 0 t≥6

MATEMATICA APLICADA - MATEMATICA III – Cursada 2020/2021. MCIbarra - 18


FIGURA 11

7.4 Resolución de ED con Funciones por Tramos


Sea la expresión general de una EDL de enésimo orden con coeficientes constantes:
(n) (n−1) (n−2)
an y + an−1 y + an−2 y + ............. + a 1 y I + a 0 y = f (t)

Si la entrada, f(t), es una función por tramos, la salida y(t), también será una función por tramos.
En estos casos, para resolver la ED mediante TL, aplicaremos el Segundo Teorema del
desplazamiento.

d 2 y dy
Ejemplo 8: Resolver el PVI: 2
+ = f (t ) ; y (0) = 0 ; y I (0 ) = 0 ; donde f(t) se
dt dt

representa en el gráfico:

FIGURA 12

En primer lugar, expresamos f(t) en términos del Escalón Unitario y hallamos la TL:
t
f (t ) = 1 [ h(t ) − h (t −2)] + (2− ) [h (t −2) − h (t −4)] ; luego de algunos pasos algebraicos:
2

MATEMATICA APLICADA - MATEMATICA III – Cursada 2020/2021. MCIbarra - 19


1 1 1 1 1
f (t ) = 1 − (t −2) h(t−2) + (t −4 ) h(t−4 ) → F(s) = − 2 e−2s + 2 e−4 s
2 2 s 2s 2s

Aplicamos TL a la ED completa:
1 1 1
s2 Y (s) − s y (0) − y I (0) + s Y (s) − y (0) = − 2 e −2 s + 2 e−4 s
s 2s 2s

1 1 1
Y (s) (s2 +s) = − 2 e−2 s + 2 e− 4 s
s 2s 2s

1 e− 2s e−4s
Y (s) = − +
s3 +s2 2 (s4 + s3 ) 2 (s4 +s3 )

Para hallar y(t) es necesario aplicar Transformación Inversa y previamente descomposición en


Fracciones Simples; para lo cual solamente se tiene en cuenta la componente racional de cada
término de Y(s) y a tales efectos, no se involucran las exponenciales. Las componentes racionales
del segundo y tercer términos de Y(s) son idénticas, por lo tanto le corresponderán las mismas
Fracciones Parciales.

1 1 1 1 1 1
3 2
= 2 = 2− + → L−1 [ 3 2
] = t − 1 + e−t
s +s s (s+1) s s s+1 s +s

1 1 1 1 1 1 1 t2
= 3 = 3− 2+ − → L−1 [ 4 3
] = − t + 1 − e−t
4
s +s 3
s (s+1) s s s s+1 s +s 2 !

En estos términos se debe desplazar la variable t y escalonar, por aplicación del Segundo Teorema,
(equivale a tomar en cuenta el efecto de las exponenciales ) y resulta así:

e−2 s (t −2)2
L−1 [ 4 3
] = [ − (t−2) + 1 − e−(t−2) ] h(t−2)
s +s 2 !

e −4 s (t −4)2
L−1 [ 4 3
] = [ − (t −4 ) + 1 − e −(t−4) ] h (t −4 )
s +s 2 !

1 1 −(t −2) 1 1 −(t −4)


y (t) = (t − 1 + e −t ) − h(t−2) [ (t−2)2 − (t−2) + 1 − e ] + h(t −4) [ (t−4)2 − (t−4) + 1−e ]
2 2 2 2

MATEMATICA APLICADA - MATEMATICA III – Cursada 2020/2021. MCIbarra - 20


Escribimos y(t) de manera que se expliciten las expresiones de cada tramo y sus dominios:

{
t − 1 + e −t 0≤t <2
1 1 −( t−2)
y (t) = t − 1 + e −t − 2 [ 2 (t−2)2 − (t−2) + 1−e ] 2 ≤t < 4
−t 1 1 2 −(t−2) 1 1 2 −(t −4)
t−1+ e − 2 [ 2 (t−2) −(t−2) + 1−e ]+ 2 [ 2 (t −4) −(t−4 ) +1−e ] t≥4

{
1 − e −t 0≤t <2
I
y (t) = 1 − e−t − 12 [(t−2) − 1 + e −(t−2) ] 2 ≤t < 4
1 − e−t − 12 [(t−2)−1 + e−( t−2)] + 12 [ (t− 4) − 1 + e−(t −4) ] t≥4

Verificamos las Condiciones Iniciales, como ambas se dan en t = 0, corresponden al primer tramo:
y (0) = 0 − 1 + 1 = 0 ; y I (0) = 1 − 1 = 0

Para completar la verificación del resultado, se realiza la segunda derivada de la función y(t):

{
e−t 0 ≤t < 2
1 −(t −2)
y II (t) = e −t − 2 [1− e ] 2≤t <4
−(t−2)
−t 1
e − 2 [1− e ]+ 1
2 [ 1− e −( t−4) ] t≥4

Al reemplazar en la ED se debe verificar: y II + y I = f (t ) . Realizaremos esta comprobación para


cada uno de los tres tramos.

Primer tramo: 0 ≤ t < 2


[e−t ] + [1 − e−t ] = 1

Segundo tramo: 2 ≤ t ≤ 4
−(t−2) −(t −2)
1 e t−2 1 e t
[e−t − + ] + [1 − e−t − + − ] = 2−
2 2 2 2 2 2

Tercer tramo: t > 4


−(t−2) −(t −4) −(t −2) −(t −4)
1 e 1 e t 1 e t 1 e
[e−t − + + − ] + [ 1 − e−t − +1+ − + −2− + ]=0
2 2 2 2 2 2 2 2 2 2

Finalmente, resulta: f (t ) =
{
−1
2
1
t +2
0 ≤t < 2
2≤t≤4

MATEMATICA APLICADA - MATEMATICA III – Cursada 2020/2021. MCIbarra - 21


A continuación se presentan las gráficas de la función solución y(t) y sus dos primeras derivadas.

Según se comprueba analíticamente, las tres funciones son continuas en los instantes t = 2 y t = 4;
pero la segunda derivada presenta esquinas en dichos puntos, como también lo hace la función de
entrada f(t); este comportamiento no es casualidad, sino que ocurre en todos los casos. La
derivada de mayor orden de la ED tiene el mismo comportamiento que la función de entrada, en
este caso, ambas son continuas y presentan esquinas en los instantes de cambio de tramos.

MATEMATICA APLICADA - MATEMATICA III – Cursada 2020/2021. MCIbarra - 22


FUNCION DELTA DE DIRAC

1. Presentación. Definición.

La Función Delta de Dirac o Función Impulso, pertenece a un grupo especial de funciones,


denominadas FUNCIONES GENERALIZADAS y a diferencia de las Funciones Clásicas (como las que
estudiamos en Cálculo 1), su comportamiento y propiedades no se ajustan estrictamente a las
mismas reglas algebraicas/analíticas y cronológicamente son de aparición mucho más reciente, por
ejemplo la Delta de Dirac surge en 1930, en el contexto de una investigación en el área de la Física,
en el intento de encontrar un Modelo Matemático para fenómenos de muy corta duración y que
involucren fuerzas de gran intensidad (por ejemplo una descarga eléctrica, un impulso de tensión
en un circuito, un golpe fuerte a una masa, etc).
Para llegar a la definición de la Delta de Dirac, partimos de una función por tramos, definida de la

{
0 0 ≤ t < t0 − a
siguiente manera: δ a (t−t 0 ) = 1/(2 a) t 0 − a ≤ t ≤ t 0 + a ; esta función tiene la forma de
0 t > t0 + a

un escalón de base 2a y altura recíproca 1/(2a); de manera que el área del rectángulo es la unidad,
por este motivo se conoce como Impulso Unitario. A medida que disminuye a, se reduce la base
del rectángulo y aumenta la altura, generando los sucesivos rectángulos indicados en la Figura 13.

FIGURA 13 – Impulso Unitario FIGURA 14– Delta de Dirac

MATEMATICA APLICADA - MATEMATICA III – Cursada 2020/2021. MCIbarra - 23


Cuando a se aproxima a cero , la base del rectángulo también y su altura tiende a infinito; así surge
la Delta de Dirac, como el límite del Impulso Unitario cuando a → 0:

δ (t −t 0 ) = lim δ a (t−t 0 )
a→0
→ δ (t −t 0 ) =
{

0 t ≠ t0
t = t0
DEFINICION DELTA DE DIRAC

Así, la Delta de Dirac, puede ser expresada como una función por tramos, nula para todo valor de t,
excepto para t0 donde presenta imagen infinita; este es el instante donde aplica la fuerza/acción
externa al sistema; entonces, teóricamente, es un impulso de duración infinitesimal (instantáneo) y
magnitud infinita. Se representa gráficamente con una flecha vertical, en el instante de aplicación,
como indica la Figura 14.
La Delta de Dirac también se concibe como la derivada de la Función Escalón Unitario:
d
[h(t−t 0)] = δ (t−t 0 )
dt

En la teoría de las funciones clásicas, el Escalón Unitario no tendría derivada en t = t 0, porque no es


continua en ese instante, sin embargo en la concepción de las Funciones Generalizadas, sí es
posible derivar una función en un salto y su resultado es otra Función Generalizada, la Delta de
Dirac.

2. Transformada de Laplace
Hemos visto que existen dos condiciones que aseguran la existencia de la TL de una función:
Seccionalmente Continua y de Orden Exponencial, en el intervalo t ≥ 0. La Delta de Dirac no
cumple ninguna de estas condiciones y sin embargo tiene TL; estas condiciones son suficientes
pero no necesarias.
Para encontrar la TL de la Delta de Dirac no es posible aplicar la definición, ya que la integral
impropia planteada sería divergente. Para encontrar esta TL, planteamos en primer lugar, la TL de
la Función Impulso Unitario mediante el Segundo Teorema del desplazamiento y entonces es
necesario expresarla en términos del Escalón Unitario:

1
δ a (t−t 0 ) = [h [t − (t 0−a)] − h [t − (t 0 +a )]]
2a

MATEMATICA APLICADA - MATEMATICA III – Cursada 2020/2021. MCIbarra - 24


1 −s (t 0 −a) −s (t 0 +a)
L[ δ a (t−t 0 )] = [e −e ]
2as
δ (t −t 0 ) = lim δ a (t−t 0 ) → L[ δ (t −t 0 )] = lim L[ δ a (t−t 0 )]
a→0 a→0

−s t
1 −s t −s t e 0 sa
L[ δ (t −t 0 )] = lim [e 0 e s a − e 0 e −s a ] = lim [e − e− sa ] =.
a→ 0 2 a s a→0 2 a s

−s t0 −s t − st
e e sa − e−s a e 0 e 0 −st
.= lim [ ] = lim [ s e s a + s e−s a ] = 2s = e 0
2s a→0 a 2 s a →0 2s

−s t 0
L[ δ (t −t 0 )] = e

La TL de la Delta de Dirac es una exponencial donde el coeficiente del exponente coincide con el
instante de aplicación del Impulso (observar que siempre será negativo el exponente). Si el instante
t0 = 0 → L[ δ (t )] = 1

La Delta de Dirac se constituye así en un Modelo Matemático para representar fuerzas/acciones de


gran intensidad que se aplican durante intervalos muy reducidos de tiempo. En este Curso
veremos su aplicación en dos modelos físicos:
• Circuitos RLC alimentados por impulsos instantáneos de tensión
• Análisis de esfuerzos en Vigas (Momento Flector, Corte, Deflexión) bajo la acción de cargas
puntuales.

Ejemplo 9: Resolver el PVI: y II + y I − 2 y = 2 δ (t −1) + δ (t −3) ; y (0) = 0 ; y I (0) = 0


En este caso, la fuerza o acción externa consiste en dos impulsos de magnitudes 2 y 1, aplicados en
los instantes 1 y 3, respectivamente.

s2 Y (s) − s y(0) − y I (0 ) + s Y (s) − y (0) − 2 Y (s) = 2 e−s + e −3 s

1
Y (s)(s 2 + s − 2) = 2 e−s + e−3s → Y ( s) = [2 e−s + e−3s ]
(s−1) (s+2)

1/ 3 1 /3
Y (s) = [2 e−s + e −3s ] [ − ]
s−1 s+2

MATEMATICA APLICADA - MATEMATICA III – Cursada 2020/2021. MCIbarra - 25


Para hallar y(t) debemos aplicar transformación inversa, en este caso tenemos la presencia de los
dos términos exponenciales, indicando que corresponden al Segundo Teorema del
desplazamiento. Según hemos visto, la antitransformada de este Teorema realiza dos acciones
sobre la función: desplaza la variable t y la multiplica por el escalón. El término e−s representa

un escalón en t = 1 y el término e−3s otro escalón en t = 3; de manera que tenemos una función

por tramos.
2 t −1 −2(t−1) 1 −2(t −3)
y (t) = [e − e ] h (t−1) + [e t−3 − e ] h (t −3)
3 3

{
0 0 ≤t < 1
2 t −1 −2(t−1)
y (t) = 3 [e − e ] 1 ≤ t < 3
−2(t−1)
2
3
t− 1
[e − e ]+ 1
3 [et−3 − e− 2( t−3) ] t ≥ 3

{
0 0 ≤t < 1
I
y (t) =
2
3 [ et −1 + 2 e−2( t−1)] 1 ≤ t < 3
2
3 [et−1 + 2 e− 2(t−1) ] + 13 [ e t−3 + 2 e−2( t−3)] t ≥ 3

Con estas funciones podemos comprobar las Condiciones Iniciales. Respecto de la continuidad en
los instantes t = 1 y t = 3, se verifica que y(t) es continua, pero no lo es yI(t):

lim yI (t ) = 0 ; lim y I (t ) = 2
x→1− x→1+

2 2 2 2
lim y I (t) = [e + 2 e−4 ] ; lim y I (t ) = [e + 2 e −4 ] + 1
x →3− 3 x →3 + 3

Se observa que en ambos instantes la derivada primera presenta discontinuidades de Salto, las
magnitudes de estos saltos en los instantes 1 y 3, son de 2 unidades y 1 unidad, respectivamente;
estas constantes corresponden a los coeficientes que multiplican a las Funciones Delta de Dirac,
que constituyen la entrada del sistema. Entonces, en estos instantes, la derivada primera presenta
dos escalones y en la derivada segunda, aparecen ambas Delta de Dirac, porque recordemos que
esta Función Generalizada, se concibe como la derivada de la Función Escalón.

{
0 0 ≤t < 1
2 δ (t −1) t = 1
2 t −1 −2( t−1)
y II (t) = 3 [e −4 e ] 1 < t < 3
δ (t −3) t = 3
2 t−1 − 2(t−1) 1 t−3 −2( t−3)
3
[e − 4 e ] + 3 [e − 4 e ] t > 3

MATEMATICA APLICADA - MATEMATICA III – Cursada 2020/2021. MCIbarra - 26


MATEMATICA APLICADA - MATEMATICA III – Cursada 2020/2021. MCIbarra - 27
Tenemos dos instantes críticos: t = 1 y t = 3; en los cuales cada una de las funciones tiene un
comportamiento diferente:

• La función solución y(t) es continua, pero presenta esquinas.


• La derivada primera presenta discontinuidades de salto, con magnitud 2 en t = 1 y
magnitud 1 en t = 3.
• La derivada segunda presenta dos Impulsos ó Funciones Delta de Dirac, como resultado de
la derivada de ambos escalones. Observamos que la derivada de mayor orden de la ED
tiene el mismo comportamiento ó la misma estructura, que la función de entrada.

MATEMATICA APLICADA - MATEMATICA III – Cursada 2020/2021. MCIbarra - 28


Universidad Nacional de MisionesFUNC

FISICO MATEMATICA APLICADA 1

Aplicación de Transformadas de Laplace a la resolución de EDO


para el análisis de esfuerzos en Vigas

Consideremos una viga homogénea y de área transversal constante, si la misma no está


sometida a ninguna carga exterior, el eje de simetría coincidirá con el eje x (eje neutro); una
vez cargada la viga, esta se deformará y su eje de simetría formará una curva llamada
“elástica”, a la cual designaremos como y(x):

FIGURA 1

Así los esfuerzos a que se verá sometida la viga podrán ser analizados a través del
Momento Flector M(x) y el esfuerzo de corte Q(x); si llamamos q(x) a la carga exterior que
soporta la viga, las relaciones que vinculan estas magnitudes están dadas por las siguientes
Ecuaciones Diferenciales Ordinarias (EDO):

d2y dM d 2M d4y
M ( x)  EI Q( x )  q ( x)   EI
dx 2 dx dx 2 dx 4

Siendo E e I constantes que representan el Módulo de Elasticidad del material y el Momento


de Inercia de la sección transversal de la viga, respectivamente.
En esta guía vamos a utilizar la Transformada de Laplace para resolver EDO, para distintas
condiciones de apoyos en los extremos de la viga (simplemente apoyada, empotrada, en
voladizo, etc.); las cuales constituyen las Condiciones de Frontera (CF) y tenemos así un
Problema con Valores en la Frontera (PVF); si suponemos una viga de longitud L,

Físico Matemática Aplicada 1 – Aplicación de TL a la resolución de EDO para el análisis de esfuerzos en vigas –
María del C. Ibarra Página 1
Universidad Nacional de MisionesFUNC

designaremos como x = 0 y x = L sus extremos. Las CF indicarán valores de deflexión,


Momento Flector y Corte en dichos extremos.

Caso 1: Carga uniformemente distribuida aplicada en viga simplemente apoyada.


En una viga simplemente apoyada las CF vienen establecidas como:
- Deflexión nula en ambos extremos: y(0) = y(L) = 0
- Momento Flector nulo en ambos extremos yII(0) = yII(L) = 0
d4y
La EDO que debemos resolver, sometida a esas CF, será: q  EI
dx 4
Tenemos así una EDO de 4° orden cuyo término independiente es la carga aplicada y la
solución buscada es la ecuación de la elástica y(x).
A continuación se resuelve el PVF mediante Transformada de Laplace:

d4y
EI  q
dx 4
1 q
s 4 L( y )  s 3 y (0)  s 2 y I (0)  sy II (0)  y III (0) 
s EI
Aplicando las CF resulta:
1 q
s 4 L( y)  s 2 y I (0)  y III (0) 
s EI
los valores de la primera y tercera derivada en x = 0 no se conocen, pero en cambio se sabe
que y(L) e yII(L) son nulas; para hacer uso de estas dos CF y así resolver el problema, será
necesario un paso previo. Llamando yI(0) = A; yIII(0) = B; resulta:
1 q
s 4 L( y )   As 2  B
s EI
A B 1 q
L( y )  2  4  5
s s s EI
x3 q x4
y ( x)  Ax  B 
6 EI 24
siendo y(x) una solución para la curva elástica que depende de los parámetros A y B;

L3 q L4
y ( L)  AL  B  0
6 EI 24
q x2 q L2
y II ( x)  Bx   y II ( L)  BL  0
EI 2 EI 2
Resolviendo las dos ecuaciones anteriores:
q L3 qL
A B
24 E I 2E I

Físico Matemática Aplicada 1 – Aplicación de TL a la resolución de EDO para el análisis de esfuerzos en vigas –
María del C. Ibarra Página 2
Universidad Nacional de MisionesFUNC

Resultan así las expresiones de la elástica, el momento y el esfuerzo de corte:

q  x4 L3 x  q q
y ( x)    Lx 3  M ( x)  x ( x  L) Q( x )  ( 2 x  L)
12 EI  2 2  2 2

FIGURA 2

Caso 2: Carga puntual aplicada en viga simplemente apoyada.


Analizaremos ahora el caso de una carga puntual “P” aplicada en el centro de una viga de
longitud L, como único esfuerzo externo. Este tipo de carga puede representarse mediante
una Delta de Dirac aplicada puntualmente en x = L/2, siendo nula para todo el resto de la
viga; las CF serán iguales que en el caso anterior, deflexión y momento flector nulos en
ambos extremos; resulta así la EDO:

d4y  L
EI  P  x  
dx 4
 2
4
d y P  L
  x  
dx 4 EI  2
P  sL / 2
s 4 L( y )  s 2 y I (0)  y III (0)  e
EI
haciendo A = yI(0); B = yIII(0)
P  sL / 2
s 4 L( y )  As 2  B  e
EI
A B P
L( y )  2
 4
 4
e  sL / 2
s s s EI

Físico Matemática Aplicada 1 – Aplicación de TL a la resolución de EDO para el análisis de esfuerzos en vigas –
María del C. Ibarra Página 3
Universidad Nacional de MisionesFUNC

Al hacer la Transformada Inversa el último término dará origen a una función escalonada en
x = L/2 que representa la deflexión de la viga:
x 3 P ( x  L / 2)3
y( x)  Ax  B  h( x  L / 2)
6 EI 6
al expresarla como una función por tramos resulta:
 x3
 Ax  B 0 x L / 2
 6
y ( x)  
 x3 P
 Ax  B  ( x  L / 2) 3 L / 2 x L
 6 6 EI
Para hallar los valores de las constantes A y B es necesario trabajar con las CF: y(L) = 0;
yII(L) = 0; resultando así:
PL2 P
A B 
16 EI 2 EI
Al reemplazar estos valores y mediante algunos pasos algebraicos, llegamos a las
expresiones de deflexión, momento y esfuerzo de corte:
 PL2 P
 x x3 0 x L / 2
16 EI 12EI
y ( x)  
2
 PL P P
16 EI x  12EI x  6 EI ( x  L / 2) L / 2 x L
3 3


 P
 x 0 x L / 2
 2  P / 2 0 x L / 2
M ( x)   Q( x)  
 P x  P ( x  L / 2) P / 2 L / 2 x L
L / 2 x L
 2

Es importante verificar la continuidad de y(x) y M(x) en x = L/2; así como también la


presencia de un vértice en la gráfica de M(x), lo cual dará origen a la discontinuidad de salto
para la función Q(x).

FIGURA 3

Físico Matemática Aplicada 1 – Aplicación de TL a la resolución de EDO para el análisis de esfuerzos en vigas –
María del C. Ibarra Página 4
Universidad Nacional de MisionesFUNC

Caso 3: Carga uniformemente distribuida aplicada en viga empotrada en un extremo


En los dos ejemplos anteriores hemos trabajado con vigas simplemente apoyadas, en este
caso se presenta una viga empotrada en un extremo (x = L). Las CF serán: y(L) = yI(L) = 0;
M(0) = Q(0) = 0. La EDO y su resolución queda así:
q 1 q
y IV ( x)  s 4 L( y )  s 3 y (0)  s 2 y I (0) 
EI s EI
1 q
s 4 L( y )  As 3  Bs 2 
s EI
1 q B A q x4
L( y )  5 EI
 2  y ( x)  Bx A
s s s EI 24

qL4 qL3
Aplicando las CF resulta: A  B 
8EI 6 EI
Y finalmente se obtienen las expresiones de deflexión, momento y esfuerzo de corte,
respectivamente:

q  L4 L3 x 4  x2
y ( x)    x M ( x)  q Q( x)  qx
2 EI  4 3 12  2

FIGURA 4

Caso 4: Carga uniformemente distribuida aplicada en un tramo en viga simplemente


apoyada
En este caso se trabajará con una viga simplemente apoyada en ambos extremos y
sometida a una carga uniforme aplicada en el tramo comprendido desde uno de los
extremos hasta la mitad de la viga (0 ≤ x ≤ L/2). Las CF será en este caso y(0) = y(L) = 0;
yII(0) = yII(L) = 0. La solicitación en este caso se expresa como una función por tramos:
q(x) = q[h(x) – h(x-L/2)]

Físico Matemática Aplicada 1 – Aplicación de TL a la resolución de EDO para el análisis de esfuerzos en vigas –
María del C. Ibarra Página 5
Universidad Nacional de MisionesFUNC

suponiendo que L = 1; quedan entonces la EDO y su resolución así:

EI y IV ( x)  q h( x)  h( x  1 / 2)
q q s / 2
s 4 L( y )  s 3 y (0)  s 2 y I (0)  s y II (0)  y III (0)   e
EI s EI s
q q s / 2
s 4 L( y )  s 2 y I (0)  y III (0)   e
EI s EI s
q q s / 2
s 4 L( y )  As 2  B   e
EI s EI s
q q s / 2
s 4 L( y )  As 2  B   e
EI s EI s
A B q 1 q 1 s / 2
L( y )  2
 4
 5
 e
s s EI s EI s 5
4
B q q  1  1
y ( x)  Ax  x 3  x4  x  h x  
6 24 EI 24 EI  2  2

Se ha obtenido así la expresión de la elástica con los parámetros A y B, cuyos valores


deben ser hallados utilizando las CF; previamente se expresará y(x) como una función por
tramos y de la misma manera se encontrarán sus derivadas sucesivas:

 B 3 q 1
 Ax  6
x 
24 EI
x4 0 x 
2

y ( x)   4
 Ax  B 3 q q  1 1
 x  x4  x   x 1
 6 24 EI 24 EI  2 2

 B 2 q 3 1
A  x  x 0 x 
 2 6 EI 2
y ( x)  
I
3
A  B 2 q 3 q  1 1
x  x  x   x 1
 2 6 EI 6 EI  2 2

 q 1
 Bx  2 EI x 0 x 
2

 2
y II ( x)   2
 Bx  q x 2  q  x  1  1
 x 1
  
 2 EI 2 EI  2 2

3 q 3 q
A B  
128 EI 8 EI

Físico Matemática Aplicada 1 – Aplicación de TL a la resolución de EDO para el análisis de esfuerzos en vigas –
María del C. Ibarra Página 6
Universidad Nacional de MisionesFUNC

Resulta así que las expresiones del Momento Flector, el Esfuerzo de Corte y la Elástica
también son funciones a tramos, cada una de las cuales consta de dos tramos, siendo x = ½
el punto donde finaliza el primer tramo un e inicia el segundo; en los tres casos las funciones
son continuas en dicho punto.

q 2 3 1  3 1
 x  qx 0 x  qx  q 0 x
2 8 2  8 2
M ( x)   Q( x)  
 1 qx  1 q 1
 x 1 1 q 1
 x 1

8 8 2 
8 2

FIGURA 5

Físico Matemática Aplicada 1 – Aplicación de TL a la resolución de EDO para el análisis de esfuerzos en vigas –
María del C. Ibarra Página 7
Universidad Nacional de MisionesFUNC

EJERCICIOS PROPUESTOS

a) Mediante Transformadas de Laplace, hallar las expresiones de la Elástica,


Momento Flector y Esfuerzo de Corte, para las vigas sometidas a las solicitaciones
indicadas en los diagramas.

b) Representar gráficamente el Momento y el Esfuerzo de Corte y comprobar las CF.


c) Determinar dónde se producen los valores máximos de dichas magnitudes y
cuáles son estos valores.

Referencias Bibliográficas
 Zill, Dennis & Cullen, Michael; Ecuaciones Diferenciales con problemas con valores
en la frontera. Cengage Learning; 2009.
 Zill, Dennis; Ecuaciones Diferenciales con aplicaciones de modelado. Thomson;
2007
 Veas B., Verónica & Jing Chang Lou; Deformación en vigas; Universidad de Chile;
2000.
 Bañón, Luis; Estructuras Metálicas; 2009.

Físico Matemática Aplicada 1 – Aplicación de TL a la resolución de EDO para el análisis de esfuerzos en vigas –
María del C. Ibarra Página 8
ECUACIONES DIFERENCIALES PARCIALES

1. Presentación. Método de Separación de Variables.

Las Ecuaciones Diferenciales Parciales (EDP) son aquellas que presentan dos o más variables
independientes y al igual que las Ecuaciones Diferenciales Ordinarias (EDO) se clasifican en
Lineales y No Lineales; en este Curso abordaremos las EDP Lineales, donde la variable dependiente
y sus derivadas, están elevadas a la primera potencia.
Sea u(x,y) la función solución de una EDP Lineal de Segundo Orden, cuya expresión general es:

δ 2u δ 2u δ 2u δu δu
A + B + C +D +E + Fu = G
δx 2 δx δy δy 2 δx δy

donde A, B, C, D, E, F y G son constantes o funciones que dependen de x e y. Si G(x, y) = 0, se tiene


una EDP Homogénea.

En caso que sea posible expresar la función solución como el producto de otras dos funciones, una
de las cuales dependa exclusivamente de x y la otra exclusivamente de y, se tiene:
u( x , y) = X (x ).Y ( y ) y se puede resolver la EDP mediante Separación de Variables,

transformándola en dos EDO.

δ2u δu
Ejemplo 1: Resolver la EDP: k −u = ; k >0
δx 2 δt

δ 2u d2u δu du
u( x , t) = X (x ).T (t ) → = = X II ( x). T (t) = X II .T ; = = X ( x ).T I (t) = X .T I
δx 2
dx 2 δ t dt

ahora es posible reescribir las derivadas parciales en términos de derivadas ordinarias:


k X II TI +T TI
k X II .T −X .T = X .T I → k X II .T = X .T I + X . T = X (T I + T ) → = = +1
X T T

k X II TI
La última igualdad de la expresión anterior, resulta : = + 1 ; donde el lado izquierdo
X T
depende exclusivamente de x , mientras que el lado derecho, exclusivamente de t; de manera que

MATEMATICA APLICADA - MATEMATICA III – Cursada 2020/2021. MCIbarra - 1


para que se verifique la igualdad, debe ser una constante, a la cual se designa con λ:
k X II TI
= + 1 = λ ; de esta surgen dos ecuaciones:
X T

k X II TI
= λ → k X II −λ X = 0 + 1 = λ → T I −T ( λ − 1) = 0
X T

Para resolver estas EDO se analizan las tres posibilidades: λ = 0 ; λ = α2 > 0 ; λ = - α2 < 0
Las ED de 1er y 2do orden que surgen así son Homogéneas y para hallar sus diferentes soluciones,
se recurre al análisis de las raíces del Polinomio Característico.

Caso 1: λ = 0
k X II
= 0 → X II = 0 ; m2 = 0 → X ( x) = c 1 + c2 x
X

TI
+ 1 = 0 → T I + T =0 ; m =−1 → T (t ) = c 3 e −t
T

u( x , t) = (c 1 + c 2 x ) c 3 e−t = a e −t + b x e−t (1)

Caso 2: λ = α2 > 0
α α x
k X II 2 x −
= α 2 → k X II −α 2 X = 0 ; m2 = α → m = ± α → X (x ) = c 1 e √ k + c2 e √ k
X k √k

TI 2
+ 1 = α 2 → T I − T (α 2−1) = 0 ; m = α 2 −1 → T (t ) = c 3 e( α −1) t
T

α x −α x α x −α x
2 2 2
u( x , t) = (c 1 e √ k + c 2 e √ k ) c 3 e( α −1) t
= a e √k e( α −1) t
+ b e √k e( α −1) t
(2)

Caso 3: λ = - α2 < 0
k X II 2 αi
= −α 2 → k X II + α 2 X = 0 ; m 2 = − α → m = ±
X k √k
X ( x) = c 1 cos( α x) + c2 sen( α x )
√k √k

MATEMATICA APLICADA - MATEMATICA III – Cursada 2020/2021. MCIbarra - 2


TI 2
+ 1 = −α 2 → T I + T (α 2 + 1) = 0 ; m = −α 2−1 → T (t) = c 3 e(−α −1) t
T

u( x , t) = [ c 1 cos ( α x ) + c 2 sen( α x)] c 3 e (−α −1) t = a cos ( α x ) e(−α −1) t + b sen( α x) e(−α −1) t
2 2 2
(3)
√k √k √k √k

Las funciones dadas por (1), (2) y (3) son las soluciones encontradas para esta ED y la combinación
lineal de las tres también será solución, por el Principio de Superposición de las Soluciones; de
manera que siendo u1, u2, ……., un un conjunto de soluciones, la combinación lineal de ellas, resulta

n
también una solución de la EDP: u( x, t) = ∑ ck uk .
k=1

Esta técnica de separación de variables, no siempre es posible aplicar, por ejemplo las dos EDP

δ 2u δ u δ 2u δ2u δ 2u
siguientes no son separables: − = y + + = 0
δ y2 δ x δ x2 δ x δ y δ y2

Ejercicio 1: Resolver las EDP dadas, mediante Separación de Variables, asignando a la constante λ
las tres posibilidades ( = 0; < 0 ; > 0). En cada caso, elegir una de las soluciones encontradas y
comprobar que verifica la EDP.
δ2u δu δ2u δ 2u
a) k = ; k>0 b) a 2 2 =
2
δx δt δx 2
δt

2. Modelos Matemáticos con EDP de Segundo Orden

Las EDP Lineales de segundo orden se encuentran como modelos matemáticos en varios sistemas
o procesos físicos; hay tres casos que se destacan:

δ2u δu
1. k = ; k>0 ECUACION DE CALOR UNIDIMENSIONAL ; la función solución u(x,t)
δx 2 δt

representa la temperatura en cada punto a lo largo de una varilla delgada; esta magnitud depende

MATEMATICA APLICADA - MATEMATICA III – Cursada 2020/2021. MCIbarra - 3


solamente de la posición x y del instante de tiempo t; siendo k una constante determinada por las
características del material.

δ2u δ 2u
2. a2 = ECUACION DE ONDA UNIDIMENSIONAL ; la función u(x,t) representa los
δ x2 δ t2

desplazamientos verticales de una cuerda vibratoria (por ejemplo, las cuerdas de una guitarra),
medida desde el eje x, en un instante de tiempo t. Estas vibraciones ocurren en el plano x-u y
producen desplazamientos de la cuerda perpendiculares al eje x.

Fuente: Ecuaciones Diferenciales con problemas con valores en la


frontera - 7ma. Edición
Autores: Dennis Zill & Michael Cullen

δ 2 u δ 2u
3. + = 0 ECUACION DE LAPLACE BIDIMENSIONAL ; en este caso ambas variables
δ x2 δ y 2
independientes son de posición, de manera que la EDP describe un proceso estacionario. Esta ED
está asociada a Campos Gradientes con divergencia nula, donde la función solución u(x,y)
representa por ejemplo, la distribución de temperaturas en una placa delgada (Ver Guía Nro. 3:
Campos Vectoriales).

Cuando a estas EDP se agregan condiciones o restricciones adicionales, se convierten en un


Problema con Valores Iniciales (PVI) o Problema con Valores en la Frontera (PVF). Las Condiciones
Iniciales son aquellas en que se especifican ciertas características del sistema en el instante t = 0
(por ejemplo la distribución inicial de temperaturas en la varilla o la posición inicial de la cuerda),
mientras que las Condiciones de Frontera pueden ser por ejemplo, la temperatura en cierto punto
de la varilla o de la placa. Estas restricciones varían de acuerdo al problema.

MATEMATICA APLICADA - MATEMATICA III – Cursada 2020/2021. MCIbarra - 4


3. Ecuación de Calor

En esta Sección se presenta el desarrollo y solución de la EDP de Segundo Orden, denominada


Ecuación de Calor Unidimensional.

Fuente: Ecuaciones Diferenciales con problemas con valores en la


frontera.

Sea una varilla delgada de Longitud L y área transversal A (L >> A), coincidente con el eje de
abscisas en 0 ≤ x ≤ L; suponiendo que se cumplen las siguientes condiciones:

• El material es homogéneo, siendo constantes: la densidad de masa (ρ), el calor específico


(γ) y la conductividad térmica (μ).
• El flujo de calor en el interior solo ocurre en la dirección x
• La superficie lateral está aislada (no hay intercambio de calor con el exterior)
• En el interior no existen fuentes ni sumideros de calor

Si u es la temperatura, la cantidad de calor en un elemento de masa m, es: Q = m γ u (4)

La rapidez con que el flujo térmico atraviesa una superficie de área A, viene dada por:
δQ δu
= −μ A (5)
δt δx

La masa del elemento comprendido entre x y (x + ∆x) es: m = ρ A Δx ; reemplazando en (4):


Q = ρ A Δx γ u (6)

Utilizando (5), se encuentra el flujo neto en el elemento de volumen:


−μ A u x (x , t ) − [−μ A u x (x + Δ x , t )] = μ A[u x( x + Δ x , t) − u x (x , t )] (7)

δQ δu
Derivando (6) respecto al tiempo, se obtiene: = ρ A Δx γ (8)
δt δt

MATEMATICA APLICADA - MATEMATICA III – Cursada 2020/2021. MCIbarra - 5


Entonces, es posible igualar las dos últimas expresiones:

δu μ [u x (x + Δ x , t ) − u x ( x , t)] δu
μ A [u x (x + Δ x , t ) − u x ( x , t )] = ρ A Δ x γ → ργ . =
δt Δx δt

μ [ u x (x + Δ x , t ) − u x (x , t )] δu μ δ2u δu
lim [ ρ γ . ] = lim → ρ γ . δ x2 = δ t
Δ x→ 0 Δx Δx→ 0 δ t

μ δ2u δu
designando como k a la constante , resulta: k =
ργ δx 2 δt

3.1 Solución de Ecuación de Calor - PVF

Se procede a resolver la Ecuación de Calor en una varilla de longitud L, cuyos extremos se


mantienen a una temperatura constante de 0 0 C y con una distribución inicial de temperatura,
dada por la función f(x); de manera que tenemos un PVF al adicionar estas restricciones a la EDP.
Así, es necesario encontrar una solución analítica al PVF dado por:
δ2u δu
k =
δx 2 δt

u(0 , t ) = 0 ; u( L, t ) = 0 ; t > 0

u( x , 0 ) = f (x ) ; 0 < x < L

Para resolver esta EDP se utiliza Separación de Variables, considerando que la temperatura se
puede expresar como el producto de dos funciones, que dependen cada una de una sola variable:
X II TI
u( x , t) = X (x ) T (t) → k X II T = X T I → = = λ
X kT

X II − λ X = 0 ; T I − λ k T = 0

Caso 1: λ=0
X II = 0 → m2 = 0 → X (x ) = c 1 + c 2 x

T I = 0 → m = 0 → T (t) = c 3

u( x , t) = a + b x (9)

MATEMATICA APLICADA - MATEMATICA III – Cursada 2020/2021. MCIbarra - 6


Caso 2: λ = α2
X II − α 2 X = 0 → m = ± α → X ( x) = c 1 e α x + c 2 e −α x
2
TI − α2 k T = 0 → m = α2k → T (t ) = c 3 eα kt

2 2
u( x, t) = a eα x e α kt
+ b e−α x eα kt
(10)

Caso 3: λ = - α2
X II + α 2 X = 0 → m =±α i → X (x ) = c 1 cos( α x ) + c 2 sen (α x )
2
TI + α2 k T = 0 → m = − α2k → T (t ) = c 3 e−α kt

2 2
u( x, t) = a cos(α x) e−α kt
+ b sen(α x ) e−α kt
(11)

Las funciones dadas por las expresiones (9), (10) y (11) son posibles soluciones al PVF, pero aún
faltan agregar las Condiciones Iniciales y de Frontera, dadas por las restricciones.

Reemplazando las temperaturas en los extremos x = 0 y x = L en (9):


u(0 , t ) = a = 0 → u (x , t ) = b x ; u( L , t) = b L = 0 → b = 0

resulta así, que la única solución sería la Trivial: u (x, t) = 0

Reemplazando ahora las temperaturas en los extremos en (10):


2 2 2 2
u(0 , t ) = a eα kt
+ b eα kt
= 0 → a +b = 0 → u (x , t ) = a e α x eα kt
− a e−α x eα kt

2 2
u (L , t ) = a e α L e α kt
− a e−α L eα kt
= 0
2
a eα kt
(e α L − e−α L) = 0 → a = 0

igual que en el caso anterior, la única posibilidad sería: u(x, t) = 0

Para la expresión (11), la aplicación de estas condiciones de frontera, resulta:


2 2
u(0 , t ) = a e−α k t
= 0 → a = 0 → u( x , t) = b sen (α x ) e−α kt

2
u( L , t) = b sen (α L) e−α kt
= 0 → b = 0 ó sen (α L) = 0

Así, la única alternativa para no llegar nuevamente a la solución Trivial, sería: sen(α L) = 0
y para esto es necesario que el argumento sea un múltiplo entero de π:

MATEMATICA APLICADA - MATEMATICA III – Cursada 2020/2021. MCIbarra - 7



sen (α L) = 0 → α L = nπ → α =
L

Resulta así la familia de soluciones que satisfacen las condiciones de frontera:


−n 2 π 2
kt
nπ L2
un (x , t ) = bn sen( x) e (12)
L

−n 2 π 2
∞ kt

Y por el Principio de Superposición: u( x, t) = ∑ b n sen( x) e L2
(13)
1 L



Aplicando la Condición Inicial: u( x , 0) = ∑ bn sen( x ) = f (x ) (14)
1 L

De (14) se deduce que f(x) corresponde a un desarrollo en Serie de Fourier en términos de la


función seno, con período L, de manera que los coeficientes b n se obtienen como:

L
bn =
2
L ∫ f ( x) sen( nLπ x ) dx (15)
0

Reemplazando (15) en (13) se llega finalmente a la solución del PVF:


L −n2 π 2
∞ kt
u( x , t) =
2
L
∑ [ ∫ f (x ) sen( nLπ x ) dx ] sen ( nLπ x) e L2
(16)
1 0

La representación gráfica de u(x, t) es una superficie en tres dimensiones, pero al asignar valores
constantes a una de las variables independientes, es posible encontrar las curvas de nivel y
representarlas en el plano. De esta manera, para valores constantes del tiempo, se determinan
funciones u(x), que representan la distribución de temperaturas en cada punto de la varilla, para
los diferentes instantes de tiempo.

MATEMATICA APLICADA - MATEMATICA III – Cursada 2020/2021. MCIbarra - 8


Ejemplo 2: Hallar la función temperatura u(x, t) en una barra delgada de longitud π, cuyos
extremos se mantienen a 00C y la temperatura en el instante inicial viene dada por:

f (x ) = { 01 0 < x < π /2
π /2 < x < π
; siendo k = 1.

El PVF a resolver es:


δ2u δu
=
δx 2 δt

u(0 , t ) = 0 ; u( π , t ) = 0 ; t > 0

u( x , 0 ) = f (x ) ; 0 < x < π

Para las condiciones dadas, la función solución dada por (16), resulta:
∞ π
2
∑ [ ∫ f ( x ) sen(nx) dx ] sen(n x) e−n
2
t
u( x, t) = π (17)
1 0

|
π π /2 π /2
cos(nx) 1 nπ
∫ f (x ) sen (nx) dx = ∫ sen (nx) dx = −
n
= −
n
[ cos(
2
) − 1]
0 0 0


2 1 nπ

2
y la solución del PVF resulta: u( x , t) = − π [cos( ) − 1] sen(n x ) e−n t
1 n 2

Al seleccionar algunos valores de tiempo, se obtienen los desarrollos en serie de la distribución de


temperaturas a lo largo de la varilla, para dicho instante; por ejemplo :

2 sen(3 x) −0.90 sen(5 x) −2.50 sen(6 x ) −3.60


u( x ,0.10) ≃ π [ senx e−0.20 + sen (2 x ) e−0.40 + e + e + e + ...........]
3 5 3

La representación de estas curvas de nivel, generan las funciones u(x), que se muestran en el
gráfico:

MATEMATICA APLICADA - MATEMATICA III – Cursada 2020/2021. MCIbarra - 9


Ejercicio 2: Resolver el PVF del Ejemplo 2, si la distribución inicial de temperatura es la indicada
en el gráfico:

4. Ecuación de Laplace

Sea una placa rectangular de dimensiones a x b; sobre la cual existe una distribución de

temperatura u(x,y) en régimen estacionario, dada por ∇ 2 u = 0 . Para encontrar la función u es

necesario en primer lugar resolver la ED y luego aplicar las condiciones de frontera; que para este
caso consisten en temperaturas constantes en los cuatro bordes de la placa, según se indica.

MATEMATICA APLICADA - MATEMATICA III – Cursada 2020/2021. MCIbarra - 10


Así, el PVF a resolver es:
δ 2 u δ 2u
+ = 0
δ x2 δ y 2

u(0 , y ) = 0 ; u(a , y ) = 0 ; u (x , 0) = 0 ; u( x , b ) = f ( x)

0< x ≤a ; 0 < y≤b

Resolución ED
X II Y II
u( x , y) = X ( x ) Y ( y) → X II Y + Y II X = 0 → =− =λ
X Y

X II − λ X = 0 ; Y II + λ Y = 0

Las condiciones de borde se pueden escribir como: X(0) = 0 ; X(a) = 0 ; Y(0) = 0

Caso 1: λ = 0
X II = 0 → X ( x) = c 1 + c 2 x ; X (0) = c1 = 0 ; X (a) = c2 a = 0 → c 2 = 0

Resulta como única solución, la Trivial: u = 0

Caso 2: λ = α2
X II − α 2 X = 0 → X (x ) = c 1 eα x + c 2 e −α x ; X (0) = c 1 +c 2 = 0 ; X (a) = c 1 e α a − c 1 e−α a = 0

c 1 [ eα a − e−α a ] = 0 → c 1 = c 2 = 0

También en este caso, la única opción es u = 0.

Caso 3: λ = - α2
X II + α 2 X = 0 → X (x ) = c 1 cos (α x ) + c 2 sen(α x) ; X (0) = c1 = 0

X (a) = c 2 sen (α a) = 0 → c 2 = 0 ó sen(α a) = 0

nπ nπ
Para conseguir una solución NoTrivial: sen (α a) = 0 → α = → X ( x) = c 2 sen ( x)
a a

MATEMATICA APLICADA - MATEMATICA III – Cursada 2020/2021. MCIbarra - 11


Y II − α 2 Y = 0 → Y ( y) = c 3 e α y + c4 e −α y ; Y (0) = c 3 + c 4 = 0 →

nπ nπ
y − y
a a
Y ( y ) = c3 e − c3 e

nπ −n π nπ −n π
nπ y y nπ y y
a a a a
un (x , y ) = c 2 sen ( x).[c 3 e − c3 e ] = A n sen( x ).[ e −e ]
a a

Por el Principio de Superposición, la función solución u(x,y) es la serie infinita:


∞ nπ −n π
nπ y y
u( x, y) = ∑ A n sen( x ).[ e a
−e a
] (18)
1 a

∞ nπ −n π
nπ b b
Aplicando la condición de borde en y = b: u( x, b) = ∑ A n sen ( x).[e a
−e a
] = f ( x)
1 a

De esta expresión, se reconoce a f(x) como un desarrollo en Serie de Fourier, en términos de la


función seno, con periodo a, cuyos coeficientes se obtienen como:

nπ nπ a
b − b
b n = A n [e a
−e a
] =
2
a
∫ f (x ) sen( naπ x ) dx
0

a
An = nπ
b
2


b
∫ f ( x ) sen( naπ x ) dx
a a 0
a (e −e )

Reemplazando en (18) se llega a la solución del PVF:


∞ a nπ −n π
2 1 nπ nπ y y
u( x , y) =
a
∑ nπ b −n π b
[ ∫ f (x ) sen (
a
x ) dx ] . sen (
a
x ).[e a
−e a
] (19)
1 a a 0
(e −e )

Ejemplo 3: Hallar la distribución de temperatura u(x,y) dada por (19) en una placa donde a = b = 1;
u(x, 1) = 100C.
∞ 1
1
u( x, y) = 20 ∑ [ ∫ sen (n π x ) dx ]. sen(n π x ).[ e n π y − e −n π y ]
1 (e n π −e−n π
) 0

MATEMATICA APLICADA - MATEMATICA III – Cursada 2020/2021. MCIbarra - 12


1
1
∫ sen (n π x) dx = −
n
[cos (n π ) − 1] → ≠ 0 para n impar
0


1
u( x, y) = 40 ∑ nπ −n π
. sen (n π x ).[ e n π y − e−n π y ] (20)
1 n (e −e )

Al hacer u = ctte, se obtienen las curvas de nivel de la superficie dada por (20), que representan las
diferentes isotermas en la placa.

MATEMATICA APLICADA - MATEMATICA III – Cursada 2020/2021. MCIbarra - 13

También podría gustarte